For Any Queries or Suggestions Email Us at or Whatsapp at For Free Study Material & Quizzes Visit

You might also like

Download as pdf or txt
Download as pdf or txt
You are on page 1of 68

For Free Study Material & Quizzes visit : www.mathsbyarunsir.

com
For any queries or suggestions email us @ mathsbyarunsir003@gmail.com or whatsapp @ 8881331466
For Free Study Material & Quizzes visit : www.mathsbyarunsir.com
For any queries or suggestions email us @ mathsbyarunsir003@gmail.com or whatsapp @ 8881331466
For Free Study Material & Quizzes visit : www.mathsbyarunsir.com
For any queries or suggestions email us @ mathsbyarunsir003@gmail.com or whatsapp @ 8881331466
For Free Study Material & Quizzes visit : www.mathsbyarunsir.com
For any queries or suggestions email us @ mathsbyarunsir003@gmail.com or whatsapp @ 8881331466
For Free Study Material & Quizzes visit : www.mathsbyarunsir.com
For any queries or suggestions email us @ mathsbyarunsir003@gmail.com or whatsapp @ 8881331466
Q.1 ? % 𝑜𝑓 24.91% 𝑜𝑓 39.93% 𝑜𝑓((11.93)2 + 5.98) = 2.25 Q. 16 (13.012)² + (21.025)² - 29.89 × 7.025 = ? – 520 +150
(a) 12 (b) 18 (c) 20 (a) 770 (b) 925 (c) 820
(d) 15 (e) 25 (d) 850 (e) 720
Q.2 (7.92) + (7.92)2 = −99.96 + (? )2
3 Q.17 18.05% of 1900.128 + ?% of 1149.89 = 684.025 – 111.89
(a) 26 (b) 24 (c) 36 (a) 35 (b) 25 (c) 20
(d) 16 (e) 25 (d) 40 (e) 30
439.92 3
32.99×41.89×4.98 Q.18 = (8.01) − (2.01)³ − (241.92 × 1.98)
Q.3 + 13.97 =? % 𝑜𝑓 279.88 ?
10.93×14.88 (a) 36 (b) 18 (c) 32
(a) 35 (b) 15 (c) 25
(d) 22 (e) 28
(d) 30 (e) 20
Q.19 −431.98 = 1239.81 + 482.21 − 1313.01
Q.4 96.96 + 996.96 − 99.66 + 999.96 + 5.96 = 399.96 × ?
(a) 29 (b) 33 (c) 39
(a) 8 (b) 7 (c) 6
(d) 19 (e) 23
(d) 5 (e) 4
Q.20 30.025 × √?+ √961.01 = 11.01% 𝑜𝑓 1300 − 22.21
Q.5 9.99% 𝑜𝑓 299.99 = (? )3 + (? )
(a) 25 (b) 9 (c) 16
(a) 1 > ? < 2 (b) 3 < ? > 6 (c) 4 > ? > 2
(d) 36 (e) 64
(d) 8 < ? > 7 (e)None of these 4
Q.21 ?3 × 17.98 + 12.03 % of 450.03 = (14.02)2 + √15.99
Q.6 (16.992)² + (21.012)² + √2915.83 = (?)²
(a) 9 (b) 2 (c) 5
(a) 26 (b) 28 (c) 22
(d) 8 (e)11
(d) 23 (e) 38 ? 3
Q.7 59.9% of 959.8 + 65.11% of 240.01 = ?% of 6099.8 Q.22 + (22.03)2 = (23.98)2 +√63.98
14.08
(a) 15 (b) 10 (c) 20 (a) 1344 (b) 1300 (c) 1296
(d) 12 (e) 7 (d) 1248 (e) 1440
Q.23 ? % of 1355.02 + 19.98% of 1210.01 = (27.99) 2
Q.8 √(12.9)2 + 27.98 ÷ 4.01 – (3.01)3 +? = (15.93) (a) 75 (b) 80 (c) 60
(a) 107 (b) 117 (c) 93 (d) 40 (e) 24
(d) 85 (e) 125 Q.24 ? + 35.09 % of 1279.98 = (24.03)2 + √195.98
Q.9 285.932 × 10.023 + 65.117 × 54.08 = ? + 163.99 (a) 142 (b) 148 (c) 156
(a) 6206 (b) 6220 (c) 6186 (d) 164 (e) 176
(d) 6236 (e) 6176 4
82.04 24.04
Q.25 56.03 % of ? + 125.02% of 96.03 = (13.98)2 − √1295.98
Q.10 × 655.98 × ? = 17.97 (a) 120 (b) 115 (c) 105
144.02
a) 815 b) 871 c) 808 (d) 125 (e) 135
d) 864 e) 829 Q.26 27.82% of 449.92 + ?% of 1199.86 = 199.89 + 225.92
Q.11 41.89% of 650.09 + (2.99)5 - √? = 2.13 × (4.09)4 (a) 45 (b) 25 (c) 40
(a) 18 (b) 16 (c) 20 (d) 28 (e) 35
(d) 24 (e) 12 Q.27 1729.79 + 1269.89 + ? = 249.91 × 19.83
(a) 3000 (b) 2800 (c) 1800
Q.12 ? % 𝑜𝑓 750.21 + (15.08)2 – 72.10 % 0f 449.93 = (7.98)2 + (2.03)5
(d) 2000 (e) 2200
(a) 42 (b) 16 (c) 22
(d) 26 (e) 32 Q.28 1149.89 + √? – 14.92 = 89.815 × 13.012
728.06 (a) 520 (b) 425 (c) 415
Q.13 × 15.02 + 23.89% of 550 – 37.96 = (22.09)2
? (d) 445 (e) 515
(a) 20 (b) 36 (c) 32
Q.29 (? ) + (11.79) + (6.01)2 + (8.12)3 = 499.825 + 448.02
2 2
(d) 24 (e) 28
(a) 32 (b) 26 (c) 24
Q.14 36.08× ? + 33.09 × 4.06 + √440.98 = (21.11)2 (d) 12 (e) 16
(a) 8 (b) 2 (c) 9
(d) 11 (e) 14 Q.30 √410.01 + 220.10– √24.98 =? +√225.05
Q.15 567.89 + 329.93 – (11.98)2 = 8.03% of ? (a) 10 (b) 25 (c) 5
(a) 9255 (b) 9425 (c) 9755 (d) 15 (e) 20
(d) 9625 (e) 10225

For Free Study Material & Quizzes visit : www.mathsbyarunsir.com


For any queries or suggestions email us @ mathsbyarunsir003@gmail.com or whatsapp @ 8881331466
Q31 (27.02)2 + 2669÷ 2.98 – 218.9 = 1869.6 - ? (d) 43 < ? < 23 (e) None of these
(a) 470 (b) 420 (c) 510 Q.47 √? = (1248.28 + 51.7) ÷ 99.9 − 7.98
(d) 540 (e) 500 (a) Multiple of 5 (b) Multiple of 7 (c) 13 > ? < 32
Q.32 17998÷ 4.99×1.52+ 7199.2 -3448.6 = ? (d) 43 < ? < 300 (e) None of these
(a) 9250 (b) 9080 (c) 9100 Q.48 35.99√?+ 32.0032√? =
68
× (? )
(d) 9150 (e) 9200 10.998
Q.33 3599÷8.98 +1244.9×2.98+(17.99) 2= ? (a) 113 < ? < 103 (b) 120 < ? > 112 (c) 133 > ? < 140
(a) 4400 (b) 4460 (c) 4500 (d) 143 < ? < 300 (e) None of these
(d) 4550 (e) 4360 Q.49 95.98 + (30.01)2 − 795.99 = (? )2 + 3.94
(a) Multiple of 7 (b) Multilple of 2 (c) Both A & B
Q.34 √7567.4× 4.52 +161.8 × 7.97= 2399.6 -?
(d) Multiple of 3 (e) None of these
(a) 800 (b) 640 (c) 660 1
(d) 760 (e) 710 Q.50 (15.95)4 + (3.01)3 − 111.99 × 2.02 + (9.98)2
Q.35 √16.9 × 44.9 + 4798.5 ÷ 3.98 + 59.8=? (a) Negative No. (b) Multiple of 5 (c) Both A & B
(a) 50 (b) 45 (c) 41 (d) 143 < ? < 300 (e) None of these
1 1
(d)37 (e) 54 Q.51 [(2197.02)3 + (343.99)3 ] = √(? )2 − 39.991 + 10.95
Q. 36 (2262.98 ÷ 31.01) × (510.01 ÷ 169.99) = + 59.91% of 159.988 (a) 13 < ? < 23 (b) 20 < ? > 22 (c) Both A & B
(a) 113 < ? < 123 (b) 120 < ? < 122 (c) 133 > ? < 150 (d) 3 < ? < 23 (e) None of these
(d) 143 < ? < 150 (e) None of these
Q.52 29.99% 𝑜𝑓 625.03
× 24.010 − 39.991 = ?3 − 21.04 × 22.06
Q.37 620.99 ÷ 23.01 + 28.11% of 749.899 = ? + 36.001% of 349.93 4.97% 𝑜𝑓 999.989

(a) 112 (b) 10 2+ 11 (c) 92 + 8 (a) 13 < ? < 23 (b) 20 < ? > 22 (c) Both A & B
(d) 121 (e) None of these (d) 43 < ? < 23 (e) Multiple of 2
Q.38 (89.99)2 – (67.03)2 – 64.99% of 780.02 = 31.93 × ? Q.53 79.998% 𝑜𝑓 2499 + 226.067 − 412.001 − 4.034 =?2 + (20.99)2
(a) 23 < ? < 33 (b) 20 < ? > 40 (c) Both A & B
(a) 113 (b) 97 (c) 109
(d) 13 < ? < 45 (e) None of these
(d) 87 (e) 107
Q.54 ? % 𝑜𝑓 (843.05 − 126.054 − 116.054)= 522.02
Q.39 √89.99 × 23.99– (112.01 × 17.9) + 480.93 = ?
(a) 93 < ? < 23 (b) 50 < ? > 22 (c) Both A & B
(a) Multiple of 5 (b) Multiple of 3 (c) Multiple of 4
(d) Only B (e) None of these
(d) Multiple of 7 (e) None of these
Q.40 ?² + 29.07 × 5.114 – 115.117 = 8.9% of 400.118 + 522.9 Q.55 ? +150.01% 𝑜𝑓 249.89 = 45.978 × 12.003
(a) Multiple of 5 (b) Prime No. (c) Non Prime No. (a) 113 < ? < 223 (b) 110 < ? > 22 (c) Both A & B
(d) Multiple of 7 (e) None of these (d) Multiple of 5 (e) None of these
Q.41 [(√530 × 36.003) ÷ 47.987] ×? = 5863.10376 Q.56 125.09 % of 440.01 + 74.98 % of 839.98 + 3√7.99 = 39.89 × ?
(a) 113 < ? < 123 (b) 120 < ? > 122 (c) 133 > ? < 320 (a) 13 < ? < 23 (b) 20 < ? > 22 (c) Both A & B
(d) 143 < ? < 300 (e) None of these (d) 3 < ? < 23 (e) None of these
Q.42 (77.987% 𝑜𝑓 358) + (68.55% 𝑜𝑓 729) = ? Q.57 ? × 128.09 + 1728.09 = (12.99)3 + 170.99
(a) Multiple of 2 (b) Multiple of 10 (c) Multiple of 5 (a) Prime No. (b) Multiple of 5 (c) Both A & B
(d) 780 (e) All of these (d) 3 < ? < 23 (e) All of these
Q.43 √624.995 + (4.9989)2 = ? ÷ 4.9900865
1 Q.58 ?3 × 15.02 + 125 % 𝑜𝑓 463.94 = (38.01)2 + 95.98
(a) 13 < ? < 23 (b) 20 < ? > 22 (c) Both A & B
(a) 9 < ? < 3 (b) 12 > ? < 122 (c) 13 < ? < 15
(d) Multiple of 2 (e) None of these
(d) 23 < ? < 150 (e) None of these
Q.59 1267.98 + ?3 = (12.02)3 + 51.98
Q.44 989.001+ 1.00982 × 76.792 = ?
(a) Multiple of 4 (b) Multiple of 2 (c) Both A & B
(a) 1150 (b) 1070 (c) 1240
(d) 3 < ? < 23 (e) All of these
(d) 1188 (e) 1044 ?
Q.45 63.9872 × 9449.8780 ÷ 243.003 = (?)² Q.60 + (11.97)2 − √1936.01 = (15.98)2
14.09
(a) Multiple of 2 (b) Multiple of 10 (c) Multiple of 5 (a) 2164 (b) 2196 (c) 2118
(d) 50 (e) All of these (d) 2184 (e) 2124
1
420.12
Q.46 √1024.002 ÷ 3.996 ÷ 9.98 + 29 = ? Q.61 = (361.11)2 − 22.01 × 6.99 + 141.99
?
(a) 113 < ? < 123 (b) 20 < ? > 22 (c) 33 > ? < 20

For Free Study Material & Quizzes visit : www.mathsbyarunsir.com


For any queries or suggestions email us @ mathsbyarunsir003@gmail.com or whatsapp @ 8881331466
(a) 13 < ? < 23 (b) 20 < ? > 22 (c) Both A & B (a) 12000 (b) 22000 (c) 16000
(d) 43 < ? < 23 (e) None of these (d) 18000 (e) 8000
Q.62 ? + 185.10 – 79.09 = (23.01)² - 70.01% of 139.99 Q. 77 (2.99 × 3.99 × 4.99) ÷ 35.99 = (? )2 – 79.99
2 2

(a) 113 < ? < 330 (b) Multiple of 5 (c) Both A & B (a) 12 (b) 20 (c) 10
(d) 43 < ? < 23 (e) None of these (d) 25 (e) 15
Q.63 √783.98 + (22.02)2 = 2 × (? )2 Q.78 324.95 – [(11.99) – 74.9] = (? )2 – 67.99
2

(a) Multiple of 2 (b) Multiple of 4 (c) Square of 4 (a) 22 (b) 18 (c) 28


(d) 15 < ? > 10 (e) All of these (d) 38 (e)42
Q.64 44.04% of 349.98 + 205.01% of 140.01 = (?)² Q.79 34.99 × 17.98 ÷ 6.97 – 72.93 = ? – 128.99
(a) Multiple of 9 (b) Multiple of 3 (c) Cube of 3 (a) 132 (b) 156 (c) 172
(d) 21 (e) Both B & D (d) 164 (e) 146
?
Q65 4
+ 44.01 + 139.99 = 78.09 + 249.01 + 86.99 Q.80 15.98 × 49.97 − 18.03 × 32.01 + (14.01)2 = ?
(a) 113 < ? < 823 (b) 240 < ? > 222 (c) 333 > ? < 120 a) 400 b) 450 c) 350
(d) 430 < ? < 223 (e) None of these d) 380 e) 420
Q.66 11.922 + 15.96 of 2.93 – 19.92 ÷ 3.99 = ? Q.81 13.99 ×? +695.01 = 36.03% of 2399.98 + 754.99
(a) 187 (b) 205 (c) 117 a) 72 b) 66 c) 75
(d) 165 (e) 198 d) 80 e) 62
Q.67 851.92 - 12.93 × 7.98 - 101.92 × 2.93 - 0.91 = ?2 Q.82 17.99% of 199.98 + ?% of 600.01 = 39.99% of 750.01
a) 19 b) 17 c) 15 a) 48 b) 44 c) 50
d) 21 e) 27 d) 30 e) 38
Q.68 3.94 + 1/12.93 of {2599.96 ÷ (117.98 – 107.98)} = ? Q.83 339.01 + 211.01 − 380.01 = ? − 320.01
(a) 16 (b) 20 (c) 32 a) 490 b) 440 c) 380
(d) 28 (e) 24 d) 520 e) 550
3 Q.84 286.97 × 56.97 + 240.91 = √624.95 × ?
Q.69 (63.94 × 15.96 ÷ 255.96)2 = √[3.94(? −3) ]
(a) 624 (b) 616 (c) 632
a) 11 b) 13 c) 9 (d) 648 (e) 664
d) 7 e) 21
1 1
Q.85 89.9% of 400.01 + 14.99% of 4800.01 – 676.99 = ?
Q.70 (25.11)2 × (80.87)1/4 × (49.22)2 + 3.13 × 7.01 + 2 = 2? a) 413 b) 403 c) 419
a) 2 b) 4 c) 7 d) 395 e) 389
d) 9 e) 10 Q.86 49.99 ×? + 12.01 × 44.99 = 759.99 + 2.99 × 60.01
3
Q. 71 (18.10 × 5.05 – 10.098) – ( 4 of 15.83) = 33.35 + ? a) 4 b) 8 c) 12
A) 37 B) 28 C) 30 d) 16 e) 20
D) 46 E) 35 Q.87 (2? ) + 499.99% of 112.01 = 1239.99 - 8.01 × 34.99
2

1 a) 18 b) 12 c) 15
Q.72 (66)3 + (3.01)3 – 69.98 × 2.02 + 30% of √6399 = ? d) 14 e) 10
(a) – 85 (b) 85 (c) – 75 ?
Q.88 1639.99 ÷ 41.01 + 3.99 % of 999.990 = 779.9909
(d) 75 (e) – 95
Q.73 90.77 + 116.81 ÷ 13.40 – 44.16 = 152.67 - ? a) 286 b) 278 c) 318
A) 97 B) 157 C) 107 d) 296 e) 306
D) 119 E) 87 Q.89 771.999 + 437.99
2.01
− 80.99 × (1.99)3 = (? )3
a) 8 b) 4 c) 6
Q.74 √? × 45.16 = 250.08 × 15.01 + 138.21 × 24.88
d) 7 e) 5
A) 25600 B) 26500 C) 160
Q.90 28.01% of 224.99 + (9.01)2 = ?2
D) 50 E) 2560
(a) 8 (b) 16 (c) 24
Q.75 [1279.98)2 ÷ 32.23 × 23.94] ÷ 47.98 = ?2 (d) 12 (e) 6
(a) 120 (b) 96 (c) 150 ?
(d) 84 (e) 160 Q.91 + (14.01)2 = (15.01)2 − 3√124.99
18.01
3 (a) 432 (b) 412 (c) 402
Q.76 599.99 × 2 𝑜𝑓 19.92% 𝑜𝑓 (109.99 – 9.99) = ?
(d) 442 (e) 472

For Free Study Material & Quizzes visit : www.mathsbyarunsir.com


For any queries or suggestions email us @ mathsbyarunsir003@gmail.com or whatsapp @ 8881331466
Q.92 32.01 × ? + (18.01)2 = 80.01% of 1404.99 Q107 √1521.02 × 169.01 − (26.99)2 + 418.001 = (? )2
(a) 35 (b) 25 (c) 15 a) 8 b) 18 c) 11
(d) 45 (e) None of these d) 14 e) 19
Q.93 56.01 % of ? + (11.99)2 = 68.01 % of 499.99 Q.108 148.01 + 832.01 ÷ 6401 − 97.03 = ?3
(a) 250 (b) 300 (c) 150 a) 4 b) 6 c) 5
(d) 450 (e) 350 d) 7 e) 3
Q.94 ?3 × 18.08 + √625.01 = 2274.98 Q.109 79.98% of 399.99 + ?% of 650.03 = 580.02
(a) 15 (b) 25 (c) 35 a) 50 b) 70 c) 30
(d) 5 (e) 45 d) 60 e) 40
Q.95 𝑥% 𝑜𝑓 (800.1 − 419.8) + 13 = 260 3
Q.110 199.99 + 12.01% of 499.99 - √63.99 =?2
(a) 50 (b) 95 (c) 90 a) 12 b) 16 c) 14
(d) 65 (e) 80
67.98+x
d) 18 e) 8
Q.96 97.97 + 3.99 = 137.95 84.01 4
Q.111 ? + 26.03% of 799.97 + √16.03 = (5.99)3
(a) 89 (b) 98 (c) 83
a) 42 b) 21 c) 12
(d) 106 (e) 92
256.02 d) 14 e) 4
Q.97 − 12.08 = √𝑥 Q.112 ? % of 549.99 + 20.03 % of 649.99 = 349.99
15.97
(a) 16 (b) 81 (c) 121 a) 36% b) 24% c) 20%
(d) 64 (e) 4 d) 30% e) 40%
Q.98 (4.97)2 + 𝑥 = √3035 Q.113 5.03 ×(? +12.01)+29.99 % of 899.99= 359.99
(a) 36 (b) 39 (c) 30 a) 10 b) 8 c) 6
(d) 24 (e) 29 d) 4 e) 12
Q.99 48.98 + 𝑥 − 32.11 = (8.93)2 Q.114 36.01% of ? + (18.01)2 = 773.98
(a) 59 (b) 52 (c) 64 a) 1250 b) 1050 c) 750
(d) 67 (e) 73 d) 1750 e) 500
16.01 1088.02 204.001 Q.115 189.98 + 49.96% of 539.98 + ? = 630.03
Q.100 ? × 35.04 = 629.98 ÷ 648.01
a) 8 b) 20 c) 16 a) 190 b) 170 c) 120
d) 240 e) 200
d) 12 e) 24 276.02 79.99
1 1
Q.101 [(729.02)3 + (1024.01)2 ] = ?2 + 5.005 Q.116 × 115.01 ×? = 31.99% of 199.98
12.03
a) 12 b) 5 c) 8 a) 4 b) 16 c) 30
d) 10 e) 6 d) 8 e) 24
Q.117 (284.02 + 184.001 ÷ 3.997) ÷ (594.01 ÷ 9.02) = ?
Q.102 36.02% of 1499.990 ÷ 18.01 = ? − 112.004
a) 25 b) 15 c) 20
a) 64 b) 112 c) 188
d) 5 e) 10
d) 142 e) 228
15.003 Q.118 684.03 + 2171.999 ÷ 5.98 + 49.98 × 5.02 = ?2
Q.103 49.99− ? × 59.99% of 800.01 = 240.04
a) 42 b) 36 c) 24
a) 40 b) 20 c) 10 d) 26 e) 16
d) 50 e) 30
1 2.01 Q.119 (79.97% of 250.02) of 4.99 ÷ (4.97)2 = ?
Q.104 [(2196.99)3.03 ] =? − 147.02 a) 40 b) 70 c) 90
a) 316 b) 329 c) 372 d) 60 e) 20
d) 248 e) 296 Q.120 66.01 + 39.97% of 475.01 = (?)²
Q.105 79.98% of 550.02 + 44.98% of 799.99 =?2 × 7.99 a) 19 b) 17 c) 15
a) 16 b) 10 c) 15 d) 14 e) 16
d) 12 e) 21 Q121. (√501.97 + 173.99) × 15.01 = ? + 219.99
9.03 194.99 659.998 a) 175 b) 170 c) 185
Q.106 of of = ? − 64.02
75.01 11.02 35.97
a) 121 b) 118 c) 103 d) 180 e) 165
d) 111 e) 95

For Free Study Material & Quizzes visit : www.mathsbyarunsir.com


For any queries or suggestions email us @ mathsbyarunsir003@gmail.com or whatsapp @ 8881331466
Q.122 (√224.99 − √81.01) × (√15.96 − √9.01) = ? Q. 137 (?)² + 89.98% of 1999.92 + 5.01 × 79.89 = 129.89 × 20.01
a) 6 b) 2 c) 4 (a) 25 (b) 20 (c) 30
d) 8 e) 10 (d) 45 (e) 40
479.99 Q.138 1589.92 ÷ 2.892 + ? % of 139.89 = 179.82 × 8.08
Q.123 ? + 24.9% of 500 - 89.99 = √3024 (a) 750 (b) 830 (c) 800
a) 20 b) 22 c) 30 (d) 610 (e) 650
d) 36 e) 24 Q.139 839.829 + 1259.925 – 119.98 × 19.89 = 749.89% of 1.892 - ?
Q.124 65.02% of 240.01 + √1935 = ? (a) 450 (b) 545 (c) 425
a) 180 b) 200 c) 210 (d) 315 (e) 310
d) 215 e) 190 Q.140 15.02% 𝑜𝑓 99.99 + √143.98 + (3.01)2 = (? )
Q.125 342.09 + 575.98 − ?2 = (12.09)2 -√99.95 a) 4 b) 6 c) 3
(a) 28 (b) 24 (c) 16 d) 8 e) 11
(d) 32 (e) 36 Q.141 29.98% 𝑜𝑓 1199.998 + (? )3 = (23.98)2
Q.126 24.09 × ? + 11.98 % 𝑜𝑓 224.99 = (19.02)2 + √676.09 a) 3 b) 5 c) 2
(a) 5 (b)10 (c) 20 d) 1 e) 6
(d) 25 (e) 15 √59.9+(13.98) 2
Q.142 +(? )2 = 199.99% 𝑜𝑓 289.99
Q. 127 (12.01)3 + ?2 = 35.98 % of 5200.08 3.98
(a) 8 (b) 12 (c) 16 a) 24 b) 22 c) 20
(d) 24 (e) 20 d) 26 e) 28
1
Q.128 472.09 + 548.09 − ?2 = (18.02)2 + √399.98 Q.143 [104.99 + 211.99 − 27.98]2 + (1.99)3 = (? )2
(a) 26 (b) 29 (c) 33 a) 2 b) 3 c) 4
(d) 36 (e) 40 d) 5 e) 6
Q.129 121.09 + ? = 96.02 % of 499.98 Q.144 (58.86 + 85.86) ÷ 4.89 = (? )3 + (? ) − 1
(a) 350 (b) 348 (c) 342 a) 1 b) 2 c) 3
(d) 336 (e) 359 d) 4 e) 5
Q.130 29.92% of 180.01 + (17.06)² = ? % of 8574.90 Q.145 31.67 × ? + 22.122 = 174.75% of 240.03 + 56.09 × 7.99
(a) 2 (b) 6 (c) 10 a) 12 b) 44 c) 96
(d) 4 (e) 8 d) 150 e) 1
Q. 131 (32.02)² = (?)3 – (14.89)² + 15.98 % of 1600.20 + 13.02% of Q.146 ?2 = (540.09 × 179.98) ÷ (24.082 – 11.782) + 16.05% of 2500.01
4999.99 A) 75 B) 52 C) 8
(a) 7 (b) 15 (c) 4 D) 25 E) 38
(d) 12 (e) 9 5.01
Q.147 {(6.92 ) × 839.890 – (4200.001 ÷ 34.900)} × 4.98 = ? + 1599.899
Q.132 52.03 × 17.89 – √2303.98 = (29.99)? - 5% of 239.987 A) 800 B) 420 C) 540
(a) 3 (b) 2 (c) 4 D) 360 E) 720
(d) 0.5 (e) 1 Q. 148 (155.34 + 234.78) ÷ 12.65 = ? ÷ 4.99
455.93
Q.133 ? = 12.98 × (1.99)5 – 44.99% 𝑜𝑓 840.1 a) 150 b) 240 c) 470
(a) 15 (b) 9 (c) 18 d) 20 e) 680
(d) 21 (e) 12 Q.149 {44.78% of (11.67 × 54.78)} ÷ 2.65 = ? + 19.25
A) 200 B) 120 C) 40
Q.134 √831.01 − √2208.97 = √?+ √529.9 D) 80 E) 160
(a) 25 (b) 36 (c) 49 Q.150 ?% of 7999.97 + (41.07)2 – 29.98% 𝑜𝑓 4149.89 = (25.91)2
(d) 16 (e) 9 (a) 5 (b) 3 (c) 2
Q.135 23.82% of 549.89 + 89.98% of 909.9 = ? + 251 (d) 4 (e) 1
(a) 780 (b) 820 (c) 700 249.83+?
Q.151 49.97 + 416.99 + (19.87)2 = (24.87)2 + 20.09 × 9.98
(d) 650 (e) 750
?
Q.136 299.80 × 8.82 + 149.89 × 19.98 = 420.01 × 29.98 + 350.21 × 40.01
(a) 154 (b) 150 (c) 158
(a) 708 (b) 908 (c) 882 (d) 162 (e) 156
(d) 742 (e) 812 Q. 152 (21.87)² + 12.493 × 19.89 – √15624.98 – (?)² of 3.96 = (14.96)²

For Free Study Material & Quizzes visit : www.mathsbyarunsir.com


For any queries or suggestions email us @ mathsbyarunsir003@gmail.com or whatsapp @ 8881331466
(a) 4 (b) 6 (c) 12 (d) 286 (e) 216
(d) 8 (e) 10 Q. 167 (7.98)³ + (14.88)² – (12. 01)² = ? – 1219.812 – 1749.98
Q.153 9899
× √678 + 485 × 0.01 + 97.8 = 80% 𝑜𝑓 ?
(a) 3643 (b) 3425 (c) 3416
10.98 ×√9.03
(d) 3563 (e) 3521
(a) 240 (b) 260 (c) 250
(d) 230 (e) 270 Q.168 19.825 × √? = 63.91% of 399.98 + 11.95% of 1200.01
Q.154 (a) 300 (b) 500 (c) 420
5435.99 (d) 350 (e) None of these
+ √323.898 + 53.89 + 35.98% 𝑜𝑓 1000 Q. 169 (?)² + 14.01% of 1599.98 = 59.01 × 12.025
302.12
108.11 (a) 18 (b) 28 (c) 22
= ×? + 216.05
5.99 (d) 36 (e) 32
(a) 10 (b) 12 (c) 15
(d) 13 (e) 18 Q.170 149.78% of 15.89 + √255.81 × 3.95 = ? – 139.59
Q.155 3√1330.99 + 43.98 − 14.95+? = 188.88 (a) 248 (b) 212 (c) 218
(a) 209 (b) 189 (c) 165 (d) 228 (e) 232
(d) 149 (e)139 Q.171 ? + 1349.71 ÷ 2.99 – 124.82 = 120.03% of 1649.82
(21.87) 2 +448.98−32.80
(a) 1655 (b) 1755 (c) 1720
Q.156 = 8.98 (d) 1225 (e) 1680
(4.99) 3 + 3√511.94−?
(a) 33 (b) 53 (c) 63 Q. 172 (?)² + 180.21 × 4.9 + 64.8 × 3.8 = 2384.78
(d) 101 (e)163 (a) 40 (b) 30 (c) 35
Q.157 (45.75)2 + 1015.98 ÷ 3.95+? = (13.97)3 (d) 25 (e) 45
(a) 473 (b) 374 (c) 743 Q.173 ? % of 299.71 = (21.03)² + (18.89)² + (6.03)3 + 2.01
(d) 466 (e) 334 (a) 225 (b) 280 (c) 250
78.78+87.87+788.87−877.78 (d) 325 (e) 340
Q.158 = 12.91
?
(a) 9 (b) 2 (c) 18 Q.174 √? + 789.81% of 119.79 + 199.81 = 2180.01 – (31.81)²
(d) 15 (e) 6 (a) 100 (b) 121 (c) 64
Q.159 3√728.88 + (? )3 − (4.87)2 = 41.91 + 5.88 (d) 144 (e) 81
(a) 5 (b) 7 (c) 6 Q.175 ?² + 190.98 – 19.01 × 6.97 = 316.99 × 1.99
(d) 4 (e)3 (a) 14 (b) 19 (c) 24
Q.160 119.79% of 119.89 + 70.12% of 79.73 = ?% of 999.89 (d) 26 (e) 16
(a) 2.5 (b) 20 (c) 15 Q.176 768.06 ÷ 11.97 × √256.05 −58.05 = ?
(d) 35 (e) 40 (a) 1033 (b) 1175 (c) 966
Q.161 1.823 + 5.126 × 7.01 – 432.001 = ? – 531.99 (d) 880 (e) 975
(a) 155 (b) 126 (c) 137
Q.177 34.02% of 550.09 ÷ ? = 297.07 ÷ √728.95
(d) 120 (e) 145
22 (a) 14 (b) 21 (c) 8
Q.162 41.98 × 7 + 19.89% 𝑜𝑓 529.87 – 43.01 =? (d) 27 (e) 17
(a) 177 (b) 185 (c) 212 Q. 178 (? ÷ 9.97) 12.08 = 20.12% of 1319.97
(d) 205 (e) 195 (a) 220 (b) 240 (c) 260
Q. 163 (23.12 × 22.98) + 11.89 × 7.79 = ?² (d) 280 (e) 200
(a) 20 (b) 25 (c) 31
(d) 22 (e) 30 Q.179 ? % 𝑜𝑓 179.99 = √(24.02)2 + (17.98)2 + 60.01% 𝑜𝑓 659.98
1 (a) 80 (b) 60 (c) 40
Q.164 25% of 2000.32 + 2000.1% of 25 – 33 % 99.9 = ? + 50.1
3 (d) 20 (e) 10
(a) 917 (b) 906 (c) 910
Q.180 √(6.93)2 + (17.01)2 + (5.13)2 − 1.99 = ?
(d) 920 (e) 927
(a) 29 (b) 21 (c) 19
Q.165 √63.82 × 36.01 + 419.92 ÷ 5.84 – 540 = ? – 799.98 (d) 11 (e) 13
(a) 426 (b) 378 (c) 526 Q.181 15.98% of 350.11 + 46.01% of 4449.89 – ?= 1782.899
(d) 328 (e) 448 (a) 310 (b) 320 (c) 315
Q.166 15.812% of 1600.125 + ? % of 1199.98 = 19.88 × 121.98 (d) 325 (e) 335
(a) 182 (b) 142 (c) 326

For Free Study Material & Quizzes visit : www.mathsbyarunsir.com


For any queries or suggestions email us @ mathsbyarunsir003@gmail.com or whatsapp @ 8881331466
Q.182 8475.98 ÷ ? × 45.01 + 31.93 = 5899.892 (a)100 (b) 105 (c) 110
(a) 65 (b) 75 (c) 55 (d) 115 (e) 95
(d) 60 (e) 51 266.47 17
Q.197 19.66% 𝑜𝑓 144.87 − 118.84 × 18.84 =?3
Q.183 ?% of 539.932 – 78.13 = 14.99% of 920.011
(a) 5 (b) 7 (c) 0
(a) 50 (b) 45 (c) 30
(d) 3 (e) 6
(d) 40 (e) 35 181 473.92 1

Q. 184 (215.98)³ ÷ (36.01)² = (5.9)? × 36.01 Q.198 √81.02 ÷ 8.76 ÷ 9.25 × 3.76 + 91.10 ×
237.40
+ 7.91 =?2
(a) 1 (b) 5 (c) 2 (a) 100 (b) 400 (c) 256
(d) 7 (e) 3 (d) 169 (e) 196
Q.185 62.01% of ? + (24.04)2 = (30.99)2 − √169.01 Q.199 6739 + 161 × 4.966 − 74.99 × 11.888 +
(a) 660 (b) 600 (c) 640 20%𝑜𝑓121 − 10%𝑜𝑓109 =?
(d) 720 (e) 780 (a) 6657 (b) 7067 (c) 7167
(d) 6587 (e) 6757
Q. 186 (3.01)? × 4.01 + 36.01% of 2099.98 = (12.03)3
(a) 1 (b) 4 (c) 2 Q.200 ? % of 625.03 + 20.08% of 954.98 = (20.99)2
(d) 3 (e) 5 (a) 35 (b) 30 (c) 48
7 (d) 40 (e) 50
Q.187 1295.98 + 219.09 × = 138.99 × 13.02
?
(a) 5 (b) 6 (c) 8 Q.201 (5.01)? + (14.98)2 = 169.98% of 500.2
(d) 9 (e) 3 (a) 4 (b) 3 (c) 2
Q.188 ?% of 1224.98 + 36.01 % of 2500.02 = (33.98)2 + 19.01% of 199.97 (d) 5 (e) 6
3
(a) 24 (b) 20 (c) 16 Q.202 28.09% of ? + (12.09)2 = √1331.01 + 29.99% of 530.04 + (9.99)2
(d) 12 (e) 28 (a) 425 (b) 400 (c) 360
(d) 450 (e) 480
Q.189 37.09 % of ? + (10.99)² – 28.88 × 5.99 + √224.98 = (31.01)²
Q.203 510.99 + (? ) = (28.01)2 + √256.01
2
(a) 2650 (b) 2600 (c) 2750
(a) 18 (b) 19 (c) 17
(d) 2700 (e) 2760
1 1
(d) 16 (e) 15
Q.190 [(1763.97)2 + (2197.04)3 ] = √(? ) 3 + 17.12 + 4.021 × 7.980 725.01
Q.204 42.01% of 799.99 + ? = (18.98)2
(a) 15 (b) 8 (c) 12
(a) 27 (b) 25 (c) 50
(d) 5 (e) 10
(d) 75 (e) 29
Q.191 34.971% 𝑜𝑓 80.13 × 20.03% 𝑜𝑓 250.01 − 423.87 = ? +
Q. 205 (24.02)2 - 16.07 = ? × 19.95
51.991 × 8.13
(a) 580 (b) 550 (c) 570 a) -47 b) -62 c) 28
(d) 560 (e) 590 d) -107 e) 73
Q.206 15.052 + 47.052 + 24.972 = ?
Q.192 45.0112 ÷ 64.992 × 312.211 + 361.11 = ? ² + 11.09 × 32.13
a) 3239 b) 3099 c) 2919
(a) 15 (b) 45 (c) 35
d) 2879 e) 3059
(d) 55 (e) 25
Q.193 ? % 𝑜𝑓 (1131.145 − 501.923 − 241.02 − 187.79) = 59.99456
Q.207 √780 + √1442 − √290 = ?
(a) 30 (b) 50 (c) 40 a) 74 b) 69 c) 64
(d) 60 (e) 70 d) 49 E )34
Q.194 (? − 554.09) − 50.045% 𝑜𝑓 (
6450.01
) = 35.79 × 41.12 ÷ 369.03 Q.208 1926.98 ÷ 47.07 × 41.03 = ? + 27.05
42.91
(a) 647 (b) 537 (c) 679 a) 1639 b) 1549 c) 1654
(d) 511 (e) None of the above. d) 1579 e) 1699
25 % 𝑜𝑓 295.7×32.02 36.99 Q.209 31.05 × 27.05 - 26.99 = ? × 30.06
Q.195 = 1
? 25% 𝑜𝑓
4
a) 10 b) 25 c) 31
(a) 2 (b) 4 (c) 6 d) 27 e) 30\
1331.01
(d) 8 (e) 1 Q210. 11.02 × 8.98 + + (30.95)2 =?
3 √120.95
√404.99×315.01×245.40
Q.196 =? (a) 1071 (b) 1041 (c) 1181
1
√33 % 𝑜𝑓 26.99
3 (d) 1161 (e) 1211

For Free Study Material & Quizzes visit : www.mathsbyarunsir.com


For any queries or suggestions email us @ mathsbyarunsir003@gmail.com or whatsapp @ 8881331466
251.9 (d) Multiple of 6 (e) None of the Above
Q.211 √960.9 + √143.9 +? % 𝑜𝑓 350.1 = (1.99)2 × 69.01
(a) 58 (b) 32 (c) 64 Q.227 ?2 + 114.09 − 24.06 × 4.98 = 163.19
(d) 74 (e) 86 (a) Prime No. (b) Multiple of 3 (c) Multiple of 7
419.97
Q.212 77.2 × 69.87 + 499.9 – 10.98 % 𝑜𝑓 599.99 = (d) Multiple of 13 (e) Both A & D
Q.228 219.89% of 380.13 + (35.96)² – ? % of 399.89 = (23.89)²
? % 𝑜𝑓 5600.01
(a) 302 (b) 284 (c) 342
(a) 20 (b) 25 (c) 16
(d) 10 (e) 30 (d) 389 (e) 430
Q. 229 (39.22)² = ? + (29.98)² – (15.89)² + 55.98 % of 1399.89
Q.213 √2499.89 × √625.02 ÷ √99.99 =? % 𝑜𝑓 2500.01 (a) 72 (b) 112 (c) 63
(a) 5 (b) 10 (c) 12.5 (d) 97 (e) 93
(d) 2 (e) 8
Q. 230 (31.87)² + 12.093 × 15.89 – √2115.98 – (?)² = (32.87)² - 39.89
Q.214 2950.01 ÷ 59.06 = ? + 28.03
(a) 7 (b) 11 (c) 15
a)15 b)25 c)14
(d) 19 (e) 21
d)30 e)22 479.93
Q.215 3√2743.69 − 3√10649.11 + 3√1727.63 = ? Q.231 ? = (8.99)3 – 14.98 × 39.89 – 9.89% 𝑜𝑓 990
a) 18 b) 53 c) 4 (a) 16 (b) 9 (c) 19
d) -38 e) -17 (d) 25 (e) 12
Q. 216 (10.97)2 + (27.99)2 - (31.96)2 = ? Q.232 √429.87 + 520.13 + √120.97 = (? )2 + √35.98
a) -119 b) -136 c) -107 (a) 2 (b) 3 (c) 8
d) -99 e) -110 (d) 5 (e) 9
Q.217 11.93 × 18.95 − 2.99 = ? × 14.94
Q. 233 (17.291) × (18.717) + (25.99)² = (? )³ – 1.005
a) 8 b) 16 c) 26
(a) 18 (b) 10 (c) 12
d) 19 e) 15
(d) 8 (e) 15
Q.218 39.998% of 440.005 +? % of 654.889 = 229.81
1 1
(a) Multiple of 2 (b) Multiple of 3 (c) Multiple of 4 Q.234 [(511.9)3 + (728.89)3 ] = √(? )2 + 24.991 + 3.845
(d) Both A & C (e) All of the above (a) 10 (b) 8 (c) 16
Q.219 40.012% of 599.87 – 250.17 =? – 79.88% of 900.11 (d) 12 (e) 20
(a) Multiple of 5 (b) Multiple of 4 (c) Multiple of 9 ?−4.981
(d) Both A & C (e) None of the above Q.235 × 39.9% 𝑜𝑓 649 = 65
14.99−?
Q.220 324.996 × 15.98 ÷ 4.004 + 36.876 = ? % of 6699.98 (a) 10 (b) 12 (c) 15
(a) Multiple of 5 (b) Multiple of 4 (c) Multiple of 10 (d) 7 (e) 18
(d) Both A & C (e) All of the above Q.236
19.8% 𝑜𝑓 624
× 342.11 = ?2 − 12.79 × 2.91
Q.221 63.8% of 8899.78 + ? % of 5300.11 = 6849.889 29.7% 𝑜𝑓 570
(a) Multiple of 2 (b) Multiple of 11 (c) Multiple of 7 (a) 11 (b) 17 (c) 13
(d) Both A & B (e) None of the above (d) 15 (e) 19
2
Q.222 √23408.98 % of 799.93 – 624.001 = √? × 23.95 + (12. 011) Q.237 79.98% 𝑜𝑓 ? = √249.98 × 44.04 + 40% 𝑜𝑓 8499
(a) 225 (b) 361 (c) 289 (a) 80 (b) 120 (c) 150
(d) 324 (e) 441 (d) 180 (e) 240
Q.223 363.72 – 269.21 + 49.987 of 11.89 ÷ (19.9% of 80.1) = ? 593.93 2299
(a) Multiple of 7 (b) Multiple of 14 (c) Multiple of 19 Q.238 ? × 40.01 ÷ 24.02 × 27.05 = 114.94 × 263.98
(d) Both A & C (e) None of the above (a) 1 (b) 2 (c) 3
Q.224 {18.99² – 27.05² + (1399.95 + 59.85)} ÷ ? = 39 (d) 4 (e) 5
(a) Multiple of 7 (b) Multiple of 4 (c) Multiple of 9 Q.239 19.99% 𝑜𝑓 (40.01 × √? ) = (31.99)2 + (16.01)²
(d) Both A & B (e) None of the above (a) 160 (b) 2560 (c) 16
Q. 225 (21.97% of 899.5) + ? + (39.77% of 309.76) =481 (d) 25600 (e) 256
(a) Multiple of 7 (b) Multiple of 3 (c) Multiple of 4 Q.240 ? + 13.02 × 49.98 = 420.05 + 44.98% of 799 + 220.10
(d) Multiple of 5 (e) None of the above (a) 300 (b) 350 (c) 400
Q.226 9.89% of 650.11 + 32.1 × 2799 ÷ 399.8 – ? = 155 (d) 450 (e) 250
(a) Prime No. (b) Multiple of 3 (c) Multiple of 7

For Free Study Material & Quizzes visit : www.mathsbyarunsir.com


For any queries or suggestions email us @ mathsbyarunsir003@gmail.com or whatsapp @ 8881331466
1331.01 Q.255 ?% of 1049.87 + 74.99% of 420.12 = 750.11% of 70
Q.241 11.02 × 8.98 + + (30.95)2 =?
√120.95 (a) 15 (b) 20 (c) 10
(a) 1071 (b) 1041 (c) 1181
(d) 35 (e) 25
(d) 1161 (e) 1211
350.98 (3.02)2 1
Q.256 √324.11 × √19.98 × 49.99 × 8.01 × 20.01 +
Q.242 √88.01 × 21.99 + 12.98 + 4−2 =? +(243.01)5
25.17% 𝑜𝑓 31.9 =?
(a) 234 (b) 212 (c) 256
(a) 368 (b) 455 (c) 312
(d) 196 (e) 182
(d) 244 (e) 632
Q243. 49.99% of 1260 + 60.99% of 1100 F : 1 = ? + 39.99% of 450.15 (?)2
(a) 1021 (b) 1221 (c) 1121 (d) Q.257 59.99 × 288.02 ÷ 14.98 ÷ 17.94 = 6
1100 (e) 1141 (a) 51 (b) 38 (c) 41
(13.98) 2−19×3.97 1 (d) 45 (e) 48
Q.244 112.01% of 249.98 + ( ) + 2 −2 =?
23 ×3 2 ×6
(a) 450 (b) 125 (c) 376 Q.258 2 × (3) × (5.989)2 = (11.979)2 × 8.917
2 ?

(d) 294 (e) 196 (a) 8 (b) 2 (c) 16


? 2 (d) 32 (e) 12
Q.245 √120.98–(2.98)2 = (19.99) − (3.99)2 × (5.01)2 + 113.01
Q.259 12.5% of 511.8 + 37.5% of 95.8 + 3.03 × 6.99 = (?)²
(a) 226 (b) 246 (c) 452 (a) 21 (b) 121 (c) 28
(d) 286 (e) 306 (d) 11 (e) 32
Q246. 37.83 × 15.13 + 3√215.87 = ? % 𝑜𝑓 399.9 + (17.88)2 1
(a) 73 (b) 56 (c) 91 Q.260 [(64.01)4 ]12 × 15.99 + 31.95 × 7.98 = ?× 7.96
(d) 63 (e) 85 (a) 25 (b) 56 (c) 40
572.89+? (d) 64 (e) 320
Q.247 15.89 + 37.87% 𝑜𝑓 449.93 − (10.8)2 = (9.9)2 1

(a) 207 (b) 197 (c) 247 Q261 (3374.81)3 × √21.11 + 14.99 × 11.01 = (? )2 + 28.91
(d) 267 (e) 227 (a) 31 (b) 21 (c) 35
(d) 26 (e) 16
Q.248 47.87% of ? + √2303.9 = 17.87 × 14.9 + 24.93% 1
3.992 ×(143.56) 2×(?)2 (2.99) 3 ×7.97
of 359.9 Q.262 =
15.95×7.999 (2.01)2
(a) 650 (b) 450 (c) 600
(a) 5 (b) 4 (c) 8
(d) 615 (e) 575
(d) 7 (e) 6
Q.249 ( 3√6858.9 × 3√4095.98) ÷ √63.98 = (? )2 + √3.8 √48.97×13.05
(a) 4 (b) 11 (c) 15 Q.263 23.99 × 26.003 + = 4.97 ×? ³
90.98
(d) 18 (e) 6 (a) 1 (b) 17 (c) 5
55.88−33.85×
13.87
+2.11 80.11
(d) 12 (e) 8
17.11
Q.250 2.89 = 61
11.84−50.89×
16.79
? Q.264 109.07√?− 21.02 ×? = 47.96√?
(a) 1 (b) 5 (c) 8 (a) 441 (b) 169 (c) 250
(d) 15 (e) 7 (d) 121 (e) 324
Q.251 28.11% of ? = 41.82% of ? – 684.85 + 180.86 Q.265 1332.89 + 171.928 + 17.01 + ?² = 1690.87
(a) 3500 (b) 3700 (c) 4200 (a) 27 (b) 17 (c) 9
(d) 3600 (e) 3900 (d) 13 (e) 19
322.9
Q.252 103 × 721 [34.81% × 80.14% × 40% − Q.266 150.09% of 20 + 17.02 + √? = (8.96)²
34.89% × 19.9% × 30%] =? (a) 984 (b) 1024 (c) 1360
(a) 65611 (b) 65000 (c) 66000 (d) 1225 (e) 674
(d) 65714 (e) 67000 Q.267 56.08% of 149.92 + √28.02 × 6.98 − 11 19 % 𝑜𝑓 998.9 = ?
2.99 3
Q.253 × √511.99 + 123.9% of 650.11 = ? (a) 17 (b) -13 (c) 8
3.99
(a) 850 (b) 792 (c) 812 (d) -16 (e) 22
(d) 841 (e) 750 272.12+?
Q.268 424.97×8.02 – = 124.93% of 2399.57
Q.254 87.08 + 913.99 – 260.13% of 129.88 = 74.98% of ? 7.86
(a) 2928 (b) 2828 (c) 2728
(a) 663 (b) 552 (c) 672
(d) 2628 (e) 3028
(d) 221 (e) 884

For Free Study Material & Quizzes visit : www.mathsbyarunsir.com


For any queries or suggestions email us @ mathsbyarunsir003@gmail.com or whatsapp @ 8881331466
Q.269 578.87 + 330.93 + 29.93% of ? = 40.05% of 2799.87 (d) 280 (e) 221
(a) 775 (b) 500 (c) 600 753+?
Q284. 17.93
+ 20.86% 𝑜𝑓 4199.87 + √840.76 = (30.89)2
(d) 700 (e) 825 (a) 217 (b) 197 (c) 227
(d) 187 (e) 147
727.11+? 3
Q.270 +√1727.83 – 59.89 = √1023.89 Q. 285 (376.89 + 538.89 – 39.83) + ?% of 799.89 = (29.8)²
14.98
(a) 593 (b) 473 (c) 373 (a) 9 (b) 3 (c) 12
(d) 273 (e) 673 (d) 7 (e) 11
Q.271 √? + (16.89)² – 24.93% of 47.89 = 49.97 × 5.93 Q.286 37.12% of 6599.87 + (12.96)² – ? =(49.92)2
(a) 169 (b) 1089 (c) 729 (a) 161 (b) 181 (c) 131
(d) 625 (e) 529 (d) 111 (e) 91
?
Q.272 [ 3√728.89 + √1295.97 + 3√3374.83 ] ÷ ? = √224.97 Q287 18.12 × 24.79 – 14.98 + 14.83% 𝑜𝑓 299.87 = (21.93)2
(a) 4 (b) 10 (c) 14 (a) 105 (b) 125 (c) 145
(d) 18 (e) 8 (d) 185 (e) 165
64.02 1
1 1
Q.273 (1330.57) × (11.79) + 62.5% 𝑜𝑓 399.671 − ? = [(343.11)2 ] × 49.02
3 6 Q.288 43.495 × 31.99 × 28.979 − 2.012 = ?
(a) 48 (b) 57 (c) 69 a) 4 b) 12 c) 6
(d) 39 (e) 18 d) 1 e) 8
Q.274 59.8% of 1539.5 + 37.5% of 95.78 + 0.99 = (?)²
(a) 28 (b) 31 (c) 26 Q289. 66.66 % 249.05 + 3.9 ×6.1 = ? +49.99% of 299.99
(d) 25 (e) 35 (a) 1200 (b) 1600 (c) 6400
Q.275 66.98 (9.084)1/2 + ? = 52.114 + (8.041)² (d) 40 (e) 2500
(a) 41 (b) 26 (c) 46 Q. 290 (33.33×80.989÷99.99)+3.024-?=4.012
(d) 56 (e) 36 a) 20 b) 26 c) 34
75.11 d) 16 e) 40
Q.276 16.004√?+ 68.899√?− 10.001 √? = 33.99 × (? )
(a) 1225 (b) 961 (c) 1024 Q.291 20.021 + 4.969 + 30.499 − 50.022 =?
(d) 729 (e) 1156 a) 5.5 b) 2 c) 8.5
1 d) 12.5 e) 14
Q.277 [(25.35)9 ]6 + 22% 𝑜𝑓 449 = ? + (2.99 × 2.01)2.99 Q.292 995.013-39.976×19.99+5.022=1.988×?
(a) 8 (b) 32 (c) 15 a) 115 b) 85 c) 100
(d) 20 (e) 12 d) 125 e) 75
Q.278 ? % of 224.99 + 19.01 % of 699.98 = Q.293 (10.011) + (23.989)2 = 275.99 + (? )2
2
(15.99)2 – 30.01% 𝑜𝑓 289.99 a) 34 b) 6 c) 28
(a) 18 (b) 16 (c) 12 d) 12 e) 20
(d) 10 (e) 8
Q.294 4.98 3.001 + 3.012 8.99 − 6.89 0.98
1.1 4.03 1 2
2.02
− 2.12 4 = ? − 16.02 9 +
3 ( 2
Q.279 ? + 18.01) = 59.99% of 899.93 1 1.1
10.95 4 − 5.011 1.98
(a) 5 (b) 4 (c) 2 1 2
(d) 6 (e) 3 (a) 9 (b) 9 2 (c) 9 3
87.01 ×7.98 3
Q.280 + 7.99% of 1149.98 = (18.01)2 (d) 9 4 (e) 10
? 1.1
(a) 3 (b) 27 (c) 12 Q.295 √1295.98 + (2744.011)2,98 = ?2 − √960.998
(d) 6 (e) 9 (a) 11 (b) 27 (c) 6
Q.281 (2)? % of 499.99 + √1296.01 = (13.98)2 (d) 81 (e) 9
(a) 4 (b) 2 (c) 5 Q.296 62.987% of 449.89 + 81.0123% of 149.890 = ?% of 675
(d) 6 (e) 3 (a) 80 (b) 45 (c) 50
(d) 60 (e) 70
Q.282 ?2 + 439.08 – 40.011% of 899.99 = 7.99% of 2500.008
(a) 9 (b) 11 (c) 16 Q.297 204.98 × ? × 12.890 = 33625.011 + 25005.0211
(a) 18 (b) 21 (c) 22
(d) 13 (e) 12
3 (d) 24 (e) 27
Q283 14.11 ÷ 98.91 × ( √728.96 × √120.86) + (14.93)2 = ?
(a) 290 (b) 239 (c) 207

For Free Study Material & Quizzes visit : www.mathsbyarunsir.com


For any queries or suggestions email us @ mathsbyarunsir003@gmail.com or whatsapp @ 8881331466
3.0123 4 9.121
Q.298 8.0111
𝑜𝑓 𝑜𝑓
7 11.11
𝑜𝑓 4312.021 = ? × √48.897 Q.312 (11.87)2 + 12.493 × 15.89 − √13224.98 −
(a) 108 (b) 96 (c) 98 (? )2 = (14.96)2
(d) 104 (e) 112 a) 9 b) 2 c) 8
Q.299 119.83% of 1249.93 + ? % of 624.89 = 3374.92 d) 5 e) 10
(a) 300 (b) 350 (c) 400 359.93
Q.313 = (8.89) − 14.5 × 39.89 + (1.95)2 − 34
3
(d) 425 (e) 250 ?
a) 8 b) 2 c) 3
Q.300 20.02 × 32.92 + √63.86 × 9.89 = 20.01 × √? d) 9 e) 5
(a) 1469 (b) 1423 (c) 1253
Q314 44.04 ÷ 3.97 × 7.49 ÷ 2.54 + 3 = √?
(d) 1369 (e) 1163
a) 6 b) 36 c) 24
Q.301 2898.98 ÷ 289.92 + 19.98 + (?)² = 93.98
d) 1296 e) 216
(a) 12 (b) 6 (c) 8
(d) 14 (e) 10 Q.315 √37.54% 𝑜𝑓 400.08 + 75.07% 𝑜𝑓 59.92 = ?
Q.302 29.8% of 701 – 350.01 + 82% of 501 = ?
a) 7 b) 14 c) 49
(a) 230 (b) 330 (c) 370
(d) 270 (e) 210 d) 98 e) 196
2
Q.303 1519.98 + 179.7 × 20.01 + ? = (1.99)⁹ × 9.8 Q.316 (√? ) + (2.01)10 + (4.99)3 = 1156.92
3

(a) 10 (b) 5 (c) 15 a) 256 b) 128 c) 512


(d) 20 (e) 0 d) 1024 e) 1296
4.01 1 1
Q.304 33.989 × 17.02 × 3.99 − 2.012 = ? 1
Q.317 (4913.09)3 + 6.06 × 5.98 − 33 3 % of 9.01 = 24.99%
a) 0 b) 3 c) 5 of 25.01% of ?
d) 4 e) 8 a) 1040 b) 1600 c) 400
1
Q.305 (11.01 + 12.97) × + 5.956 = 18 − ? d) 800 e) 1200
7.99
a) 15 b) 13 c) 5 Q.318 (9.01) + (7.94)3 + 5.12 × 17.95 = 𝑥 2
3

d) 9 e) 18 a) 11 b) 15 c) 13
Q.306 119.022 + 40.99 + 9.03 = ?2 d) 10 e) 14
a) 13 b) 17 c) 11 Q.319 ? % of 299.99 + 54.95% of 239.94 = 18.99% of 899.99
d) 15 e) 19 (a) 17 (b) 19 (c) 23
(d) 29 (e) 13
Q.307 58.99 + 52.11 − 47.94 + ? = 85.96
Q.320 818.88 ÷ 62.93 + 18.9 × 21.9 = 34.95 × 19.92 - ?
a) 27 b) 19 c) 23
(a) 329 (b) 269 (c) 215
d) 15 e) 30
(d) 278 (e) 302
Q.308 (14.96) + (5.011)3 + 50.02 = ?2
2
Q.321 2.99 × (9.99 + 4.95) + (48.04/5.99)/3.94 – 6.97 = ?
a) 28 b) 16 c) 25 (a) 30 (b) 25 (c) 40
d) 12 e) 20 (d) 22 (e) 16
Q.309 624.89 + (13.89)2 − 49.01 = (? )2 Q.322 5.94 + 1/12.93 of {2599.96 ÷ (117.98 – 107.98)} = ?
a) 35 b) 40 c) 36 (a) 32 (b) 26 (c) 36
d) 44 e) 46 (d) 42 (e) 46
163.98+?
Q.310 + 389.97 + 19.98% of 724.89 = 24.98% of Q. 323 [(22.9 × 1.922 × 23.92 )] ÷ (2.02 × √1295.96) =
24.98
2203.89 (2.93)? + 6.97
a) 206 b) 216 c) 226 (a) 8 (b) 12 (c) 4
d) 236 e) 246 (d) 6 (e) 14
3
Q.311 ? % of 749.89 + √728.89 = 26.89% of 499.87 + 29.89% Q.324 52.034 + 617.121 + 32.004 = ? + 576.71 + 68.284
(a) 56 (b) 51 (c) 48
of 349.89 + √80.87 (d) 66 (e) 62
a) 38 b) 32 c) 40 2
d) 42 e) 28 Q. 325 (50.001% of 51800.10) ÷ ? = (160% 𝑜𝑓 10.01) × 2.01
(a) 40 (b) 50 (c) 75
(d) 80 (e) 65

For Free Study Material & Quizzes visit : www.mathsbyarunsir.com


For any queries or suggestions email us @ mathsbyarunsir003@gmail.com or whatsapp @ 8881331466
Q.326 23.87 + 24.29 – 12.038 = 34.31 × 4.18 - ?2 Q.341 19.87% of 5000.11 – 24.98% of 3000.11 = ? × (4.9)2
(a) 100 (b) 5 (c) 10 A) 10 B) 15 C) 8
(d) 20 (e) 15 D) 16 E) None of these
Q. 327 (63.95 + ?) ÷ 25.02 + 390.09 + 19.97% of 724.87 = 540.07 Q.342 16.11 × 14.78 – 240.21 – 32.02 × 9.99 + 419.99 = ?2
(a) 50 (b) 25 (c) 61 A) 10 B) 15 C) 5
(d) 72 (e) 47 D) 12 E) None of these
Q.328 14.99% of 115.95 × 18.892% of 179.77 = (?) – 372 Q.343 64.8% of 199.99 – 25.88% of 499.99 = ?
(a) 904 (b) 1964 (c) 4064 A) 5 B) 8 C) 4
(d) 2984 (e) 1864 D) 10 E) None of these
22
Q.329 41.979 × 7 + 19.989% of 530.014-26.021= ? Q.344 45.012% of 1499.98 – 2.11 × 52 = ?2
a) 244 b) 198 c) 236 A) 25 B) 35 C) 15
d) 212 e) 252 D) 20 E) None of these
Q.330 (23.012 × 22.989) + 20.985 × 7.014 = ?2 Q. 345 (32.12 × 24.98) – (19.876)2= 20.11% of ?
a) 8 b) 38 c) 26 A) 1000 B) 3200 C) 2000
d) 12 e) 44 D) 2500 E) None of these
Q.346 21.88% of 5000.001 + 14.21% of 399.78 = ?2
Q.331 √1443.979 ÷ 18.981 + 3.2 × √16.017 = (? ) A) 26 B) 14 C) 24
a) 16 b) 30 c) 8 D) 36 E) None of these
d) 26 e) 10 3 2
Q.347 7 –(13.11) – ? = 25.11% of 120.021
Q.332 779.98÷ 48.014 × 15.989 = ? A) 216 B) 144 C) 156
a) 280 b) 248 c) 275 D) 246 E) None of these
d) 242 e) 260 Q.348 79.86% of 2500.11 - ?% of 2499 = (9.8)3
Q.333 1485.988 + 212.04 − 1703.99 = ? −(11.02)2 A) 20 B) 60 C) 30
a) 95 b) 115 c) 130 D) 40 E) None of these
d) 102 e) 135 Q.349 41.97 × 5.12 ÷ 2.99 + 49.89 = ?× 1.99
Q.334 737.11 × 2.9 + 362.90 × 2.01 + 10.98 × 32.9 = ? × 11.11 A) 120 B) 80 C) 15
A) 120 B) 300 C) 240 D) 40 E) 60
D) 280 E) None of these Q.350 18.003 × 3.99 × (135.003 ÷ 26.97) – (15.001 × 3.94) = ?
Q.335 ? × 31.98 – 25.11% of (480.01 × 127.89) = 0 A) 300 B) 360 C) 420
A) 80 B) 240 C) 64 D) 250 E) 460
D) 480 E) None of these Q.351 30.01 × √1023 + 196 = ?2 – 287
Q.336 (8.91)3 × (8910.11 ÷ 109.89) = (14580.11 ÷ 19.89) × ? A) 15 B) 38 C) 24
A) 27 B) 42 C) 81 D) 48 E) 30
D) 729 E) None of these Q.352 35.1% of 1599 = ?–(449.96 ÷ 6.12)
Q.337 (25.98)2 + 10.883 - √48 = 2.1 × ?3 A) 960 B) 840 C) 500
A) 8 B) 10 C) 12 D) 700 E) 610
D) 5 E) None of these Q.353 16.12 × 15.94 + 654.92 – 344.83 = ?× 5.95
A) 12 B) 60 C) 90
1 1 800
Q338. 24.99% of 319.99  33 % of 167.99  37 % of D) 145 E) 200
3 2 3 Q354. 21 + 4.9 × 7.9 + 9.88 = ?
(a) 8 (b) 5 (c) 4 (a) 65 (b) 71 (c) 66
(d) 6 (e) 36 (d) 75 (e) 81
Q.339 64.9% of 1200.11 – 15.11% of 400.09 = ?3 × 44.9% of 199.9 Q355. 138% of 3781 + 38.74% of 141 = ?
A) 4 B) 1 C) 3 (a) 5288 (b) 5248 (c) 5271
D) 2 E) None of these (d) 5444 (e) 6444
Q.340 35.21% of 5000.11 – (31.98)2 + 3.12 = ? Q356. (340 × 9.98) ÷ 6.4001 + 1245.15 = ?
A) 825 B) 569 C) 729 (a) 1766 (b) 1776 (c) 1676
D) 689 E) None of these (d) 1876 (e) 1806

For Free Study Material & Quizzes visit : www.mathsbyarunsir.com


For any queries or suggestions email us @ mathsbyarunsir003@gmail.com or whatsapp @ 8881331466
Q357. 31.952 – 12.052 + (1987.25 + 21.85) ÷ ? = 900 (a) 1050 (b) 975 (c) 950
(a) 115 (b) 120 (c) 90 (d) 1000 (e) 1100
(d) 85 (e) 100 Q373. (27.97) ² – (21.92) ² + (2345.88 + 154.14) ÷ ? = 350
Q358. 1576 ÷ 45.02 + 23.99 × √255 = ? (a) 36 (b) 45 (c) 50
(a) 340 (b) 420 (c) 380 (d) 65 (e) 55
1
(d) 460 (e) 360 Q374. (36.01)3 × (4096)2 × (37.99)2 ÷ (93 × 75.982 ) = 4?
24.17 2 1 399.86 40.13 (a) 7 (b) 3 (c) 5
Q359. ( 4.07 ) × 4.95 × 39.94 ÷ 899.79 = ?
(d) 8 (e) 7
(a) 1620 (b) 1680 (c) 1700 Q375. 18% of 256 + 35% of 290 – 15% of 385 =?
(d) 1550 (e) 1750 (a) 83 (b) 80 (c) 90
Q360. 67.99% of 1401 − 13.99% of 1299 = ? (d) 70 (e) 85
(a) 700 (b) 720 (c) 770
Q376. 69.008% 𝑜𝑓 699.998 + 32.99% 𝑜𝑓 399.999 = ?
(d) 800 (e) 740
(a) 615 (b) 645 (c) 675
Q361. 5466.97 − 3245.01 + 1122.99 = ? +2309.99 (d) 715 (e) 725
(a) 1130 (b) 1000 (c) 1100 Q377. 29.97% of 781 + 43% of 801 = 528 + ?
(d) 1035 (e) 1060 (a) 100 (b) 30 (c) 70
Q362. 40.1% of 601 − 250.17 = ? −35.15% of 2000.09 (d) 80 (e) 50
(a) 800 (b) 500 (c) 690 Q378. (77.987% of 358) + (68.55% of 729) = ?
(d) 760 (e) 550
(a) 780 (b) 705 (c) 840
Q363. 54001 ÷ 60.14 × 29.96 = ? × 40.07 (d) 825 (e) 695
(a) 700 (b) 675 (c) 500 1 1 1
(d) 550 (e) 680 Q379. (960.89)2 × (2743.8)3 + 12.991 × (35.81)2 =
(?)3
Q364. 449.98 ÷ 5.99 + (2.99)4 − 74.97 =? + (15.99)2
2
(a) 71 (b) 81 (c) 86 (a) 5 (b) 11 (c) 8
(d) 75 (e) 91 (d) 3 (e) 4
3
Q365. √1728 × 861.08 ÷ 83.98 − 57.98 =? Q380. 712.815 + 87.11 + 25.921 × 39.89 = 23.11 ×?
(a) 55 (b) 52 (c) 65 (a) 75 (b) 66 (c) 45
(d) 61 (e) 59 (d) 50 (e) 80
Q366. 69.98% of 259.98 – 29.98% of 529.98 =? 12.141×11.11 ?2 −(3.11)2 ×52.85
Q381. =
(a) 19 (b) 23 (c) 20 3.21 10.79
(d) 27 (e) 18 (a) 31 (b) 25 (c) 18
4 4 (d) 35 (e) 21
Q367. 11 𝑜𝑓 1330.98 ÷ √483.97 + √1296.01 =? 1

(a) 23 (b) 19 (c) 28 Q382. (12.95+ ? )(? −13.21) = 15.31 × (511.91)3


(d) 24 (e) 35 (a) 23 (b) 21 (c) 17
23 4 19 (d) 13 (e) 25
Q368. 349 ÷ 5 ÷ 35 + 499 =?
511.91×255.89 2
(a) 489 (b) 476 (c) 550 Q383. ( 7.99×32.11 ) = (63.99)?
(d) 525 (e) 500 (a) 3 (b) 6 (c) 7
Q369. 12.994 × 22.009 × 17.982 =? (d) 9 (e) 2
(a) 5,100 (b) 5,200 (c) 5,148 Q384. 25.05% of 2844.01 + 14.95 × 2400 =?
(d) 5,199 (e) 5,348 (a) 36,710 (b) 36,510 (c) 35,810
Q370. 18% of 602 + 27.8% of 450 =? (d) 35,610 (e) 36,210
(a) 234 (b) 260 (c) 225 Q385. 1080.18 ÷ 36 + 187 × 20.05 =?
(d) 220 (e) 250 (a) 3650 (b) 3770 (c) 3825
Q371. 479.7 × 26.12 ÷ 38.99 + ? = 25 + 54 (d) 3800 (e) 3700
(a) 373 (b) 337 (c) 411 Q386. 125% of 4875 + 88.005 × 14.995 =?
(d) 437 (e) 397 (a) 7395 (b) 7490 (c) 7510
Q372. 3194 ÷ 7.85 + 74.85% of 798 =? (d) 7375 (e) 7415

For Free Study Material & Quizzes visit : www.mathsbyarunsir.com


For any queries or suggestions email us @ mathsbyarunsir003@gmail.com or whatsapp @ 8881331466
Q387. 39.05 × 14.95 – 27.99 × 10.12 = (36 +?) × 5 (a) 6 (b) 4 (c) 2
(a) 22 (b) 29 (c) 34 (d) 8 (e) 10
(d) 32 (e) 25 ?
Q403. 3 of 299.98 – 489.89 = 509.94
Q388. 158.004 × 4.06 + 20.12% of 849.86 + ? = 950.93
(a) 25 (b) 20 (c) 15
(a) 350 (b) 480 (c) 280
(d) 10 (e) 18
(d) 250 (e) 150
Q389. 1884.88 ÷ 144.89 + 6.99 + (? )2 = 69.09 Q404. √? = (1248.28 + 51.7) ÷ 99.9 − 7.98
(a) 4 (b) 9 (c) 6 (a) 49 (b) 81 (c) 64
(d) 7 (e) 8 (d) 16 (e) 25
1
Q390. √12000 × 34.98 + 150.04 =? Q405.(15.95)4 + (3.01)3 − 111.99 × 2.02 + (9.98)2 =?
(a) 3000 (b) 4700 (c) 4000 (a) 95 (b) –95 (c) 105
(d) 3500 (e) 5600 (d) –105 (e) –115
Q391. 0.2% of 356 × 0.8% of 779 =? Q406. 69.008% of 699.998 + 32.99% of 399.999 =?
(a) 4 (b) 1 (c) 9 (a) 615 (b) 645 (c) 675
(d) 8 (e) 12 (d) 715 (e) 725
Q392. 63.9% of 8920.2 +? % of 5320.3 = 6830.162 Q407. 180% of 25501 + 50% of 28999 =?
(a) 36 (b) 21 (c) 17 (a) 62400 (b) 64000 (c) 60400
(d) 31 (e) 9 (d) 64200 (e) 61600
340 29.997 179.909
5 7 Q408. 20.002 ÷ 510 × 59.919 =?
Q393. 8 of 4011.83 + 10 of 3410.12 =?
(a) 4810 (b) 4980 (c) 4890 (a) 760 (b) 800 (c) 690
(d) 4930 (e) 4850 (d) 870 (e) 780
Q394. 5144.98 + 3449.95 – 4559.98 =? Q409. (9321+5406+1001) ÷ (498+929+660) =?
(a) 4035 (b) 4050 (c) 4135 (a) 13.5 (b) 4.5 (c) 16.5
(d) 5200 (e) 4345 (d) 7.5 (e)18.5
2 Q410. 182.225×21.652×33.684 =?
Q395. 49.98% of ? – 3 of 243.02 = 49.94 (a) 136136 (b) 49428 (c) 120426
(a) 436 (b) 424 (c) 450 (d) 106438 (e) 84456
(d) 474 (e) 502 Q411. 63.9% of 8924.95 +? % of 5324.96 = 6827.9862
Q396. 1.007 + 2.0008 + 30.09 ÷ 10 – 5.99 = ? (a) 36 (b) 21 (c) 17
(a) 1 (b) 12 (c) 3 (d) 31 (e) 41
(d) 2 (e) 0 Q412. 40.02% of 601+ 250.17 =? – 75.01% of 911.98
Q397. 44.95% of 599.98 + 34.98% of 299.98 = ? (a) 1274 (b) 1147 (c) 1174
(a) 295 (b) 425 (c) 395 (d) 1074 (e) 1474
(d) 375 (e) 430 Q413. 20.06% of 599 + 10.01% of 901 =?
Q398. √441.13 ÷ 2.96 + √? = 46.98 (a) 150 (b) 210 (c) 250
(a) 1900 (b) 1760 (c) 1650 (d) 280 (e) 380
(d) 1700 (e) 1600 Q414. √289.19 × (2.08) ÷ 10.97 =?
Q399. 133.99 + 1333.98 + 13.98 = ? (a) 2 (b) 3 (c) 5
(a) 1452 (b) 1482 (c) 1575 (d) 6 (e) 4
(d) 1650 (e) 1730 Q415. 1884 ÷ 144.89 + 6.99 + (? )2 = 69.09
2 3
Q400. of 34.98 + of 489.98 =? (a) 4 (b) 9 (c) 6
5 7
(a) 224 (b) 254 (c) 242 (d) 7 (e) 8
(d) 340 (e) 324 Q416. √12102 × 34.98 + 150.04 =?
Q401. √121.01 × 99.98 × 324.001 =? (a) 3000 (b) 4700 (c) 4000
(a) 1900 (b) 1980 (c) 1890 (d) 3500 (e) 5600
(d) 2010 (e) 2100 Q417. 1010 ÷ 36 + 187 × 20.05 =?
0.63 −0.43 (a) 3650 (b) 3770 (c) 3825
Q402. 9.98 × (0.62 +0.6×0.4+0.42 ) =? (d) 3800 (e) 3700

For Free Study Material & Quizzes visit : www.mathsbyarunsir.com


For any queries or suggestions email us @ mathsbyarunsir003@gmail.com or whatsapp @ 8881331466
Q418. (3.2)2 + (9.8)2 + (8.13)2 + (4.24)2 =? Q433. 25.896 × 5249.80 ÷ 349.78 + √575.94 = ?
(a) 180 (b) 198 (c) 190 (a) 526 (b) 457 (c) 482
(d) 206 (e) 184 (d) 392 (e) 414
2
3 14
Q419. [(8 × 2 ) ÷ (2.5 − 0.8)] =? Q434. (11.02)2 + (15.89)2 – (14.01)2 =?
(a) 1.6 (b) 2.4 (c) 4.1 (a) 281 (b) 181 (c) 141
(d) 5.2 (e) 6.8 (d) 161 (e) 81
Q420. 2418.065 + 88 ÷ 14.2 × 6 =? Q435. 36.02% of 225 + 42.02% of 400 =? % of 199.98
(a) 136.5 (b) 12.45 (c) 124.5
(a) 2470 (b) 2465 (c) 2460
(d) 144.5 (e) 184.5
(d) 2450 (e) 2455
Q436. 192.89 + 299.98 + 3999.98 ÷ 7.99 = ? + 499.98
Q421. 564.866 + 82.1091 × 44.881 − 34.111 =?
(a) 493 (b) 349 (c) 394
(a) 4650 (b) 4350 (c) 4250
(d) 293 (e) 693
(d) 4220 (e) 4400
3 2
Q437. ? % of 9.89 = 23.98% of 49.98 + 43.9% of 149.98
Q422. √64002 + (46.85) =? (a) 570 (b) 900 (c) 860
(a) 2090 (b) 2100 (c) 2250 (d) 625 (e) 780
(d) 2175 (e) 2325 3
Q423. 125% of 4875 + 88.005 × 14.995 =? Q438. √1330 + √1763 + √1023 =? +√785
(a) 7395 (b) 7490 (c) 7510 (a) 37 (b) 43 (c) 75
(d) 7375 (e) 7415 (d) 57 (e) 67
Q424. 63.5% of 8924.2 +? of 5324.4 = 6827.5862 Q439. √3248 × √1297 ÷ √360 =?
(a) 36 (b) 22 (c) 17 (a) 130 (b) 108 (c) 140
(d) 31 (e) 9 (d) 160 (e) 90
5 7 Q440. 20.06% of 599 + 10.01% of 901 =?
Q425. 8 of 4011.33 + 10 of 3411.22 =?
(a) 450 (b) 300 (c) 210
(a) 4810 (b) 4980 (c) 4890
(d) 280 (e) 350
(d) 4930 (e) 4850 249.89 299.9 15
Q441. 15 × 19.99 ÷ 99.88 = ?
Q426. 8787 ÷ 343 × √50 =?
(a) 180 (b) 185 (c) 195 (a) 1850 (b) 1667 (c) 1750
(d) 190 (e) 175 (d) 1900 (e) 2000
Q442. 1201 ÷ 14.99 × 19.91 + 400.01 =?
Q427. √1296.12 × (303 ÷ 8) = (? )2 (a) 1700 (b) 1850 (c) 1800
(a) 18 (b) 28 (c) 48 (d) 1920 (e) 2000
(d) 37 (e) 58 Q443. 40.005% of 439.998 + ?% of 655.011 = 228.5
Q428. 5940 ÷ 28 ÷ 6 =? (a) 8 (b) 17 (c) 12
(a) 40 (b) 35 (c) 46 (d) 20 (e) 5
(d) 52 (e) 27
1920.79
Q444. √? = (1346.92 + 53.11) ÷ 99.9 − 6.98
Q429. (16.97) ² + 11.09 × 27.97 - 288.91 =? (a) 121 (b) 441 (c) 1024

(a) 525 (b) 386 (c) 484 (d) 49 (e) 196
(d) 505 (e) 462 1
Q445. 32.01² × 5123 × 33.992 ÷ (29 × 16.972 ) = 2?
?–√120.86 18.96×21.13 32
Q430. – = 2–2 (a) 3 (b) 4 (c) 9
32 √48.91
(a) 848 (b) 724 (c) 775 (d) 10 (e) 6
(d) 874 (e) 905 Q446. (14.99% of 4799.995) ÷ ? = (60.11% of 4.111) ²
Q431. 44.11% of 224.96 + 807.9 × 2–3 – (9.97)2 = ?2 (a) 150 (b) 25 (c) 100
(a) 18 (b) 10 (c) 13 (d) 50 (e) 125
3
(d) 26 (e) 7 Q447. 20 𝑜𝑓 239.98 = ? ÷ (1.99 × 0.99)
3 (a) 72 (b) 33 (c) 45
Q432. 3 5 of 259.99 =? % of 1247.9
(a) 55 (b) 45 (c) 75 (d) 37 (e) 80
(d) 65 (e) 50

For Free Study Material & Quizzes visit : www.mathsbyarunsir.com


For any queries or suggestions email us @ mathsbyarunsir003@gmail.com or whatsapp @ 8881331466
Q448. √1296.002 ÷ 8.996 × 9.98 + 39.99 = ? (d) 51 (e) 65
(a) 80 (b) 8 (c) 4 Q464. 39% 𝑜𝑓 405 + 61% 𝑜𝑓 610 − 183.67 = ?
(d) 120 (e) 40 (a) 450 (b) 300 (c) 230
Q449. {(4444 + 333 + 22 + 1) − (2 × 3 × 4 × 5)} × 2.532 =? (d) 280 (e) 344
(a) 11700 (b) 12250 (c) 10800 Q465. 5682 ÷ 63 × 36 = ? × 17.998
(d) 12100 (e) 10500 (a) 180 (b) 190 (c) 210
Q450. 40.05% 𝑜𝑓 349.9 + 59.89% 𝑜𝑓 249.98 =? (d) 240 (e) 140
(a) 280 (b) 290 (c) 270 Q466. 4.99 × 12.865 + 599 = ?
(d) 275 (e) 298 (a) 650 (b) 655 (c) 665
Q451. 17% 𝑜𝑓 760 + 57% 𝑜𝑓 78.99 + 77.77 =? (d) 670 (e) 675
(a) 238 (b) 242 (c) 245
Q467. 250.01 − 429.99 + 549.99 = ?
(d) 251 (e) 256
68 (a) 370 (b) 420 (c) 340
Q452. 35.99√?+ 32.0032√? = 10.998 × (? ) (d) 410 (e) 320
(a) 81 (b) 72 (c) 169 340 46.98 119
Q468. 34.023 × 510 × 94.01 = ?
(d) 121 (e) 144
(a) 1.5 (b) 1.2 (c) 2.1
Q453. (3.02) + (9.98)2 + (8.13)2 + (4.04)2 =?
2
(d) 2.4 (e) 1.7
(a) 190 (b) 230 (c) 150
(d) 210 (e) 160
Q469. 399.89 ÷ 7.88 × 11.88 + 249.87 – 189.88 =?
Q454. 8599.999 ÷ 420.002 × 14.996 = ? (a) 755 (b) 655 (c) 680
(a) 250 (b) 325 (c) 275 (d) 555 (e) 695
(d) 307 (e) 315 4
Q470. 47.89% of 449.9 8+ 52.01% of 439.89 = 5 of ?
Q455. √2703 − √1156 + √483 = ?
(a) 456 (b) 655 (c) 556
(a) 50 (b) 90 (c) 40
(d) 756 (e) 856
(d) 20 (e) 30 3 4 3
Q456. 3001 × 749 ÷ 1001– 1399 = ? Q471. √1727 + √675 + √1295– √1332 =?
(a) 650 (b) 700 (c) 950 (a) 43 (b) 24 (c) 45
(d) 850 (e) 1000 (d) 33 (e) 21
Q472. 43.05% of 799.89 + 56.89% of 899.89 =?
Q457. 149.9% 𝑜𝑓 149.9 + 149.9 = ? (a) 857 (b) 785 (c) 587
(a) 375 (b) 400 (c) 350 (d) 875 (e) 890
(d) 425 (e) 450
989 65 515
Q473. 114.88% of 559.88 + 83.98% of 419.88 =?
Q458. 34 ÷ 869 × 207 = ? (a) 721 (b) 799 (c) 697
(a) 840 (b) 920 (c) 970 (d) 997 (e) 897
(d) 780 (e) 1000 Q474. 8348.98 + 6539.98 ÷ 9.9 + 3544.98 ÷ 99.98 – 6449.9 =?
Q459. (32.13)2 + (23.96)2 − (17.11)2 = ? (a) 2288 (b) 2588 (c) 2488
(a) 1270 (b) 1421 (c) 1451 (d) 2858 (e) 2128
3 4 4 3
(d) 1361 (e) 1311 Q475. 5 of 7 of 699 – 9 of 4 of 198 = ?
Q460. 6894.986 + 5025.005 + 600.020 = ? (a) 274 (b) 471 (c) 174
(a) 12170 (b) 13540 (c) 12950 (d) 74 (e) 1074
(d) 11560 (e) 12520 Q476. 63.98% of 749.98 + 49.01% of 399.98 =10% of ?
Q461. 31.999 × 12.001 × 17.5001 = ? (a) 2640 (b) 2880 (c) 2480
(a) 6600 (b) 6720 (c) 6480 (d) 2840 (e) 2740
(d) 6070 (e) 6270 Q477. 23.98% of 259.98 + 29.98% of 709.98 – 279.89 =?
Q462. 12.164 × 22.009 × 17.932 = ? (a) –5 (b) –10 (c) –15
(a) 4901 (b) 4895 (c) 4752 (d) 9 (e) 5
(d) 4959 (e) 4350 Q478. 4649.98 ÷ 99.9 + 3749.9 ÷ 9.98 – 3659.98 ÷ 39.98 =?
(a) 430 (b) 303 (c) 330
Q463. 16.978 + 27.007 + 36.984 − 12.969 − 9.003 = ?
(d) 230 (e) 130
(a) 72 (b) 42 (c) 60

For Free Study Material & Quizzes visit : www.mathsbyarunsir.com


For any queries or suggestions email us @ mathsbyarunsir003@gmail.com or whatsapp @ 8881331466
Q479. 7429.98 ÷ 300.02 × 10.004 =? (a) 6400 (b) 5625 (c) 900
(a) 256 (b) 251 (c) 258 (d) 1600 (e) 2025
(d) 248 (e) 266 16.022
Q495. 24.982 × (7.98×15.04) × 38.93 = 130 ×?2
Q480. 15.2% of 725 + 12.8% of 643 =?
(a) 110 (b) 192 (c) 172 (a) 25 (b) 45 (c) 40
(d) 196 (e) 207 (d) 30 (e) 20
Q481. (36.04)2 – (21.08)2 =? Q496. 71.98% of 1200 + 35.06% of 270 =? % of 600
(a) 888 (b) 855 (c) 846 (a) 140 (b) 125 (c) 120
(d) 871 (e) 865 (d) 135 (e) 160
3 Q497. 88.05% of 450 =? % of 530
Q482. √728 × √485 =? (a) 70 (b) 68 (c) 75
(a) 150 (b) 180 (c) 190 (d) 80 (e) 65
(d) 210 (e) 198
Q483. 496 ÷ 0.6 × 0.5 =? Q498. √899 × (12.005)2 +? = 5000
(a) 413 (b) 595 (c) 148 (a) 680 (b) 720 (c) 750
(d) 653 (e) 498 (d) 620 (e) 630
150 199 16 Q499. 8537.986 – 2416.005 – 221.998 =?
Q484. × ÷ =? (a) 6500 (b) 5900 (c) 4300
17 13 91
(a) 650 (b) 700 (c) 770 (d) 3900 (e) 4500
(d) 820 (e) 850
Q485.151.011 – 419.999 + 649.991 =? Q500. 1019.999 ÷ 60.007 =?
(a) 380 (b) 420 (c) 350 (a) 11 (b) 23 (c) 17
(d) 410 (e) 360 (d) 27 (e) 21
Q486. 1299 ÷ 19.99 × 25.01 + 400.01 =? Q501. 111111 ÷ 1111 ÷ 11 =?
(a) 2025 (b) 2300 (c) 1925 (a) 1180 (b) 15 (c) 1100
(d) 2200 (e) 1700 (d) 9 (e) 11
Q487. √5183.98 ÷ √36 = ? Q502. 16.001 × 30.999 × 8.999 =?
(a) 21 (b) 6 (c) 12 (a) 4464 (b) 4864 (c) 4160
(d) 18 (e) 26 (d) 5900 (e) 4644
Q488. 48.01 × 34.9 ÷ 2.95 = ? Q503. 88.25% of 450 =? % of 530.
(a) 576 (b) 591 (c) 564 (a) 70 (b) 68 (c) 75
(d) 553 (e) 560 (d) 80 (e) 90
Q489. √9026 × (5.96)2 =? Q504. 21 + 4.9 × 7.9 + 9.88 =?
(a) 3050 (b) 3780 (c) 2340 (a) 65 (b) 71 (c) 66
(d) 3420 (e) 3950 (d) 75 (e) 81
39.94
Q490. 4734.96 − 3454.03 − 1612.86 = ? −1611.43 Q505. 74.99% of 255.89 + 𝑥 = 47.99% of 649.81

(a) 1280 (b) 2290 (c) 1020 1 1 1
(a) 16 (b) 9 (c) 25
(d) 18150 (e) 1040 1 1
323 971 56 (d) 4 (e) 36
Q491. × 251 × 61 = ? 𝑥
55
(a) 27 (b) 9 (c) 4 Q506. 294.01 × 8.01 − 19.99% 𝑜𝑓 119.99𝑥 = 254.9 ÷
(d) 16 (e) 21 1.9
Q492. 133.008 × 2.97 − 111.87 + 74.13 = ? (a) 10 (b)16 (c) 8
(a) 311 (b) 234 (c) 360 (d) 5 (e) 12
(d) 290 (e) 399 Q507. 31.952 – 12.052 + (1987.25 + 21.85) ÷ ? = 900
(a) 115 (b) 120 (c) 90
Q493. 32.1 × 2799 ÷ 549 + 120 = ? (d) 85 (e) 100
(a) 220 (b) 280 (c) 375
(d) 505 (e) 190 Q508. 1576 ÷ 45.02 + 23.99 × √255 =?
160 (a) 340 (b) 420 (c) 380
Q494. 2775 × = 5550
√(?) (d) 460 (e) 360

For Free Study Material & Quizzes visit : www.mathsbyarunsir.com


For any queries or suggestions email us @ mathsbyarunsir003@gmail.com or whatsapp @ 8881331466
Q509. 1884 ÷ 144.89 + 6.99 + (? )2 = 69.09 (d) 80566 (e) 76566
(a) 4 (b) 9 (c) 6 Q525. 432.62 − 269.21 ÷ (11.9% 𝑜𝑓 74.95) = ?
(d) 7 (e) 8 (a) 370 (b) 380 (c) 403
Q510. (3.2) + (9.8) + (8.13)2 + (4.24)2 =?
2 2 (d) 410 (e) 420
(a) 180 (b) 190 (c) 198 Q526. 899.99 ÷ 45.072 = ? −224.488
(d) 206 (e) 184 (a) 224 (b) 230 (c) 250
(d) 244 (e) 260
Q511. 2418.065 + 88 ÷ 14.2 × 6 =?
Q527. (17.95)2 − (14.05)2 + (2343.75 + 81.55) ÷ ? = 229
(a) 2470 (b) 2465 (c) 2460
(d) 2450 (e) 2455 (a) 24 (b) 28 (c) 30
3
(d) 20 (e) 34
Q512. √13824.3 + (46.85)2 =? Q528. 39.97% 𝑜𝑓 649.8 ÷ 13.05 = 45.12− ?
(a) 2090 (b) 2100 (c) 2230 (a) 40 (b) 15 (c) 25
(d) 2175 (e) 2325 (d) 10 (e) 30
Q513. 64.9% of 8920.2 + ? %of 5320.4 = 6830.1862
Q529. √624.98 + √729.25 = ?
(a) 36 (b) 20 (c) 17
(a) 58 (b) 56 (c) 52
(d) 31 (e) 9
(d) 61 (e) 62
Q514. 32.99% of 549.98 + 61.98% of 840.01 = 3.99% of ?
Q530. 69.008% of 699.98 + 32.99% of 399.999 = ?
(a) 175575.5 (b) 175.575 (c) 17557.5
(a) 615 (b) 645 (c) 675
(d) 15557.5 (e) 19557.5
(d) 715 (e) 815
5 3 4
Q515. √5 𝑜𝑓 254.89 + 29.9% 𝑜𝑓 299.9 − 5 𝑜𝑓 ? = −366.9 Q531. (9321 + 5406 + 1001) ÷ (498 + 929 + 660) = ?
(a) 462.5 (b) 46.25 (c) 426.5 (a) 13.5 (b) 4.5 (c) 2.5
(d) 450.5 (e) 442.5 (d) 7.5 (e) 21.5
Q516. 13534.98 + 16753.76 + 18934.34 – 34353.84 =? Q532 63.5% of 8924.2 + ?% of 5324.4 = 6827.5862
(a) 12869 (b) 14968 (c) 14689 (a) 36 (b) 52 (c) 13
(d) 14869 (e) 13869 (d) 21 (e) 41
2.99 291.01 Q533. 67% of 801 – 231.17 = ? – 23% of 789
Q517. 6.98 ÷ 238.02 × 387.98 + 46.02% 444.98 = ? (a) 490 (b) 440 (c) 540
(a) 320 (b) 361 (c) 381 (d) 520 (e) 590
(d) 241 (e) 341 Q.534 45.96 × 29.98 + 167.98 × 1/3.98 × 1/1.92 – 120.99 = ?
Q518. 87.66% of 550.12 + 13.76% of 850.14 =? (a). 1240 (b). 1280 (c). 1220
(a) 650 (b) 600 (c) 635 (d). 1260 (e). 1250
(d) 563 (e) 453 Q. 535 [(6.973 × 7.073) / {7.172 × 6.94}3] × 6.978 = 7.13?
Q519. 447.75 ÷ 28 × 4.99 = ? (a). 4 (b). 3 (c). 1
(a) 60 (b) 70 (c) 72 (d). 5 (e). 6
(d) 80 (e) 75 Q.536 91.99 – [70.97 + {4.04 – (4.95 – 1.99))}] = ?
Q520. 1679 ÷ 14.95 × 5.02 = ? (a). 10 (b). 15 (c). 25
(a) 540 (b) 525 (c) 545 (d). 20 (e). 30
(d) 560 (e) 520 Q. 537 (19.99)2 × [35.98 × 24.04] ÷ (11.98 × 1.982) = ?
Q521. 5237.897 – 6629.010 + 7153.999 – 2205.102 = ? (a). 5600 (b). 6400 (c). 9600
(a) 6340 (b) 4688 (c) 5240 (d). 7200 (e). 8400
(d) 3558 (e) 6290 Q.538 51.98 + 105.98 – ? × 2.93 = 130.91
Q522. 459% of 849.947 + 266.5% of 6284.012 – 1486.002 = ? (a). 6 (b). 18 (c). 8
(a) 20330 (b) 12640 (c) 15000 (d). 12 (e). 9
(d) 22160 (e) 19140 Q. 539 (7.072 - 30.98) × 2.98 ÷ (7.982 - 45.96) = ?
Q523. (9321 + 5406 + 1001) ÷ (498 + 929 + 660) = ? (a). 5 (b). 8 (c). 11
(a) 13.5 (b) 4.5 (c) 16.5 (d). 15 (e). 3
(d) 7.5 (e) 10.5 Q.540 849.99 ÷ 33.94 + 4.953 = 5.96 × 4.85 × ?
Q524. 12.95 × 7.05 + (85.01)2 × 10.99 = ? (a). 5 (b). 2 (c). 10
(a) 69566 (b) 79566 (c) 81000 (d). 8 (e). 14

For Free Study Material & Quizzes visit : www.mathsbyarunsir.com


For any queries or suggestions email us @ mathsbyarunsir003@gmail.com or whatsapp @ 8881331466
Q.541 {74.95 - (118.99 - 128.08 ÷ 8.08 × 6.98)} = ? Q557. (14.99% of 4799.995) ÷ ? = (170% of 7.111)²
(a). 56 (b). 74 (c). 68 (a) 150 (b) 25 (c) 100
(d). 92 (e). 86 (d) 50 (e) 5
Q.542 11.98 × 84.98 ÷ 4.95 + 39.99% of 349.99 = ? 3
Q558. 20 𝑜𝑓 239.98 = ? ÷ (1.6 × 0.499)
(a). 314 (b). 334 (c). 344
(a) 29 (b) 33 (c) 45
(d). 324 (e). 354
(d) 37 (e) 80
Q.543 16.992 - 88.99 - 13.942 + 3.94 = 1.922 × ? ÷ 10.98
(a). 32 (b). 28 (c). 38 Q559. √1296.002 ÷ 8.996 ÷ 9.98 + 39.4 = ?
(d). 18 (e). 22 (a) 80 (b) 8 (c) 4
Q.544 11.922 + 15.982 – ?2 = 10.102 × 3.03 (d) 120 (e) 40
(a). 5 (b). 15 (c). 10 Q.560 79.98% of 299.99 + 39.94% of 349.99 = ? + 59.99% of 399.99
(d). 25 (e). 20 (a) 160 (b) 180 (c) 210
Q.545 24.95 - {16.98 - 11.98 ÷ (5.15 + 17.98 - 16.98)} = ? (d) 140 (c) 120
(a). 15 (b). 25 (c). 20 Q.561 3.03 – (9.09 – 2.93 × 7.98 ÷ 1.92) = ?
(d). 30 (e). 10 (a). 12 (b). 8 (c). 21
Q.546 21.03 × 6.97 + 12% of 401.09 - 2/7 of 699.81 = ? - 105.07 (d). 2 (e). 6
(a). 80 (b). 90 (c). 100 Q.562 241.98 – [18.962 – (227.93 + 171.98) ÷ 1.92] = ?2
(d). 110 (e). 120 (a). 7 (b). 11 (c). 15
Q.547 104.042 - 96.022 + 53.992 - 46.072 = 100 + ? (d). 9 (e). 5
(a). 1200 (b). 2000 (c). 2300
(d). 1950 (e). 2150 Q. 563 (21.93)2 - (7.98)3 - √1295.96 + 63.98 = ?
Q.548 2% of (298.99 + 15.982 - 255.11) = ? (a). 2 (b). 10 (c). 5
(a). 2 (b). 3 (c). 4 (d). 15 (e). 0
(d). 5 (e). 6 Q.564 143.99 ÷ (5.93)2 × 7.92 – 23.97 = √?
Q.549 13.97 × √9.08 + 300.04 - 99.97 = ? (a). √8 (b). 8 (c). 64
(a). 235 (b). 248 (c). 255 (d). 512 (e). 16
(d). 242 (e). 225 Q565. 149.98% of 30.03 + 25.05% of 139.96 = ?
Q.550 18.02 - (? - 19.01) + 10.982 = 120.04 (a) 85 (b) 70 (c) 65
(a). 41 (b). 38 (c). 35 (d) 80 (e) 75
(d). 43 (e). 31 4
Q566. √0.000128 ÷ 0.06 × 972.0002 = ?
Q.551 8787 ÷ 343 × √50 = ? (a) 0.12 (b) 1.2 (c) 2.4
(a). 75 (b). 145 (c). 150 (d) 12 (e) 24
(d). 175 (e). 225 Q567. 1444.9998 × 80.0001 ÷ 99.999 = (? )2
Q.552 8399.99 ÷ 12.02 - 199.99 = ? (a) 34 (b) 30 (c) 24
(a). 500 (b). 600 (c). 200 (d) 12 (e) 17
(d). 400 (e). 450 Q568. (74.98)2 − (24.99)2 − (30.02)2 = (? )3
Q.553 √2115.88 × (4.96) + 896.32 = ?
2
(a) 14 (b) 15 (c) 18
(a). 2046 (b). 3114 (c). 2678 (d) 17 (e) 16
Q569. 3 5 × 3 4 × 1 9 − 6 25 ÷ 2 23 = ?
3 3 2
(d). 2298 (e). 3482
Q.554 666.66 + 66.66 + 16 + 0.66 + 0.06 ÷ 0.001 = ? (a) 17 (b) 20 (c) 14
(a). 810 (b). 755 (c). 750 (d) 24 (e) 10
(d). 756 (e). 850 Q.570 47.88 + 84.07 - 99.95 = 7.86 × ?
Q555. √? = (1346.92 + 46.94) ÷ 99.9 − 6.98 (a). 4 (b). 3 (c). 8
(a) 121 (b) 441 (c) 1024 (d). 2 (e). 6
(d) 49 (e) 196 Q. 571 (11.90% of 1200) ÷ 6.10 = (?)3 – 2.91
1 (a). 27 (b). 9 (c). 11
Q556. 32.01² × 5123 × 33.992 ÷ (29 × 16.972 ) = 2? (d). 5 (e). 3
(a) 3 (b) 4 (c) 9 Q.572 54.86 + 19.98 + (?)2 = 28.01% of 700
(d) 10 (e) 6 (a). 121 (b). 11 (c). 10

For Free Study Material & Quizzes visit : www.mathsbyarunsir.com


For any queries or suggestions email us @ mathsbyarunsir003@gmail.com or whatsapp @ 8881331466
(d). 12 (e). 15 Q589. 44.04 ÷ 3.97 × 7.49 ÷ 2.54 + 3 = √?
Q. 573 (18.64) + (8.32) ÷ (1.85)2 = ?2 + (1.23)2
2 2
(a) 6 (b) 36 (c) 24
(a). 24 (b). 19 (c). 15 (d) 1296 (e) 216
(d). 22 (e). 26 2
Q. 574 (453.81 + 29.89)1/2 ÷ 10.85 = (0.1 × ?) ÷ 4 Q590. (√? )3 + (2.01)10 + (4.99)3 = 1156.92
(a). 85 (b). 8 (c). 80 (a) 256 (b) 128 (c) 512
(d). 0.8 (e). 2 (d) 1024 (e) 1296
Q.575 49.99% of (152.01 + 63.99 ÷ 2) × 7.01 = 46 × ? Q591. 8.07% of 449.97 + 5.96% of 1449.97 = 24.98% of ?
(a). 16 (b). 22 (c). 18 (a) 576 (b) 378 (c) 550
(d). 14 (e). 24 (d) 625 (e) 492
Q.576 59.99% of 310 – 102.97 + 30.08% of 260 = ? Q592. 84.04% of 1649.97 + 19.98% of 870.08 = ?
(a). 143 (b). 161 (c). 153 (a) 1600 (b) 1560 (c) 1410
(d). 171 (e). 139 (d) 1375 (e) 1800
Q.577 79.74 ÷ 4.25 + 772.80 = 1009.14 - (?)3 Q593. 64.07% of 249.96 + 35.93% of 150.17 = ?
(a). 7 (b). 12 (c). 6 (a) 214 (b) 180 (c) 235
(d). 4 (e). 8⇒ ? = 6 (d) 245 (e) 340
Q.578 2998.88 ÷ 101.02 – 4.999 × 2.8998 + 6.978 = ? Q594. √2498 × √626 ÷ √99 = x % of 2500
(a). 32 (b). 26 (c). 28 (a) 5 (b) 10 (c) 12.5
(d). 42 (e). 22 (d) 2 (e) 8
Q. 579 (171.94 ÷ 7.99) × 12.01 = ? × 5.81 + 23.99 410 1
Q595. 55.82 × 69.89 + 499.9 – 127.9 = x% of 5600 × 2
(a). 39 (b). 36 (c). 23
(d). 42 (e). 48 (a) 17.5 (b) 25 (c) 10
Q.580 255.96 ÷ 1.923 × ? = 15.66% of 2999.99 (d) 15 (e) 8
(a). 17 (b). 15 (c). 25 Q596. 56.09 × ? + 25.98 % of 450.10 + √15.99 = (31.08)2
(d). 22 (e). 20 (a) 5 (b) 25 (c) 10
Q.581 598.01 ÷ 22.93 + √143.94 = √? + 287.88 ÷ 15.89 (d) 15 (e) 40
545.93
(a). 200 (b). 350 (c). 600 Q597. ? + 56.09% 𝑜𝑓 549.92 =
(d). 500 (e). 400 28.06 % 𝑜𝑓 1249.98
Q. 582 (683.94 + 387.01) ÷ 2.93 + 162.93 - 163.99 = ? (a) 17 (b) 13 (c) 21
(a). 296 (b). 214 (c). 308 (d) 25 (e) 28
(d). 356 (e). 454
Q.583 4499.95 + 499.95 ÷ 24.95 – 919.92 = 17.91 × ? Q598. (12.09)3 + (16.21)2 - √? + (25.93)2 = (36.11)2 - 79.98
(a). 150 (b). 120 (c). 100 (a) 30 (b) 46 (c) 38
(d). 200 (e). 180 (d) 50 (e) 54
Q. 584 (979.8 + 888.08) ÷ 4.04 + 632.93 = 683.84 + 25.96 × ? Q599. ? % of 340.09 + 11.98 % of 2174.99 + √3721.09 = (21.09)2
(a). 20 (b). 18 (c). 22 (a) 35 (b) 42 (c) 45
(d). 26 (e). 16 (d) 30 (e) 28
Q.585 96.98 + 709.99 – 142.93 = 3.98 × ? Q600. 547.05 + 243.02 - ? = 24.89 % of 2584.11
(a). 154 (b). 150 (c). 166 (a) 128 (b) 144 (c) 120
(d). 160 (e). 172 (d)118 (e) 156
Q.586 325.05 – [(11.92)2 – 74.95] = (?)2 – 67.8
(a). 16 (b). 14 (c). 22
(d). 18 (e). 24
Q.587 620.99 ÷ 26.97 × 1.92 – 36.97 = √?
(a). 64 (b). 36 (c). 144
(d). 121 (e). 81
Q.588 673.8 – 457.2 + 518.9 = ? + (18.89)2
(a). 324 (b). 348 (c). 382
(d). 375 (e). 368

For Free Study Material & Quizzes visit : www.mathsbyarunsir.com


For any queries or suggestions email us @ mathsbyarunsir003@gmail.com or whatsapp @ 8881331466
For Free Study Material & Quizzes visit : www.mathsbyarunsir.com
For any queries or suggestions email us @ mathsbyarunsir003@gmail.com or whatsapp @ 8881331466
For Free Study Material & Quizzes visit : www.mathsbyarunsir.com
For any queries or suggestions email us @ mathsbyarunsir003@gmail.com or whatsapp @ 8881331466
Solutions Ans.10
Ans1.(d) ? ≈ (18 x 656 x 144)/(82 x 24)
Sol. ? % 𝑜𝑓 25% 𝑜𝑓 40% 𝑜𝑓 (144 + 6) = 2.25 = 18 x (656/82) x (144/24) = 18 x 8 x 6 = 864
Sol.11
? % 𝑜𝑓 25% 𝑜𝑓 40% 𝑜𝑓 150 = 2.25 42
× 650 + (3)5 – √? ≈ 2 × (4)4
100
? % 𝑜𝑓 25% 𝑜𝑓 60 = 2.25 273 + 243 – √? = 2 × 256
? % 𝑜𝑓 15 = 2.25 √? = 516 – 512
? ≈ 15 √? = 4
Ans2.(a) ? = 16
Sol. (8)3 + (8)2 = −100 + (? )2 Sol.12
? 72
512 + 64 + 100 = (? )2 × 750 + (15)2 – 100 × 450 ≈ (8)2 + (2)5
100
? ≈ 26 ?
× 750 + 225 – 324 = 64 + 32
100
?
Ans3.(e) × 750 = 96 + 99
33×42×5 100
Sol. + 14 =? % 𝑜𝑓 280 195×100
11×15 ?=
750
? ≈20 ? = 26
Sol.13
Sol.4 728 24
× 15 + 100 × 550– 38 ≈ (22)2
97 + 997 − 100 + 1000 + 6 = 400 ×? ?
728×15
?≈ 5 = 484 + 38– 132
?
728×15
?= 390
Sol5. 10% 𝑜𝑓 300 = (? )3 + (? ) ? = 28
30 = (? )3 + (? ) Sol.14
?≈ 3 36 ×? +33 × 4 + √441 ≈ (21)2
Ans6.(b)
36 ×? = 441– 21– 132
Sol. 288
(17)2 + (21)2 + √2916 ≈ (? )2 ? = 36
⇒ 289 + 441 + 54 = (? )2 ?=8
⇒ ? = √784 = 28 Sol.15
8
Ans.7 568 + 330 – (12)2 ≈ 100 ×?
8
Sol. ×? = 898– 144
100
60% of 960 + 65% of 240 ≈?% of 6100 754×100
60 65 ?
⇒ 100 × 960 + 100 × 240 = 100 × 6100 ?= 8
732 ? = 9425
⇒?= = 12
61 Sol.16
Sol.8
(13)² + (21)² - 30 × 7 ≃ ? – 520 + 150
⇒ √(13)2 + 28 ÷ 4– (27)+? ≈ 16 169 + 441 – 210 = ? – 370
? = 770
⇒ √169 + 7– 27 + ? = 16
Sol.17
⇒ 149 + ? = 256 18 ?
× 1900 + × 1150 = 684 − 112
⇒ ? = 107 100 100
Ans. 9 ?
× 115 =572 − 342
10
Sol. ? = 20
⇒ 286 × 10 + 65 × 54 ≈ ? + 164 Sol.18
⇒ 2860 + 3510 = ? + 164 440
⇒ ? = 6206 = 512 − 8 − 484
?

For Free Study Material & Quizzes visit : www.mathsbyarunsir.com


For any queries or suggestions email us @ mathsbyarunsir003@gmail.com or whatsapp @ 8881331466
440
? = 20
Sol.25
?= 22 56 125 4
Sol.19 × ? + 100 × 96 = (14)2 − √1296
100
(? )2 − 432 = 1240 + 482 − 1313 56
× ? + 120 = 196 – 6
100
(? )2 = 409 + 432 56
(? )2 = 841 × ? = 190 − 120
100
? = 29 56
× ? = 70
100
70×100
Sol.20 ? = 56
11 ? = 125
30 × √?+ √961 = × 1300 − 22 Sol.26
100
30 × √?+ 31 = 143 − 22 28 ?
× 450 + × 1200 ≃ 200 + 226
30 × √? = 90 100 100
? × 12 = 426 – 126
?=9 300
? = 12 = 25
Sol.21 Sol.27
1730 + 1270 + ? ≃ 250 × 20
12 4
?3 × 18 + 100 × 450 = (14)2 + √16 ? = 5000 – 3000
? = 2000
?3 × 18 + 54 = 196 + 2 Sol.28
?3 × 18 = 198 − 54 1150 + √? – 15 ≃ 90 × 13
?3 × 18 = 144 1150 + √? – 15 = 1170
?3 = 8
?=2 √? – 15 = 20
? = 415
Sol.29
Sol.22
? 3 (? )2 + (12)2 + (6)2 + (8)3 ≃ 500 + 448
+ (22)2 = (24)2 + √64 (? )2 = 948 – 144 – 36 – 512
14
?
14
+ 484 = 576 + 4 (? )2 = 256
? ? = 16
= 580 − 484
14 Sol.30
? = 96 × 14
? = 1344 √410 + 220– √25 ≃ ? +15
Sol.23
? 20 ? = √630– 5 – 15
× 1355 + × 1210 = (28)2 ? = 25 – 15 = 10
100 100
?
× 1355 + 242 = 784 Sol.31
100 (27.02)2 + 2669÷ 2.98 – 218.9 = 1869.6 - ?
?
× 1355 = 784 − 242 (27)2 +2670 ÷ 3 -219 =1870 - ?
100
? 729+ 890 -219=1870 - ?
× 1355 = 542
100 1400=1870 -?
542×100
? = 1355 ? = 1870 -1400 = 470
? = 40 Sol.32
Sol.24 17998÷ 4.99×1.52+ 7199.2 -3448.6
35 ≈18000÷ 5×1.52+ 7200 -3450
? + 100 × 1280 = (24)2 + √196
≈3600× 1.5 +3750
? + 448 = 576 + 14
≈ 5400+ 3750
? = 590 − 448
? = 142 ≈9150
Sol.33

For Free Study Material & Quizzes visit : www.mathsbyarunsir.com


For any queries or suggestions email us @ mathsbyarunsir003@gmail.com or whatsapp @ 8881331466
3599÷8.98 +1244.9×2.98+(17.99) 2 ?² + 29 × 5 – 115 ≈ 9% of 400 + 523
≈3600÷9 + 1245×3+324 9
⇒ ?² + 30 ≈ 100 × 400 + 523
≈400+3735+324
⇒ ?² + 30 ≈ 36 + 523
≈4460
⇒ ?² ≈ 529
Sol.34
⇒ ? ≈ 23
√7567.4× 4.52 +161.8 × 7.97= 2399.6 -? Sol. 41
≈87× 4.5 +162×8=2400-? ≈ 340
≈391.5+1296 =2400-? Sol.42
1687.5= 2400-? ≈ 280 +500
? ≈2400 -1687.5 =712.5 ≈ 780
≈710 Sol43.
Ans.35 ≈
25+25
Sol. √16.9 × 44.9 + 4798.5 ÷ 3.98 + 59.8 5
≈ √17 × 45 + 4800 ÷ 4 + 60 ≈ 10
≈ √765 + 1200 + 60 Sol44. ≈ 990 + 77.5
≈ √2025
≈45 ≈ 1070
Sol45. ≈ 50
Sol.36 Sol.46
(2262.98 ÷ 31.01) × (510.01 ÷ 169.99) = ? + 59.91% of 159.988 32 ÷ 4 ÷ 10 + 29 = ?
(2263 ÷ 31) × (510 ÷ 170) ≈ ? + 60% of 160 ? = 8 ÷ 10 + 29
⇒ 73 × 3 ≈ ? + 96 ? = 29.8 ≈ 30
Sol47.
⇒ ? ≈ 219 – 96 ≈ 123
Sol.37 √? = (1248.28 + 51.7) ÷ 99.9 − 7.98
620.99 ÷ 23.01 + 28.11% of 749.899 = ? + 36.001% of 349.93 √? = (1300 ÷ 100) − 8
621 ÷ 23 + 28% of 750 ≈ ? +36% of 350 √? = 5
36×350
⇒ 27 + 210 ≈ ? + 100 ? = 25
Sol48.
⇒ 237 ≈ ? + 126 36√𝑥 + 32√𝑥 = 11 × 𝑥
68

⇒ ? ≈ 111 68
Sol.38 68√𝑥 = 11 × 𝑥
(89.99)2 – (67.03)2 – 64.99% of 780.02 = 31.93 × ? √𝑥 = 11
𝑥 = 121
(90)² - (67)² - 65% of 780 ≈ 32 × ? Sol49.
⇒ 8100 − 4489 – 507 ≈ 32 × ? (? )2 ≈ 96 − 4 + 104 = 196
⇒ 3611 – 507 ≈ 32 × ? ? ≈ 14
⇒ ? ≈ 97 Sol50. 129 − 224 = −95
Sol.39 Ans.51
√89.99 × 23.99– (112.01 × 17.9) + 480.93 = ?
√90 × 24 – (112 × 18) + 481 ≈?
⇒ √2160– 2016 + 481 ≈ ?
⇒ ? ≈ √625 ≈ 25

Sol.40
?² + 29.07 × 5.114 – 115.117 = 8.9% of 400.118 + 522.9

For Free Study Material & Quizzes visit : www.mathsbyarunsir.com


For any queries or suggestions email us @ mathsbyarunsir003@gmail.com or whatsapp @ 8881331466
?3 × 15 = 1444 + 96 −580
Ans.52
?3 × 15 = 960
?3 = 64
?=4

Sol.59
1268 + ?3 = (12)3 + 52
?3 = 1780 − 1268
?3 = 512
?=8
Ans.53
Sol.60
?
+ 122 − √1936 = (16)2
14
?
= 256 + 44 − 144
14
?
= 156
14
? = 2184
Ans.54 Sol.61
420.12 1
= (361.11)2 − 22.01 × 6.99 + 141.99
?
420 1
⇒ ≃ (361)2 − 22 × 7 + 142
?
420
⇒ ≃ 19 − 154 + 142
?
420
⇒ ≃7
?
420
⇒ ?≃ ≃ 60
7
Ans.55
Sol.62
? + 185.10 – 79.09 = (23.01)² - 70.01% of 139.99
70
⇒? + 185 − 79 ≃ (23)2 − × 140
100
⇒ ? + 106 ≃ 529 − 98
⇒ ? ≃ 431 − 106 ≃ 325
Sol.56
125 75
× 440 + 100 × 840 + 2 = 40 ×? Sol.63
100
550 + 630+2 = 40 × ? ? ≈ 30 √783.98 + (22.02)2 = 2 × (? )2
Sol.57 ⇒ √784 + (22)2 ≃ 2 × (? )²
? × 128 + 1728 = (13)3 + 171 ⇒ 28 + 484 ≃ 2 × (? )²
⇒512 ≃ 2 × (?)²
? × 128 = 2197 + 171 − 1728
640 ⇒ (?)² ≃ 256
? = 128
⇒ ? = 16
?=5
Sol.58 Sol.64
125
?3 × 15 + 100 × 464 = (38)2 + 96 44.04% of 349.98 + 205.01% of 140.01 = (?)²

For Free Study Material & Quizzes visit : www.mathsbyarunsir.com


For any queries or suggestions email us @ mathsbyarunsir003@gmail.com or whatsapp @ 8881331466
44 205 ⇒ 46 = 4(? −3)
⇒ × 350 + × 140 ≃ (? )2
100 100 ⇒ (?) – 3 = 6
⇒ 154 + 287 ≃ (?)²
⇒ (?) = 9
⇒ 441 ≃ (?)² Sol.70
⇒ ? = 21 1 1
Sol.65 (25.11)2 × (80.87)1/4 × (49.22)2 + 3.13 × 7.01 + 2 = 2?
? Taking the approximate values,
+ 44.01 + 139.99 = 78.09 + 249.01 + 86.99
4 25.11 ≈ 25, 80.87 ≈ 81, 49.22 ≈ 49, 3.13 ≈ 3, 7.01 ≈ 7
? 1 1
⇒ + 44 + 140 ≃ 78 + 249 + 87 ⇒ (25)2 × (81)1/4 × (49)2 + 3 × 7 + 2 = 2?
4
? ⇒ 5 × 3 × 7 + 23 = 2?
⇒ ≃ 414 − 184
4 ⇒ 2? = 128
?
⇒ ≃ 230 ⇒ ? ≃ 920 ∴?=7
4
Ans.66 Sol.71
Given expression is Given expression becomes,
11.922 + 15.96 of 2.93 – 19.92 ÷ 3.99 = ? ⇒ (18 × 5 – 10) – ( ¾ of 16) = 33 + ?
⇒ 122 + 16 of 3 – 20 ÷ 4 ⇒ (90 – 10) – (3/4 × 16) = 33 + ?
⇒ 144 + 16 × 3 – 5 ⇒ (80) – (3 × 4) = 33 + ?
⇒ 144 + 48 – 5 ⇒ 80 – 12 = 33 + ?
⇒ 187 ⇒ 68 = 33 + ?
Ans.67 ⇒ ? = 68 – 33
Given expression is ⇒ ? ≈ 35
851.92 - 12.93 × 7.98 - 101.92 × 2.93 - 0.91 = ?2 Ans.72
⇒ 852 - 13 × 8 - 102 × 3 - 1 = ?2 Given expression is,
1
⇒ 852 - 104 - 306 -1 = ?2 (66)3 + (3.01)3 – 69.98 × 2.02 + 30% of √6399 = ?
⇒ 441 = ?2 Taking the approximate values as,
⇒ 21 = ? 66 ≈ 64, 3.01 ≈ 3, 69.98 ≈ 70, 2.02 ≈ 2, 6399 ≈ 6400
Ans.68 1
⇒ (66)3 + (3)3 – 70 × 2 + 30% of √6400 = ?
Given expression is
3.94 + 1/12.93 of {2599.96 ÷ (117.98 – 107.98)} = ? ⇒ 4 + 27 – 140 + 24 = ?
⇒ 4 + 1/13 of {2600 ÷ (118 – 108)} = ? ∴ ? = -85
Ans,73
⇒ 4 + 1/13 of {2600 ÷ 10} = ?
We can also write values as:
⇒ 4 + 1/13 of 260 = ?
90.77 ≈ 91, 116.81 ≈ 117, 13.40 ≈ 13, 44.16 ≈ 44, 152.67 ≈ 153
⇒ 4 + 20 = ? Given expression becomes,
∴ ? = 24 ⇒ 91 + 117 ÷ 13 – 44 = 153 - ?
Ans.69
⇒ 91 + 9 – 44 = 153 - ?
Given expression is
⇒ 100 – 44 = 153 - ?
3
(63.94 × 15.96 ÷ 255.96)2 = √[3.94(? −3) ] ⇒ 56 = 153 - ?
3 ⇒ ? = 153 – 56
⇒ (64 × 16 ÷ 256)2 = √[4(? −3) ] ⇒ ? ≈ 97
3 Ans.74
⇒ (64 × 16/256)2 = √[4(? −3) ] Given expression is,
3 √?× 45.16 = 250.08 × 15.01 + 138.21 × 24.88
⇒ 42 = √[4(? −3) ]
√? × 45 ≈ 250 × 15 + 138 × 25

For Free Study Material & Quizzes visit : www.mathsbyarunsir.com


For any queries or suggestions email us @ mathsbyarunsir003@gmail.com or whatsapp @ 8881331466
√? ≈ 160 ? × 14 = 864 + 755 − 695
924
? = 25600 ? = 14
Ans.75
? = 66
[1279.98)2 ÷ 32.23 × 23.94] ÷ 47.98 = ?2 Sol.82
⇒ [(1280)2 ÷ 32 × 24] ÷ 48 =?2 18
× 200 +
? 40
× 600 = 100 × 750
100 100
⇒ [(1280)2 × 24]/(32 × 48) = ?2 ? × 6 = 300 − 36
⇒ (1280)2 /64 = ?2 264
?= 6
⇒ ? = 1280/8 = 160
Ans.76 ? = 44
Sol.83
599.99 × 3/2 of 19.92% of (109.99 – 9.99) = ? 54 78
= 600 × 3/2 of 20% of 100 = ? × 7000 − 100 × 4000 + 16 = ?2
100
= 600 × 3/2 of 20/100 of 100 = ? 2
? = 3780 − 3120 + 16
= 600 × 3/2 of 20 = ? ?2 = 676
= 600 × 3/2 × 20 = ? ? = 26
= 600 × 30 = ?
= 18000 = ? Sol.84
Ans.77 Given expression is
(2.992 × 3.992 × 4.99) ÷ 35.99 = (? )2 – 79.99 286.97 × 56.97 + 240.91 = √624.95 × ?
⇒ (32 × 42 × 5) ÷ 36 = (? )2 – 80 ⇒ 287 × 57 + 241 = √625 × ?
⇒ (? )2 = ((9 × 16 × 5) ÷ 36) + 80 ⇒ 16359 + 241 = 25 × ?
⇒ (? )2 = (720 ÷ 36) + 80 ⇒ 25 × ? = 16600
⇒ (? )2 = 20 + 80 = 100 ⇒ ? = 16600/25
⇒ (?) = √100 = 10 ⇒ ? = 664
Sol.78 Sol.85
90 15
324.95 – [(11.99)2 – 74.9] = (? )2 – 67.99 100
× 400 + 100 × 4800 − 677 = ?
⇒ 325 – [(12)2 – 75] = (? )2 – 68 ? = 360 + 720 − 677
⇒ 325 – [144 – 75] + 68 = (? )2 ? = 403
Sol.86
⇒ 325 – 69 + 68 = (? )2
? × 50 + 540 = 760 + 180
⇒ 324 = (? )2 400
⇒ 18 = ? ? = 50
Sol..79 ?= 8
Given expression is Sol.87
500
34.99 × 17.98 ÷ 6.97 – 72.93 = ? – 128.99 (2? )2 + × 112 = 1240 − 280
100
⇒ 35 × 18 ÷ 7 - 73 = ? -129 (2? )2 = 1240 − 280 − 560
⇒ 35 × (18/7) - 73 = ? -129 (2? )2 = 400
⇒ ? = 35 × (18/7) - 73 + 129 ? = 10
⇒ ? = 90 - 73 + 129
Sol.88
⇒ ? = 146 1640 ?
Sol.80 + 400 × 1000 = 780
41
16 × 50 − 18 × 32 + 196 = ? ? × 2.5 = 780 − 40
740
? = 800 − 576 + 196 ? = 2.5
? = 420 ? = 296
Sol.81 Sol.89
36
? + 14 + 695 = 100 × 2400 + 755 991 − 648 = (? )3

For Free Study Material & Quizzes visit : www.mathsbyarunsir.com


For any queries or suggestions email us @ mathsbyarunsir003@gmail.com or whatsapp @ 8881331466
(? )3 = 343 256
– 12 = √𝑥
16
?= 7
Sol.90 √𝑥 = 4
28 x = 16
× 225 + 81 = ?2
100
63 + 81 = ?2 Sol98. (5)2 + 𝑥 = √3025
25 + x = 55
?2 = 144
? = 12 x = 30
Sol.91 Sol99. 49 + x – 32 = (9)²
? x + 17 = 81
+ 196 = 225 −5 x = 64
18
? Sol.100
= 24
18 1088 648 35
? = 432 ? = 630 × 204 × 16
? = 12
Sol.92 Sol.101
3
32 × ? + 324 =
80
× 1405 √729 + √1024 = ?2 + 5
100
?2 = 9 + 32 − 5
32 × ? + 324 = 1124
? = √36
32 × ? = 800 ?= 6
? = 25 Sol.102
Sol.93 36 1
× 1500 × 18 = ? − 112
56 68 100
× ? + 144 = 100 × 500 30 + 112 = ?
100
56
× ? = 340 − 144 ? = 142
100
56
× ? = 196 Sol.103
100
196×100 15 60
? = 56 × 100 × 80 = 240
50−?
? = 350
50 − ? = 30
Sol. 94
? = 20
?3 × 18 + 25 = 2275 Sol.104
?3 × 18 = 2250 1 2
2250 [(2197)3 ] = ? − 147
?3 = 18
? = 147 + 169
?3 = 125
? = 316
?=5 Ans.105
Sol.95 80 45
𝑥 × 550 + 100 × 800 = ?2 × 8
× (800 − 420) + 13 = 260 100
100
380×𝑥 ?2 × 8 = 440 + 360
= 247 800
100 ?2 = 8
247×100
x = 380 ? = 10
x = 65 Sol.106
9 195 660
× 11 × 36 = ? − 64
Sol.96 75
68 + 𝑥
? = 39 + 64
98 + 4 = 138 ? = 103
68 + x = 40 × 4 Sol.107
x = 160 – 68 (39 × 13) − 729 + 418 = ?2
x = 92 ?2 = 507 − 729 + 418
?2 = 196
Sol.97 ? = 14

For Free Study Material & Quizzes visit : www.mathsbyarunsir.com


For any queries or suggestions email us @ mathsbyarunsir003@gmail.com or whatsapp @ 8881331466
Sol.108 ? = 330 × 66
1
832
148 + 64 − 97 = ?3 ?= 5
?3 = 51 + 13 Sol.118
2172
?3 = 64 684 + + 50 × 5 = ?2
6
?= 4
Sol.109 ? = √1296
80 ? ? = 36
× 400 + 100 × 650 = 580 Sol.119
100
? 80 1
× 650 = 580 − 320 (100 × 250) × 5 × (5)2 = ?
100
260
? = 650 × 100 ? = 40
? = 40 Sol.120
40
Sol.110 66 + 100 × 475 = (? )2
12
200 + 100 × 500 − 4 = ?2 256 = (? )2
256 = ?2 ? = 16
Sol.121
? = 16
(√502 + 174) × 15 = ? + 220
Ans.111 390 − 220 = ?
84 26
+ 100 × 800 + 2 = 216 ? = 170
? Sol.122
84
= 216 − 210
? ? ≈ (√225 − √81)(√16 − √9)
? = 14 ? ≈ (15 − 9)(4 − 3)
Ans.112
? 20
=6
× 550 + 100 × 650 = 350 Sol.123
100
? 480 25
× 550 = 350 − 130 + 100 × 500 − 90 = 55
100 ?
220×100 480
? = 550 = 40% = 20
?
Sol.113 ? = 24
30 Sol.124
5 × (? + 12) + 100 × 900 = 360
⇒ 65% of 240 √1936 = ?
5 × (? + 12) = 360 − 270
⇒ 200 = ?
5 × ? = 90 − 60 Sol.125
?= 6
Sol.114 342 + 576 – ?² = (12)² – √100
36 918 – ?² = 144 – 10
×? + 324 = 774
100 ?² = 918 – 134
36
×? = 450 ?² = 784
100
? = 1250 ? = 28
Sol.115 Sol.126
50 12
190 + 100 × 540+? = 630 24 × ? + 100 × 225 = (19)² + √676
? = 630 − 460 = 170 24 × ? + 27 = 361 + 26
Sol.116 24 × ? = 387 – 27
276 80 32 24 × ? = 360
× 115 ×? = 100 × 100
12 ? = 15
1
? = 64 × 16 Sol.127
36
?= 4 (12)³ + ?² = 100 × 5200
Sol.117 36
184 594 1728 + ?² = 100 × 5200
(284 + )÷( )= ?
4 9 1728 + ?² = 1872

For Free Study Material & Quizzes visit : www.mathsbyarunsir.com


For any queries or suggestions email us @ mathsbyarunsir003@gmail.com or whatsapp @ 8881331466
?² = 1872 – 1728 ?
?² = 144 300 × + 150 × 20 ≃ 420 × 30 + 350 × 40
9
? = 12 300
Sol.128 ×? + 3000 = 12600 + 14000
9
300
472 + 548 – ?² = (18)² + √400 ? × = 23600
1020 – ?² = 324 + 20 9
23600
?² = 1020 – 344 ?= × 3 = 708
?² = 676 100
Sol.137
? = 26
90
Sol.129 (? )2 + × 2000 + 5 × 80 ≃ 130 × 20
96 100
121 + ? = × 500 (? )2 + 1800 + 400 = 2600
100
? = 480 – 121 (? )2 = 2600 − 2200 = 400
? = 359 ? =20
Sol. 130 Sol.138
30 ? 1590 ?
× 180 + (17)2 = 100 × 8575 + × 140 ≃ 180 × 8
100 3 100
? 7
54 + 289 = × 8575 530 + ×? = 1440
100
100 5
343 × 8575 =? ? = 650
?=4 Sol.139
Sol.131 750
840 + 1260 – 120 × 20 ≃ 100 × 2−?
16 13
(32)2 = (? )3 − (15)2 + × 1600 + 100 × 5000 2100 – 2400 = 15 - ?
100
3
1024 =? – 225 + 256 + 650 ? = 315
?3 = 343
?=7 Ans.140
Sol132
5
. 52 × 18– √2304 = (30)? − 100 × 240
936 – 48 + 12 = (30)?
900 = (30)? Ans.141
?=2
Sol.133
456 45
?
= 13 × 32 − 100 × 840
456
= 416 − 378
?
456 Ans.142
= 38
?
? = 12
Sol.134
√831 − √2209 = √?+ √529
√784 = √?+ 23
? = 25
Sol.135 Ans.143
24 90
× 550 + × 910 ≃ ? + 251
100 100
132 + 819 ≃ ? + 251
? = 700 Ans.144
Sol.136

For Free Study Material & Quizzes visit : www.mathsbyarunsir.com


For any queries or suggestions email us @ mathsbyarunsir003@gmail.com or whatsapp @ 8881331466
9900 80
× 34 × 0.01 + 98 = ×?
11×3 100
200 ×100
?= ≈ 250
80

Ans.145
Sol154.
31.67 × ? + 22.122 = 174.75% of 240.03 + 56.09 × 7.99 5436 36 108
32 × ? + 222 ~ 175% of 240 + 56 × 8 + 18 + 54 + 100 × 1000 = × ? + 216
302 6
32 × ? ~ 1.75 × 240 + 448 – 484 ?=
(18 + 72 + 360 – 216)
≈ 13
32 × ? ~ 868 – 484 18
Ans.155
? ~ 384/32 ~ 12 3
≈ √1331 + 44 − 15+? = 189
Ans.146 ? = 189 - 29 – 11
?2 = (540.09 × 179.98) ÷ (24.082 – 11.782) + 16.05% of 2500.01 ? = 149
?2 ~ (540 × 180) ÷ (242 – 122) + 16% of 2500 Sol.156
(22) 2 +449−33 3
?2 ~ (540 × 180) ÷ (36 × 12) + 0.16 × 2500 ≈ = (5)3 + √512−?
9
?2 ~ 225 + 400
⇒ 100 = 133 – ?
?2 ~ 625
? ~ 25 ⇒ ? = 33
Sol.157
Ans.147 ≈ (46)² + (1016÷ 4) + ? = (14)³
{(5/7) × 840 – (4200 ÷ 35)} × 5 = ? + 1600
{(5 × 120) – 120} × 5 = ? + 1600 ? = 2744 − 2116 − 254
? = 2400 – 1600 ? = 374
? = 800 Sol.158
Ans.148 79+88+789−878
≈ = 13
?
(155.34 + 234.78) ÷ 12.65 = ? ÷ 4.99 ?= 6
(155 + 235) ÷ 13 ~ ? ÷ 5
? ~ 30 × 5 Sol.159
? ~ 150
Ans.149 ≈ 9 + (?)³ – 25 = 42 +6
= (?)³ = 48 + 25 – 9
{44.78% of (11.67 × 54.78)} ÷ 2.65 = ? + 19.25
(?) = 4
{45% of (12 × 55)} ÷ 3 ~ ? + 19
Ans.160
(0.45 × 660) ÷ 3 ~ ? + 19 120 70 1000
? ~ 99 – 19 × 120 + 100 × 80 ≈ ?×
100 100
? ~ 80 144 + 56 ≈ ? × 10
Sol150.
? 30
? ≈20
× 8000 + 1681 − × 4150 = 676
100 100
100 Sol.161
? = (676 - 436)× 8000 ≈ 3
Sol.151 2 + 5 × 7 – 432 + 532 ≈ ?
250+ ? 137 ≈?
+ 417 + 400 = 625 + 20 × 10 Sol.162
50
? = (625 + 200 – 817)×50 – 250 ≈ 150 42×22 20
+ 100 × 530 – 43 ≈ ?
7
132 + 106 – 43 ≈ ?
Sol.152 484 + 12.5× 20 − 125 −?2 × 4 = 225
? ≈ 195
734−350 Sol.163
?=√ ≈ 10
4
23 × 23 + 12 × 8 ≈ ?²
Sol.153 ? ≈ 25
Sol.164

For Free Study Material & Quizzes visit : www.mathsbyarunsir.com


For any queries or suggestions email us @ mathsbyarunsir003@gmail.com or whatsapp @ 8881331466
25×2000 25×2000 1 1020
+ – 3 × 100 ≈ ? +50 ?= 3
= 340
100 100
1000 – 33 – 50 ≈ ?
Sol.174
? ≈ 917 790
Sol.165 √?+ 100 × 120 + 200 ≃ 2180 – 1024
420 √?+ 948 + 200 ≃ 1156
√64 × 36 + – 540 =? – 800
6 √? = 8
? = √2304 + 70 – 540 + 800 ? =64
? = 378 Sol.175
Sol.166 ?² + 190.98 – 19.01 × 6.97 = 316.99 × 1.99
16 ? ?² + 191 – 19 × 7 ≈ 317 × 2
× 1600 + × 1200 = 20 × 122
100 100 ?² ≈ 576
256 + ? × 12 = 2440
2184 ? ≈ 24
? = 12 = 182
Sol.167 Sol.176
(8)³ + (15)² – (12)² = ? – 1220 – 1750 768.06 ÷ 11.97 × √256.05 −58.05 = ?
512 + 225 –144 = ? – 2970
768 ÷ 12 × 16 – 58 ≈ ?
? = 3563 768×16
Sol.168 ? ≈ 12 − 58
64 12 ? ≈ 966
20 × √? = × 400 + × 1200 Sol.177
100 100
20 × √? = 256 + 144 34.02% of 550.09 ÷ ? = 297.07 ÷ √728.95
400
√? = 20 = 20 34 × 550
÷ ? ≈ 297 ÷ √729
? = 400 100
Sol.169 187 297

14 ? 27
(? )2 + × 1600 = 59 × 12 ? ≈ 17
100
(?)² + 224 = 708 Ans.178
(?)² = 484 Sol.
? = 22 (? ÷ 9.97) × 12.08 ≈20.12% of 1319.97
Sol.170 20 × 1320
150 (? ÷ 10) × 12 ≈
× 16 + 16 × 4 ≃ ? – 140 100
100 264
24 + 64 + 140 = ? ?≈ × 10 ≈ 220
? = 228 12
Sol.179
Sol.171
1350 120 ? % 𝑜𝑓 179.99 =
? + 3 – 125 ≃ 100 × 1650
? + 450 – 125 = 1980 √(24.02)2 + (17.98)2 + 60.01% 𝑜𝑓 659.98
? = 1655
? % 𝑜𝑓 180 ≈ √(24)2 + (18)2 + 60% 𝑜𝑓 660
Sol.172 ?
× 180 ≈ √576 + 324 + 396
(?)² + 900 + 260 ≃ 2385 100
?
(?)² = 1225 × 180 ≈ √1296
100
? = 35 36
? ≈ 180 × 100
Sol.173
? ? ≈ 20
× 300 ≃ (21)2 + (19)2 + (6)3 + 2.01
100
? × 3 = 441 + 361 + 216 + 2

For Free Study Material & Quizzes visit : www.mathsbyarunsir.com


For any queries or suggestions email us @ mathsbyarunsir003@gmail.com or whatsapp @ 8881331466
Sol.180 7
1296 + 219 × ? = 139 × 13
√(7)2 + (17)2 + (5)2 − 2 =? 7
219 × ? = 1807 − 1296
⇒ √361 = ? 7
⇒ ? = 19 219 × ? = 511
219
Sol.181 ?=
73
16 46 ?=3
× 350 + × 4450−? = 1783
100 100 Sol.188
⇒ 56 + 2047 – ? = 1783 ? 36 19
× 1225 + 100 × 2500 = 1156 + 100 × 200
100
⇒ ? = 320
?
Sol.182 × 1225 = 1156 + 38 − 900
100
8476 ÷ ? × 45 + 32 = 5900 ?
× 1225 = 294
100
8476 ? = 24
⇒ × 45 = 5868
? Sol.189
8476×45
⇒?= = 65 37
× ? + 121 – 174 + 15 = 961
5868
100
37
Sol.183 × ? = 961 + 38
100
? 15 37×? = 99900
× 540 − 78 = × 920 ? = 2700
100 100
? Sol.190
⇒ × 540 = 138 + 78 = 216 1 1
100 (1764)2 + (2197)3 = √(? )3 + 17 + 4 × 8
216 × 100
⇒ ?= = 40
540 42 + 13 = √(? )3 + 17 + 32

Sol.184 23 = √(? )3 + 17
529 = (?)³ + 17
(216)3 ÷ (36)2 = (6)? × 36 1
(216)³ (?) = (512)3
⇒ (6 )? = = 216 = (6)³ ?=8
(36)² × 36
⇒ ?= 3 Sol191
35 20
Sol.185 . 100 × 80 × 100 × 250 – 424 = ? + 52 × 8
1400 – 424 = ? + 416
62 ? = 560
×? + 576 = 961 − 13
100
62
×? = 948 − 576 Sol.192
100
62 1
×? = 372 45 × 65 × 312 + 361 = ?² + 11 × 32
100
? = 600 577 = ?² + 352
1

Sol.186 ? = (225)2
36 ? = 15
3? × 4 + 100 × 2100 = 1728
3? × 4 = 1728 − 756 ?
Sol193. 100 × (1131– 502–241–188) = 60
3? × 4 = 972 ?
× 200 = 60
3? = 243 100
60
?=5 ?= 2
? = 30
Sol.187

For Free Study Material & Quizzes visit : www.mathsbyarunsir.com


For any queries or suggestions email us @ mathsbyarunsir003@gmail.com or whatsapp @ 8881331466
Sol194 (5)? = 625
50 6450 36 × 41 (5)? = (5)4
. ? – 554– 100 × =
43 369 ?=4
? –554 – 75 = 4 Sol.202
? = 4 + 629 28 30
? = 633 × ? + 144 = 11 + 100 × 530 + 100
100
28
Sol.195 × ? = 11 + 159 + 100 – 144
1 296×32 37 100
⇒ 4× =1 28
? ×
1 × ? = 126
4 4 100
126 ×100
1 1 1 296
? = 28
⇒ ?= 4 × 4 × 4 × × 32 ? = 450
37
⇒? = 4 Sol.203
511 + (?)2 = 784 + 16
Sol.196 511 + (?)2 = 800
3
√405×315×245 (?)2 = 289
=?
√ ×27
1 ? = 17
3
3
√5×81×5×63×5×49
⇒ ?= 3
5×7×9 Sol.204
⇒ ?= 42 725
3
× 800 + = 361
⇒ ? = 105 100 ?
725
= 361 − 336
?
Sol.197 725
20 266 17 ? = 25
3
× 145 − 119 × 19 =? ? = 29
100
⇒ ?3 = 29 − 2
⇒ ?3 = 33 Ans.205
⇒ ?= 3 ? = (242 - 16)/20
Sol198 = (576 - 16)/20
1 = 560/20 = 28
1 1 182 474
. √81 × 9 × 9 × 4 + × 237 + 8 =?2
91
1 Ans.206
⇒ ? = 2+4+8
2
? ≈ 152 + 472 + 252
⇒ ? = 196 = 225 + 2209 + 625 = 3059
20 10
Sol.199 6739 + 161 × 5 − 75 × 12 + 100 × 120 − 100 ×
110 =? Ans.207
⇒ ? = 6739 + 805 − 900 + 24 − 11 ? ≈ √784 + √1444 - √289
⇒ ? = 6657 = 28 + 38 - 17
= 49
Sol.200 Ans.208
? 20 ? = 1927 ÷ 47 x 41 - 27
× 625 + 100 × 955 = 441 = 41 x 41 - 27
100
6.25 × ? = 441 − 191 = 1681 - 27 = 1654
250 Ans.209
? = 6.25
? = (31 x 27 - 27)/30
? = 40 = (837 - 27)/30
= 810/30 = 27
Sol.201 Sol. 210
170
(5)? + 225 = 100 × 500 11.02 × 8.98 +
1331.01
+ (30.95)2 =?
? √120.95
(5) = 850 −225

For Free Study Material & Quizzes visit : www.mathsbyarunsir.com


For any queries or suggestions email us @ mathsbyarunsir003@gmail.com or whatsapp @ 8881331466
1331
11 × 9 + + (31)2 ≈ ?
√121 Sol.220
99 + 121 + 961 ≈ ?
325 × 16 ÷ 4 + 37 ≈ ?% of 6700
? ≈ 1181 ?
Sol. 211 ⇒ 1337 = 100 × 6700
251.9 1337
√960.9 + +? % 𝑜𝑓 350.1 = (1.99)2 × 69.01 ⇒?= = 19.95 ≈ 20
√ 143.9 67
252
√961 + +? % 𝑜𝑓 350 ≈ 22 × 69 Sol.221
√144
31 + 21 + ? % 𝑜𝑓 350 ≈ 276 64% of 8900 + ?% of 5300 ≈6850
? % 𝑜𝑓 350 ≈ 276 – 31 – 21 ?
⇒ 5696 + 100 × 5300 = 6850
? ≈ 64
Sol. 212 ⇒ 53? = 1154
1154
77 ×
420 11
+ 500 − 100 × 600 = ?×
5600 ⇒ ? = 53 = 21.77 ≈ 22
70 100
462 + 434 = 56 × ? Sol.222
896
? = 56 √23409% of 800 – 624 ≈ √? × 24 + (12)2
? = 16 153
⇒ 100 × 800 – 624 = √?× 24 + 144
Sol.213
25 × ? = 10
50×25 ⇒ 1224 – 624 = √? × 24 + 144
?=5 ⇒ 600 – 144 = √? × 24
456
Ans.214 ⇒ √? = 24 = 19
? = 2950 ÷ 59 - 28
⇒ ? = (19)2 = 361
= 50 - 28
= 22
Sol.223
Sol.215
Given decimal can be rounded off as
143 = 2744, 223 = 10648, 123 = 1728 364 – 269 + 50 × 12 ÷ (20% of 80)
? ≈ ∛2744 - ∛10648 + ∛1728
⇒ 364 – 269 + 600 ÷ (16)
= 14 - 22 + 12 = 4
sol.216 ⇒ 364 – 269 + 37.5 ⇒ 132.5 ≈ 133
? ≈ 112 + 282 - 322 Sol.224
= 121 + 784 - 1024
It can be written as
= -119 {(19)² – (27)² + (1400 + 60)} ÷ ? = 39
sol.217
? = (12 x 19 - 3)/15 ⇒ {361 – 729 + (1460)} ÷ ? = 39
= (228 - 3)/15 ⇒ 1092 ÷ ? = 39
= 225/15 = 15 ⇒ ? = 1092 ÷ 39 ≈ 28

Sol.218 Sol.225
40% of 440 + ?% of 655 ≈ 230 It can be written as
? (22% of 900) + ? + (40% of 310) = 481
⇒ 176 + 100 × 655 = 230
54×100 ⇒ 198 + ? + 124 = 481
⇒ ? = 655 = 8.24% ≈ 8% ⇒ ? = 481 – 198 – 124 = 159
Sol.219
40% of 600 – 250 ≈ ? – 80% of 900
⇒ 240 – 250 = ? – 720 Sol.226
⇒ ? = – 10 + 720 = 710 It can be written as
10% of 650 + 32 × 2800 ÷ 400 - ? = 155

For Free Study Material & Quizzes visit : www.mathsbyarunsir.com


For any queries or suggestions email us @ mathsbyarunsir003@gmail.com or whatsapp @ 8881331466
⇒ 65 + 224 – ? = 155 (?)³ = 1000
⇒ ? = 134 ? = 10

Sol.227 Sol.234
1 1
?2 + 114 − 24 × 5 = 163 [(512)3 + (729)3 ] ≈ √?2 + 25 + 4
?2 = 163 − 114 + 24 × 5 = 163 + 6 8 + 9 – 4 = √?2 + 25
?2 = 169 ⇒ ? = 13
13 = √?2 + 25
Sol.228 ?² + 25 = 169
220 ? ?² = 144
× 380 + (36)2 − 100 × 400 = (24)2 ? = 12
100
836 + 1296– 4 ×? = 576
4 ×? = 2132– 576 Sol.235
1556 ? −5 40
?= 4 × × 650 ≈ 65
? = 389 15−? 100
? −5
× 260 = 65
Sol.229 15−?
56
? −5 65
(39)2 =? +(30)2 − (16)2 + × 1400 =
100 15−? 260
1521 = ? + 900 – 256 + 784 ? −5 1
? = 1521 – 1428 =
15−? 4
? = 93 4 × ? – 20 = 15 - ?
4 × ? + ? = 35
Sol.230 5 × ? = 35
(32)2 + 12 × 16– √2116 – (? )2 = (33)2 − 40 ?=7
1024 + 192 – 46 – (?)2 = 1089 – 40 Sol.236
20
1170 – 1049 = (? )2 100
× 625
(? )2 = 121 30 × 342 ≈ ?2 − 13 × 3
× 570
? = 11 100
125
× 342 =?2 − 39
Sol.231 171
480 10 250 + 39 = ?²
= (9)3 − 15 × 40 − 100 × 990 ?² = 289
?
480
= 729 − 600 − 99 ? = 17
?
480 Sol.237
= 30
?
? = 16 79.98% of ? = √249.98 × 44.04 + 40% of 8499

Sol.232 40 × 8500
80% 𝑜𝑓 ? ≃ √250 × 44 +
√430 + 520 + √121 = (? )2 + √36 100
√430 + 520 + 11 = (? )2 + 6 80
⇒ × ? ≃ √11000 + 3400
(? )? = 31 − 6 100
(? )2 = 25 10
⇒? ≃ √14400 ×
?=5 8
10
Sol.233 ⇒ ? ≃ 120 × 8 = 150
17 × 19 + (26)² ≈ (?)³ - 1
(18 - 1) × (18 + 1) + 676 = (?)³ - 1 Sol.238
(18)² - 1 + 676 = (?)³ - 1 593.93 2299
324 + 676 = (?)³ ? × 40.001 ÷ 24.02 × 27.05 = ×
114.94 263.98

For Free Study Material & Quizzes visit : www.mathsbyarunsir.com


For any queries or suggestions email us @ mathsbyarunsir003@gmail.com or whatsapp @ 8881331466
40 594 2300 ? ≈ 280 + 5 + 9
⇒ ?× × 27 ≃ ×
24 115 264 ? ≈ 294
⇒ ? × 45 ≃ 45 ⇒ ? = 1
Sol.239 Sol.245
19.99% 𝑜𝑓 (40.01 × √? ) = (31.99)2 + (16.01)² ?
= (19.99)2 − (3.99)2 × (5.01)2 +
20 √120.98–(2.98) 2
⇒ × 40 × √? ≃ 322 + 16² 113.01
100 ?
1 ≈ (20)2 − (4)2 × (5)2
√ 121–(3)2
⇒ √? ≃ × (1024 + 256)
8 +113
1 ?
⇒ √? ≃ × 1280 ≃ 160 ≈ 400
8 11 − 9
⇒ ? = (160)² = 25600 – 400 + 113
Sol.240
44.98
? ≈ 113 × 2
? + 13.02 × 49.98 = 420.05 +
× 799 + 220.10 ? ≈ 226
100
45 Sol.246
⇒ ? + 13 × 50 ≃ 420 + × 800 + 220 3 ?
100 38 × 15 + √216 = 100 × 400 + (18)2
⇒ ? + 650 ≃ 420 + 360 + 220
570 + 6 = 4 × ? + 324
⇒ ? = 1000 − 650 = 350 576 – 324 = 4 × ?
Sol.241 252
1331.01 ?= 4
11.02 × 8.98 + + (30.95)2 =? ? = 63
√120.95
1331
11 × 9 + ( )2
+ 31 ≈ ? Sol247
√121 573+? 38×450
99 + 121 + 961 ≈ ? . + − (11)2 = (10)2
16 100
573+?
? ≈ 1181 + 171– 121 = 100
16
573 + ? = 50 × 16
Sol. 242 ? = 800 – 573
350.98 (3.02)2 ? = 227
√88.01 × 21.99 + +
12.98 4−2
1 Sol.248
=? +(243.01)5 48 25
351 2 2
1
×? +√2304 = 18 × 15 + 100 × 360
√88 × 22 + + 3 × 4 = ? + (243) 5 100
13 48×?
? ≈ 44 + 27 + (9 × 16) – 3 + 48 = 270 + 90
100
48× ?
? ≈ 44 + 27 + 144 – 3 = 212 = 312
100
312×100
Sol. 243. (c) ? = 48
50 61 40 ? = 650
1260  100 ?   450
100 100 100
Sol.249
630  671 ?180 3 3
? ~1301 -180 √6859 × √4096
= ( ? ) 2 + √4
√64
~ 1121 19×16
= (? )2 +2
8
Sol244 ( )2
38 – 2 = ?
((13.98)2 –19×3.97) 1 (? )2 = 36
. 112.01% 𝑜𝑓 249.98 + + 22 ×6−2 ≈?
23 ×3 ?=6
((14)2 −19×4)
112 % of 250 + +9≈ ? Sol.250
24
(196–76)
? ≈ 280 + +9
24

For Free Study Material & Quizzes visit : www.mathsbyarunsir.com


For any queries or suggestions email us @ mathsbyarunsir003@gmail.com or whatsapp @ 8881331466
14
56 − 34 × 17 + 2 80 22 × (3)? × (6)2 = (12)2 × 9
=
12 − 51 × 17
3
? 22 × (3)? × 36 = 144 × 9
56 − 28 + 2 80 144 × 9
= = (3 )? =
12 − 9 ? 4 × 36
?= 8 (3 )? = (3)2
Sol.251 ?=2
685 – 181 = (42% - 28%)of ? Ans.259
504 = 14% of 12.5 37.5
504×100 × 512 + × 96 + 3 × 7 = (? )2
? = 14 100 100
1 3
? = 3600 ⇒ × 512 + × 96 + 21 = (? )2
Sol.252 8 8
⇒ 64 + 36 + 21 = (? )2
103 × 721 [35% × 80% × 40% − 35% × 20% ⇒ (? )2 = 121
× 30%] = ? ⇒ ? = 11
112000 21000 Sol.260
= 721000 [ − ] =?
1000000 1000000 1
721 × 91 = ? (64)4×12 × 16 + 32 × 8 =?× 8
1
?= 65611 (64)3 × 16 + 256 =?× 8
Sol.253 4 × 16 + 256 = ?× 8
3 124 64 + 256 = ? × 8
≈ 4 × 8 + 100 × 650 =? 1
? = 320 × 8
≈ 6 + 806 =?
≈? = 812 ? = 40
Sol.261
Sol.254 1
(3375)3 × √21 + 15 × 11 = (? )2 + 29
75
87 + 914 – 338 ≈ × (? ) 15 × √36 × 11 = (? )2 + 29
100 (? )2 = 15 × 6 × 11 − 29
663 × 100
=? (? )2 = 990 − 29
75
? = 884 (? )2 = 961
Sol.255 (? )= 31
75 750 × 70
? % of 1050 + × 420 = Sol.262
100 100 1
?% of 1050 = 525 – 315 (4)2 × (144)2 × (? )2 (3)3 × 8
210 =
?= × 100 ≈ 20 16 × 8 ( 2)2
1050 2
16 × 12 × (? )
⇒ = 27 × 2
Sol.256 16 × 8
27 × 2 × 8
25 × 32 ⇒ (? )2 =
√324√20 × 50 × 8 × 20 + ≈ ? 12
100
⇒ (?)² = 36
√324 × 20 × 20 + 8 ≈?
⇒?=6
360 + 8 ≈ ?
? = 368
Sol.263
Sol.257
360 × 288 (? )2 √48.97 × 13.05
= 23.99 × 26.003 + = 4.97 ×? ³
15 × 18 6 90.98
(?)² = 2304 √49 × 13
24 × 26 + = 5 ×?3
(?) ≈ 48 91
Sol.258 624 + 1 = 5 × ?³
?=5

For Free Study Material & Quizzes visit : www.mathsbyarunsir.com


For any queries or suggestions email us @ mathsbyarunsir003@gmail.com or whatsapp @ 8881331466
√? + 289 –12 ≈ 300
Sol.264
61 √? ≈ 300 – 277
109.07√?− ×? = 47.96√? √? ≈ 23
21.02
61 ? ≈ 529
109√?− 48√? ≈ ×?
21 Sol.272
61 3 3
[ √729 +√1296 + √3375 ]
61√? = ×? ≈ √225
21 ?
? = 441 [9+36+15]
≈ 15
?
60
Sol.265 ? ≈ 15
1332.89 + 171.928 + 17.01 + ?² = 1690.67 ?≈4
1333 + 172 + 17 – 1691 ≈ -?²
?² = 169 Sol.273
? = 13 1 62.5 2

Sol.266 (1331)3 × 12 + × 400 − ? ≈ (343)6 × 49


100
322.9 5 1
150.09% 𝑜𝑓 20 + + √? = (8.96)2 ⇒ 11 × 12 + × 400−? = (343)3 × 49
17.02 8
30 + 19 + √? = 81 ⇒ 132 + 250 – ? = 7 × 49
? = 1024 ⇒ ? = 382 – 343
⇒ ? = 39
Sol.267
1
56.08% of 149.92 + √28.02 × 6.98 − 11 9 % 998.9 = ? Sol.274
1 37.5
56% of 150 + √28 × 7 − 9 × 999 ≈? 60% 𝑜𝑓 1540 + × 96 + 1 ≈ (? )2
84 + 14 – 111 = -13 100
3
Sol.268 ⇒ 924 + × 96 + 1 = (? )2
272+? 125 8
425×8− ≈ 100 × 2400 ⇒ (? )2 = 924 + 36 + 1
8
272+? ⇒ (?)² = 961
≈ 3400 − 3000
8
⇒ ? = 31
272+? ≈ 3200
? ≈ 2928 Sol.275
Sol.269
67 + 3 + ? = 52 + 64
30 40
579 +331 + 100 ×? ≈ 100 × 2800 ? = 46
3×? ≈ (1120–910) × 1 0 Sol.276
75
3 × ? ≈ 2100 16√?+ 69√?− 10√? ≈ × (? )
34
? ≈ 700 75
75√? = × (? )
Sol.270 34
?
727+?
+ 12– 60 ≈ √1024 ⇒ √? =
15 34
727+?
≈ 80 ⇒ √? = 34
15
⇒ ? = (34)² ⇒ ? = 1156
? ≈ 1200–727 Sol.277
? ≈ 473 9 22
(25)6 + × 450 ≈ ? +(3 × 2)3
100
Sol.271 3
25 (25)2 + 99 =? +(6)3
√? + (17)² – 100×48 ≈ 50×6 125 + 99 = ? + 216

For Free Study Material & Quizzes visit : www.mathsbyarunsir.com


For any queries or suggestions email us @ mathsbyarunsir003@gmail.com or whatsapp @ 8881331466
? = 224 – 216 753+?
?=8 = 961 – 911
18
? = 900 – 753
Sol.278 ? = 147
? 19 30 Sol.285
× 225 + 100 × 700 = 256 − 100 × 290
100 ?
?
× 225 = 169 − 133 (377 + 539 – 40) + × 800 = (30)2
100 100
? 876 + 8? = 900
× 225 = 36 (900– 876)
100
3600 ?=
? = 225 8
? = 16 ?=3
Sol.279
60 Sol.286
?3 + 324 = 100 × 900 37
3 × 6600 + 169 – ? = (50)2
? = 540 – 324 100
?3 = 216 2442 + 169 – ? = 2500
?3 = (6)3 ? = 2611 – 2500
?=6 ? = 111
Sol.280
87 ×8 8 Sol.287
+ 100 × 1150 = 324 ? 15
?
87 ×8 18 × 25 – + × 300 = (22)2
= 324 − 92 15 100
?
696
?
?= 450 – + 45 = 484
232 15
?=3 ?
Sol.281 = 495 – 484
15
2? ? = 165
× 500 + 36 = 196
100
2? Sol.288
× 500 = 160
100 64.02 1
?
2 = 32 43.495 × 31.99 × 28.979 − 2.012 = ?
2? = (2)5 64 1
43.5 × 32 × 29 − 2 ≈ ?
?=5
Sol.282 ? ≈1
40 8
?2 + 439 − 100 × 900 = 100 × 2500 Sol 289.Ans. (b)
?2 + 79 = 200 2 50
?2 = 121  249  4  6 ?   300
3 100
?2 = (11)2
? = 11 166  24 ? 150
? 40
Sol.283 ? 1600
14 3
× (√729 × √121) + (15)2 = ? Sol.290
99
14 + 225 =? (33.33×80.989÷99.99)+3.024-?=4.012
? = 239 33.33
( 99.99 × 81) + 3 − ? ≈ 4
Sol.284
753+? 21 ? = 26
+ × 4200 + √841 = (31)2
18 100 Sol.291
753+?
+ 882 + 29 = 961
18

For Free Study Material & Quizzes visit : www.mathsbyarunsir.com


For any queries or suggestions email us @ mathsbyarunsir003@gmail.com or whatsapp @ 8881331466
20.021+4.969+30.499-50.022=? Sol.299
20+5+30.5-50≈? 120 ?
× 1250 + × 625 ≃ 3375
?≈5.5 100 100
?
× 625 = 1875
100
? = 300
Sol.292
Sol.300
995.013-39.976×19.99+5.022=1.988×?
995-40×20+5=2×? 20 × 33 + √64 × 10 ≃ 20 × √?
?≈100 660 + 80 ≃ 20 × √?
740
Sol.293 √? ≃ = 37
20
(10.011)2 + (23.989)2 = 275.99 + (? )2 ? = 1369
102 + 242 = 276 + ?2 Sol.301
? = 20 2900
+ 20 + (? )2 ≃ 94
Sol.294 290
1 4 1 1 2 1 1 (? )2 = 94 – 30
5 + 3 − 7 − 2 = ? − 16 + 11 − 5
3 9 2 4 9 4 2 ? = √64 = 8
1 4 1 1 Sol.302
? = (5 + 3 − 7 − 2) + + − −
3 9 2 4 30 82
2 1 1 × 700 – 350 + × 500 ≃?
+ (16 − 11 + 5) + − + 100 100
9 4 2 210 – 350 + 410 ≃ ?
1 1
=9+ =9 ? = 620 – 350
2 2 ? = 270
Sol.303
Sol.295
1 1520 + 180 × 20 + ? ≃ (2)⁹ × 10
√1296 + (2744)3 = ? − √961 1520 + 3600 + ? ≃ 5120
?² = 36 + 14 + 31 = 81
?=0
?=9
Sol.304
Sol.296 4.01 1
33.989 × 17.02 × 3.99 − 2.012 = ?
63% of 450 + 81% of 150 = ? % of 675 4 1
283.5 + 121.5 = ?% × 675 34 × 17 × 4 − 2 ≈ ?
405 ? ≈0
× 100 = ?
675 Sol.305
⇒ ? = 60 (11.01 + 12.97) ×
1
+ 5.956 = 18 − ?
7.99
1
Sol.297 (11 + 13) × + 6 ≈ 18 − ?
8
205 × ? × 13 = 33625 + 25005 ?= 9
58630 Sol.306
?= = 22
205 × 13 119.022 + 40.99 + 9.03 = ?2
119 + 41 + 9 ≈?2
Sol.298 ? = 13
3 4 9
× × × 4312 =?× 7
8 7 11 Sol.307
1 3 1 9
⇒ ? = 4312 × × × × 58.99 + 52.11 − 47.94 + ? = 85.96
7 2 7 11 59 + 52 − 48 + ? ≈ 86
? = 108
? = 86 − 63 ≈ 23

For Free Study Material & Quizzes visit : www.mathsbyarunsir.com


For any queries or suggestions email us @ mathsbyarunsir003@gmail.com or whatsapp @ 8881331466
Sol.308
(14.96)2 + (5.011)3 + 50.02 = ?3
152 + 53 + 50 = ?2
225 + 125 + 50 = ?2
?2 ≈ 400
? = 20
Sol.309 Sol.316

Sol.310

Sol.317

Sol.318
Ans.311

Sol.319
Ans.312 Given expression is
? % of 299.99 + 54.95% of 239.94 = 18.99% of 899.99
⇒ ? % of 300 + 55% of 240 =19% of 900
⇒ 3 × ? + 24 × 5.5 = 171
⇒ 3 × ? + 132 = 171
Ans.313 ⇒ 3 × ? = 39
⇒ ? = 13
Sol.320
818.88 ÷ 62.93 + 18.9 × 21.9 = 34.95 × 19.92 - ?
⇒ 819 ÷ 63 + 19 × 22 = 35 × 20 - ?
⇒ 13 + 19 × 22 = 35 × 20 - ?
⇒ 13 + 418 = 35 × 20 - ?
⇒ 13 + 418 = 700 - ?
⇒ 431 = 700 - ?
Ans.314 ⇒ ? = 700 - 431
⇒ ? = 269

Ans.321
Given expression is
Ans.315 2.99 × (9.99 + 4.95) + (48.04/5.99)/3.94 – 6.97 = ?

For Free Study Material & Quizzes visit : www.mathsbyarunsir.com


For any queries or suggestions email us @ mathsbyarunsir003@gmail.com or whatsapp @ 8881331466
3 × (10 + 5) + (48/6) / 4-7 ⇒ ?2 = 100
= 3 × (15) + (48/6) / 4-7
⇒ ? ≈ 10
= 45 + (8) / 4 - 7 Sol.327
= 45 + 2 - 7
(63.95 + ?) ÷ 25.02 + 390.09 + 19.97% of 724.87 = 540.07
= 47 - 7 = 40
Ans.322 ⇒ (64 + ?) ÷ 25 + 390 + 20% of 725 = 540
Given expression is ⇒ (64 + ?) ÷ 25 + 390 + 145 = 540
5.94 + 1/12.93 of {2599.96 ÷ (117.98 – 107.98)} = ? ⇒ (64 + ?) ÷ 25 = 540 - 390 - 145
6 + 1/13 of {2600 ÷ (118 – 108)} = ? ⇒ (64 + ?) ÷ 25 = 5
⇒ 6 + 1/13 of {2600 ÷ 10} = ? ⇒ (64 + ?) = 125
⇒ 6 + 1/13 of 260 = ? ⇒ ? = 61
⇒ 6 + 20 = ?
∴ ? = 26 Sol.328
14.99% of 115.95 × 18.892% of 179.77 = (?) – 372
Ans.323 Approximating the values to the nearest integer:
Given expression is ⇒ 15% of 116 × 19% of 180 = (?) – 372
[(22.9 × 1.922 × 23.92 )] ÷ (2.02 × √1295.96) = (2.93)? + ⇒ (?) = 17.4 × 34.2 + 1369
6.97 ⇒ (?) = 595.08 + 1369
[(23 × 22 × 242)] ÷ (2 × √1296) = (3)? + 7 ⇒ (?) = 1964
⇒ (23 × 4 × 576) / (2×36) = 7 + (3)? [∵ √1296 = 36]
⇒ (23 × 32) = 7 + (3)? Sol.329
22
⇒ 736 = 7 + (3)? 41.979 ×
7
+ 19.989% of 530.14 −26.021 = ?
22
⇒ 729 = (3)? 42 × + 20% of 530 – 26 ≈ ?
7
⇒ (3 )6 = (3)? ? ≈ 132 + 106 − 26 ≈ 212
∴?=6 Sol.330
Ans.324 (23.012 × 22.989) + 20.985 × 7.014 = ?2
52.034 ≈ 52, 617.121 ≈ 617 and 32.004 ≈ 32 (23 × 23) + 21 × 7 ≈ ?2
576.71 ≈ 577 and 68.284 ≈ 68 ?2 ≈ 529 + 147 ≈ 676
Then, ? = 26
⇒ 52 + 617 + 32 = ? + 577 + 68
Sol.331
⇒ 701 = ? + 645
⇒ ? ≈ 56 √1443.979 ÷ 18.981 + 3.2 × √16.017 = (? )
Sol.325
50 160 2 √1444 ÷ 19 + 3.2 × √16 ≈ ?
⇒ (100 × 51800) ÷? = (100 × 10.01) × 2.01 38
? = 19 + 3.5 × 4
⇒ 25900 ÷ ? = 256 ×2 ? ≈ 2 + 14 ≈ 16
⇒ 25900 ÷? = 512
⇒? = 50.6 Sol.332
⇒? ≈ 50 779.98÷ 48.014 × 15.989 = ?
Sol.326 780
× 16 = ?
Given expression becomes, 48
780
⇒ 24 + 24 – 12 = 34 × 4 - ?2 ?= ≈ 260
3
⇒ 24 + 24 – 12 = 136 - ?2
Sol.333
⇒ 48 – 12 = 136 - ?2
1485.988 + 212.04 − 1703.99 = ? −(11.02)2
⇒ 36 = 136 - ?2
1486 + 212 − 1704 ≈? − (11)2

For Free Study Material & Quizzes visit : www.mathsbyarunsir.com


For any queries or suggestions email us @ mathsbyarunsir003@gmail.com or whatsapp @ 8881331466
? ≈ 1698 − 1704 + 121 ≈ 115 1750 – 1024 + 3 = ?
Sol.334 ? = 729
737 × 3 + 363 × 2 + 11 × 33 = ? × 11 Hence, option c.
Dividing by 11
67 × 3 + 33 × 2 + 33 = ? Sol.341
3 × (67 + 22 + 11) = ? 20% of 5000 – 25% of 3000 = ? × 52
? = 3 × 100 0.20 × 5000 – 0.25 × 3000 = ? × 25
? = 300 1000 – 750 = ? × 25
Hence, option b. ? = 250/25 = 10
Hence, option a.
Sol.335
? × 32 – 25% of (480 × 128) = 0
? × 32 – 0.25 × 480 × 128 = 0 Sol.342
? × 32 – 120 × 128 = 0 16 × 15 – 240 – 32 × 10 + 420 = ?2
? = 480 240 – 240 – 320 + 420 = ?2
Hence, option d. -320 + 420 = ?2
? = 10
Sol.336 Hence, option a.
93 × (8910 ÷ 110) = (14580 ÷ 20) × ? Sol.343
729 × 81 = 729 × ? 65% of 200 – 26% of 500 = ?
? = 81 0.65 × 200 – 0.26 × 500 = ?
Hence, option c. ? = 130 – 130
?=0
Sol.337 Hence, option e.
262 + 113 - √49 = 2 × ?3
676 + 1331 – 7 = 2 × ?3
Sol.344
2000 = 2 × ?3
45% of 1500 – 2 × 52= ?2
? = 10
0.45 × 1500 – 2 × 25 = ?2
Hence, option b.
675 – 50= ?2
Sol.338 ? = 25
Hence, option a.
25 1 3 800 Sol.345
?~  320  168  
100 3 8 3 (32 × 25) – 202 = 20% of ?
~ 80  56 100 (800 – 400) × 5 = ?
~ 36 ? = 2000
?~6 Hence, option c.

Sol.339 Sol.346
65% of 1200 – 15% of 400 = ?3 × 45% of 200 22% of 5000 + 14% of 400 = ?2
0.65 × 1200 – 0.15 × 400 = ?3 × 0.45 × 200 0.22 × 5000 + 0.14 × 400 = ?2
(780 – 60)/90 = ?3 1100 + 56 = ?2
?3 = 8 ? = 34
?=2 Hence, option e.
Hence, option d.
Sol.340 Sol.347
35% of 5000 – 322 + 3 = ?
0.35 × 5000 – 1024 + 3 = ? 73 – 132 – ? = 25% of 120
343 – 169 – 0.25 × 120= ?

For Free Study Material & Quizzes visit : www.mathsbyarunsir.com


For any queries or suggestions email us @ mathsbyarunsir003@gmail.com or whatsapp @ 8881331466
174 – 30 = ? 566/6 ~ ?
? = 144 ? ~ 94.3 ~ 90
Hence, option c.
Sol.348 S354. Ans.(b)
80% of 2500 - ?% of 2500 = 103 Sol. ? ≃ 21 + 5 × 8 + 10
(0.80 × 2500 – 1000) ÷ 25 = ? ≃ 71
? = (2000 – 1000) ÷ 25
? = 40 S355. Ans.(c)
39
Sol. ? ≃ 1.38 × 3780 + 100 × 140
Sol.349
41.97 × 5.12 ÷ 2.99 + 49.89 = ? × 1.99 ≃ 5,271
42 × 5 ÷ 3 + 50 ~ ?× 2
70 + 50 ~ ?× 2 S356. Ans.(b)
340×10
120/2 ~ ? Sol. ? ≃ 6.4 + 1245
? ~ 60 ≃ 1776.25
Hence, option e. ≃ 1776
Sol.350 S357. Ans.(e)
18.003 × 3.99 × (135.003 ÷ 26.97) – (15.001 × 3.94) = ? 2009
18 × 4 × (135 ÷ 27) – (15 × 4) ~ ? Sol. 322 − 122 + ≃ 900
?
2009
72 × 5 – 60 ~ ? ⇒ ? ≃ 20
? ~ 360 – 60 ⇒ ? ≃ 100
? ~ 300
Hence, option a. S358. Ans.(b)
1575
Sol.351 Sol. ? ≃ 45 + 24 × 16
≃ 419
30.01 × √1023 + 196 = ?2 – 287
≃ 420
30 × √1024 + 196 ~ ?2 - 287 S359. Ans.(a)
30 × 32 + 196 ~ ?2 - 287 Sol.
960 + 196 + 287 ~ ?2 576 1 400 900
?2 ~ 1443 ?= × × × = 1620
16 5 40 40
?2 ~ 1444
? ~ 38 S360. Ans.(c)
Hence, option b. Sol.
? = 68 × 14 − 14 × 13 = 770
Sol.352
35.1% of 1599 = ?–(449.96 ÷ 6.12) × 2 S361. Ans.(d)
35% of 1600 ~ ?–(450 ÷ 6)× 2 Sol.
560 ~ ? - 75 × 2 ? = 5467 − 3245 + 1123 − 2310 = 1035
560 ~ ? - 150
560 + 150 ~ ? S362. Ans.(c)
? ~ 710 ~ 700 Sol.
Hence, option d. ? = 40 × 6 − 250 + 700 = 690
Sol.353 S363. Ans.(b)
16.12 × 15.94 + 654.92 – 344.83 = ? × 5.95 54000×30
Sol. = = 675
16 × 16 + 655 – 345 ~ ?× 6 60×40
256 + 655 – 345 ~ ?× 6
911 – 345 ~ ?× 6

For Free Study Material & Quizzes visit : www.mathsbyarunsir.com


For any queries or suggestions email us @ mathsbyarunsir003@gmail.com or whatsapp @ 8881331466
S364. Ans.(b) 2500
450 ⇒ ?=
Sol. ? ≃ 6 + 34 – 75 50
⇒ ? = 50
≃ 75 + 81 – 75
≃ 81 S374. Ans.(c)
S365. Ans.(c) Sol.
861 1
Sol. ? ≃ 12 × – 58
84 36.013 × 40962 × 37.992 ÷ (93 × 75.982 ) = 4?
? ≃ 65 363 × √4096 × 382
S366. Ans.(b) or, 4? ≈
70 30 93 × 762
Sol. ? ≃ 100 × 260 – 100 × 530 4 × 9 × 43 × 38 × 38 43 × 43
3 3
≈ ≈
≃ 182 – 159 93 × 76 × 76 2×2
≃ 23 or, 4? ≈ 43 × 42 = 45
S367. Ans.(c) ∴ ?≈ 5
4
Sol. ? ≃ 11 × 1331 ÷ 22 + 6 S375. Ans.(c)
? ≃ 28 Sol.
S368. Ans.(e) 18% of 256 + 35% of 290 – 15% of 385 = ?
24 5 35
Sol. ? ≃ 350 × 4 × 20 + 500 18 35 15
or, ? ≈ × 260 + × 300 − × 400
100 100 100
≃ 500.15 = 46.8 + 105-60 = 151.8 – 60 = 91.8 ≈ 90
≃ 500
S376. Ans.(a)
S369. Ans.(c) 69 33
Sol. Sol. ? ≃ 100 𝑜𝑓 700 + 100 𝑜𝑓 400
= 483 + 132 ≃ 615 (𝑎𝑝𝑝𝑟𝑜𝑥. )
? ≃ 13 × 22 × 18
≃ 5148 S377. Ans.(e)
Sol.
S370. Ans.(a) 30 43
Sol. ⇒ × 780 + × 800 = 528+ ?
100 100
18 28 ⇒ 234 + 344 = 528+ ?
?≃ × 600 + × 450
100 100 ? = 578 − 528 = 50
≃ 234
S378. Ans.(a)
S371. Ans.(b)
Sol. ≈ 280 +500
Sol.
480 × 26 ≈ 780
? ≃ 32 + 625 – S379. Ans.(c)
39 Sol.
≃ 337
1 1 1 (? )3
(961)2 × (2744)3 + 13 × (36)2 = + (16)2
S372. Ans.(d) 2
Sol. (? )3
31 × 14 + 13 × 6 = + 256
3194 75 2
?≃ + × 800 (?)3
8 100 434 + 78 – 256 = 2
≃ 1000 (? )3 = 512
?=8
S373. Ans.(c)
Sol. S380. Ans.(e)
(28) ² – (22) ² + 2500 ÷ ? = 350 Sol.

For Free Study Material & Quizzes visit : www.mathsbyarunsir.com


For any queries or suggestions email us @ mathsbyarunsir003@gmail.com or whatsapp @ 8881331466
713 + 87 + 26 × 40 = 23 ×? ⇒ 61 = 36 +?
800 + 1040 = 23 ×? ∴ ? = 25
1840
? = 23
? = 80
S388. Ans.(e)
S381. Ans.(a) Sol.
1
Sol. 158 × 4 + 5 × 850+? = 951
12 × 11 (? )2 − (3)2 × 53 ⇒632 + 170 +? = 951
=
3 11 ∴ ? ≈ 150
4 × 121 = (?)² – 9 × 53
(?)² = 484 + 477 S389. Ans.(d)
(?)² = 961 Sol.
? = 31
1884 ÷ 144.89 + 6.99 + (? )2 = 69.09
S382. Ans.(c) ≈ 13 + 7 + (? )2 = 69.09
Sol. ≈?2 = 69 − 20 = 49
1 ∴ ? = √49 = 7
(? +13)(? −13) = 15 × (512)3
(? )2 − (13)2 = 15 × 8 S390. Ans.(c)
(?)² – 169 = 120 Sol.
(?)² = 289 √12000 × 34.98 + 150.04 =?
? = 17
≈ 110 × 35 + 150
= 4000
S383. Ans.(a)
Sol.
S391. Ans.(a)
512 × 256 2 Sol. 0.2% of 356 × 0.8% of 779
( ) = (64)?
8 × 32 = 0.712 × 6.232
(512)2 = (8)2×? = 0.7 × 6
(8)? = 512 ≈ 4.2 ≃ 4
(8)? = (8)3
? =3 S392. Ans.(b)
S384. Ans.(a) Sol. ≈ 63.9% of 8920 +? % of 5320 = 6830
Sol.
1
≈ 5320 ×? = (6830 − 5709) × 100
× 2844 + 15 × 2400 ? ≈ 21
4
= 711 + 36000 ≈ 36710
S393. Ans.(c)
S385. Ans.(b) 5 7
Sol. ≈ 8 of 4012 + 10 of 3410
Sol.
1080 ≈ 2507 + 2387
+ 187 × 20 ≈ 30 + 3740 ≈ 3770 = 4894 ≈ 4890
36

S386. Ans.(e) S394. Ans.(a)


Sol. Sol.
5 ? = 5145+3450-4560
× 4875 + 88 × 15 = 5 × 1219 + 1320
4 ? = 4035
= 6095 + 1320 = 7415

S387. Ans.(e) S395. Ans.(b)


2
Sol. Sol. 50% 𝑜𝑓 ? − 3 𝑜𝑓 243 = 50
39×15−28×10
= 36+?
5

For Free Study Material & Quizzes visit : www.mathsbyarunsir.com


For any queries or suggestions email us @ mathsbyarunsir003@gmail.com or whatsapp @ 8881331466
50 510 + 490
?× = 50 + 162 ?=
100 100
⇒ ? = 424 ? = 10

S396. Ans.(e) S404. Ans.(e)


Sol. Sol. √? = (1248.28 + 51.7) ÷ 99.9 − 7.98
? = 1+2+3-6
?=6–6 √? = (1300 ÷ 100) − 8
=0 √? = 5
? = 25
S397. Ans.(d)
Sol. S405. Ans.(b)
45 35 Sol. 129 − 224 = −95
?= × 600 + × 300
100 100
? = 270+105 S406. Ans.(a)
= 375 Sol.
69 33
× 700 + 100 × 400
S398. Ans.(e) 100
= 483 + 132 = 615
Sol. √441 ÷ 3 + √? = 47
21 S407. Ans.(c)
√? = 47 − = 40
3 Sol.
⇒ ? = 1600 180 1
× 25,500 + 2 (29,000)
100
S399. Ans.(b) = 45,900 + 14,500 = 60,400
Sol. ? = 134+1334+14
? = 1482 S408. Ans.(d)
Sol.
340 30 180
S400. Ans.(a) ÷ 510 × = 17 × 17 × 3 = 867 ≈ 870
20 60
Sol.
2 3
?= × 35 + × 490
5 7 S409. Ans.(d)
= 224 Sol.
S401. Ans.(b) ? ≃ (14725 + 1000) ÷ (500 + 930 + 660)
Sol. √121 × 100 × 324 =? ≃ 7.5
? = 11 × 10 × 18 S410. Ans.(a)
= 1980 Sol.
S402. Ans.(c) ? ≃ 182 × 22 × 34
Sol. ≃ 136136
(0.6 − 0.4)(0.62 + 0.6 × 0.4 + 0.42 )
?= × 10 S411. Ans.(b)
(0.62 + 0.6 × 0.4 + 0.42 )
? = 0.2 × 10 Sol.
?=2 ? × 53.25 ≃ 6828 – 5712
1116
S403. Ans.(d) ?≃
53
Sol. ≃ 21
?
. of 300 – 490 = 510
3
S412. Ans.(c)
Sol.

For Free Study Material & Quizzes visit : www.mathsbyarunsir.com


For any queries or suggestions email us @ mathsbyarunsir003@gmail.com or whatsapp @ 8881331466
40 75
?≃ × 600 + 250 + × 912 S420. Ans.(e)
100 100
≃ 1174 Sol.
2418.065 + 88 ÷ 14.2 × 6
S413. Ans.(b) ≈ 2418 + 6 × 6
Sol. = 2454 ≈ 2455
20 10
?≃ × 600 + × 900 S421. Ans.(d)
100 100
≃ 120 + 90 Sol.
≃ 210 564.866 + 82.1091 × 44.881 − 34.111
≈ 565 + 3690 − 34
S414. Ans.(b) ≃ 4220
Sol.
S422. Ans.(c)
√289.19 × 2.08 ÷ 10.97 Sol.
17×2 34
≈ 11 = 11 ≈ 3 3
√64001 + (46.85)2
≈ 40 + 472
S415. Ans.(d) = 2209 + 40
Sol. ≈ 2250
1884 ÷ 144.89 + 6.99 + (? )2 = 69.09
⇒ 13 + 7 + (? )2 ≃ 69.09 S423. Ans.(e)
⇒ ?2 ≃ 69 − 20 ≃ 49 Sol.
5
∴ ? = √49 = 7 × 4875 + 88 × 15 ≃ 5 × 1219 + 1320
4
≃ 6095 + 1320 ≃ 7415
S416. Ans.(c)
Sol. S424. Ans.(b)
√12102 × 34.98 + 150.04 =? Sol.
≈ 110 × 35 + 150 ≈ 63.5% of 8920 + ? % of 5320 = 6830
= 4000 ≈ 5320 ×? = (6830 − 5664) × 100
? ≈ 22
S417. Ans.(b)
Sol. S425. Ans.(c)
1010
+ 187 × 20 ≈ 28 + 3740 ≃ 3768 ≈ 3770 Sol.
36
5 7
≈ 8 of 4012 + 10 of 3410
S418. Ans.(c)
≈ 2507 + 2387
Sol.
= 4894 ≈ 4890
(3.2)2 + (9.8)2 + (8.13)2 + (4.24)2
≈ 9 + 100 + 64 + 16 S426. Ans.(a)
≈ 190 Sol.
S419. Ans.(b) 8787 ÷ 343 × √50
Sol. ≈ 25.6 × √49
3 14 2 = 179.2 ≈ 180
[( × ) ÷ (2.5 − 0.8)]
8 2
21 2 S427. Ans.(d)
=[ ÷ 1.7] Sol.
8
2.625 2
=( ) ≈ (1.5)2 √1296 × (303 ÷ 8) = (? )2
1.7
⇒ 37 × 37 ≃?2
≈ 2.4
⇒ ? ≃ 37

For Free Study Material & Quizzes visit : www.mathsbyarunsir.com


For any queries or suggestions email us @ mathsbyarunsir003@gmail.com or whatsapp @ 8881331466
? = 124.5
S428. Ans.(b)
Sol. S436. Ans.(a)
5940 ÷ 28 ÷ 6 Sol.
= 35.35 ≈ 35 ? ≃ 193 + 300 + 4000 ÷ 8 – 500
S429. Ans.(c) ≃ 493
Sol.
1921
172 + 11 × 28 – 289 =? S437. Ans.(e)

289 + 308 – 113 =? Sol.
? 24 44
? = 484 × 10 ≃ 100 × 50 + 100 × 150
100

S430. Ans.(a) ? ≃ (12 + 66) × 10


Sol. =780
?–√121 19×21
= 32 × 22 + S438. Ans.(d)
32 7
⇒ ? – 11 = (36 + 57) × 9 Sol.
? = 848 ? ≃ 11 + 42 + 32 – 28
≃ 57
S431. Ans.(b) S439. Ans.(b)
Sol. Sol.
808 57 × 36
225% of 44 + – 102 =?2 ?≃
8
⇒ ? ² = 99 + 101 – 100 19
? ≃ 108
? ² = 100
? = 10 S440. Ans.(c)
Sol.
S432. Ans.(c)
20 10
Sol. ?≃ × 600 + × 900
3 100 100
3 5 of 260 = ? % of 1248 ? ≃ 210
18
× 260 =? % of 1248
5 S441. Ans.(b)
? % of 1248 = 936 Sol.
? = 75 250 300 100
?≃ × ×
S433. Ans.(e) 15 20 15
5000
Sol. ?≃
5250 3
26 × + √576 = ? ? ≃ 1667
350
? = 26 × 15 + 24 = 414 S442. Ans.(e)
S434. Ans.(b) Sol.
Sol.
1200
? ≃ 11² + 16² – 14² ?≃ × 20 + 400
15
≃ 121 + 256 – 196 ? ≃ 2000
≃ 181
S443. Ans.(a)
S435. Ans.(c) Sol.
Sol. 40 ?
? 36 42
× 440 + × 655 ≃ 228.5
× 200 = 100 × 225 + 100 × 400 100 100
100 ⇒ 176 + 6.55? ≃ 228.5
? × 2 = 81 + 168

For Free Study Material & Quizzes visit : www.mathsbyarunsir.com


For any queries or suggestions email us @ mathsbyarunsir003@gmail.com or whatsapp @ 8881331466
⇒? ≃ 8 ≈129+ 45+ 77 ≈ 251

S444. Ans.(d) S452. Ans.(d)


Sol. Sol.
√? ≃ (1347 + 53) ÷ 100 − 7 68
36√𝑥 + 32√𝑥 = ×𝑥
11
⇒ √? ≃ 14 − 7 68
68√ = 11 × 𝑥
⇒ √? ≃ 7
⇒ ? ≃ 49 𝑥 2 − 121𝑥 = 0
⇒ x = 0, 121
S445. Ans.(e)
Sol. S453. Ans.(a)
2? ≃ 322 × 8 × 342 ÷ (29 × 172 ) Sol.
215 × 172 ≈ 9 + 100 + 64 + 16 ≈ 190
≃ 9
2 × 172
Solutions
2 ≃ 26
?
S454. Ans.(d)
⇒? ≃ 6
Sol.
S446. Ans.(e) ? ≃ 8600 ÷ 420 × 15
Sol. ≃ 307
2
15 60
( × 4800) ÷? ≃ ( × 4) S455. Ans.(c)
100 100
72,000 Sol.
⇒? ≃ ? ≃ 52 – 34 + 22
24 × 24
⇒? ≃ 125 ≃ 40
S447. Ans.(a) S456. Ans.(d)
Sol. Sol.
3 3000 × 750
× 240 ≃?÷ (2 × 1) ?≃ – 1400
20 1000
⇒? ≃ 72 ≃ 850
S448. Ans.(a)
Sol. S457. Ans.(a)
36
?≃ × 10 + 40 Sol.
9
≃ 40+40 150
?≃ × 150 + 150
? ≃ 80 100
S449. Ans.(a) ≃ 375
Sol. S458. Ans.(c)
≈ [4800 − (120)] × 2.5 Sol.
= 11700 990 870 515
?≃ × ×
34 65 207
S450. Ans.(b) ≃ 970
Sol.
? ≃ 4 × 35 + 6 × 25 S459. Ans.(e)
Sol.
≃ 290
? ≃ 32² + 24² – 17²
S451. Ans.(d) ≃ 1024 + 576 – 289
Sol.

For Free Study Material & Quizzes visit : www.mathsbyarunsir.com


For any queries or suggestions email us @ mathsbyarunsir003@gmail.com or whatsapp @ 8881331466
≃ 1311
S469. Ans.(b)
S460. Ans.(e) Sol.
Sol. ? ≈ 400 ÷ 8 × 12 + 245 – 190
? ≃ 6895 + 5025 +600 ≃ 655
≃ 12,520
S461. Ans.(b) S470. Ans.(c)
Sol. Sol.
? ≃ 32 × 12 × 17.5 4 48 52
of ? ≃ × 450 + × 440
≃ 6720 5 100 100
? ≃ 556
S462. Ans.(c)
Sol. S471. Ans.(d)
Sol.
? ≃ 12 × 22 × 18
? ≃ 12 + 26 + 6 – 11
≃ 4752
≃ 33
S463. Ans.(c)
Sol. S472. Ans.(a)
Sol.
? ≃ 17 + 27 + 37 – 13 – 9 43 57
≃ 59 ≃ 60 ?≃ × 800 + × 900
100 100
≃ 857
S464. Ans.(e)
Sol. S473. Ans.(d)
40 60 Sol.
?≃ × 405 + × 610 – 184
100 100 115 84
≃ 344 ?≃ × 560 + × 420
100 100
≃ 997
S465. Ans.(a)
Sol.
5682 × 36 S474. Ans.(b)
?≃
63 × 18 Sol.
≃ 180 ? ≃ 8349 + 654 + 35.45 – 6450
S466. Ans.(c) ≃ 2588
Sol.
S475. Ans.(c)
? ≃ 5 × 13 + 600 Sol.
≃ 665 3 4 4 3
? ≃ × × 700 – × × 198
5 7 9 4
S467. Ans.(a)
≃ 240 – 66
Sol.
≃ 174
? ≃ 370
S476. Ans.(d)
S468. Ans.(b) Sol.
Sol.
? 64 49
340 47 119 ≃ × 750 – × 400
?≃ × × 10 100 100
34 510 94 ⇒ ? ≃ 2840
≃ 1.2

For Free Study Material & Quizzes visit : www.mathsbyarunsir.com


For any queries or suggestions email us @ mathsbyarunsir003@gmail.com or whatsapp @ 8881331466
S477. Ans.(a) ? ≃ 770
Sol.
24 30 S485. Ans.(a)
?≃ × 260 + × 710 – 280 Sol.
100 100
≃ –5 ? ≃ 151 – 420 + 650
? ≃ 381
S478. Ans.(c)
In options, the nearest value is 380
Sol.
? ≃ 4650 ÷ 100 + 3750 ÷ 10 – 3660 ÷ 40 S486. Ans.(a)
≃ 330 Sol.
? ≃ 1300 ÷ 20 × 25 + 400
S479. Ans.(d)
? ≃ 2025
Sol.
7430 S487. Ans.(c)
?≃ × 10
300 Sol.
? ≃ 247.66
? ≃ 248 ? ≃ √5184 ÷ √36
? ≃ 72 ÷ 6
S480. Ans.(b) ? ≃ 12
Sol.
15 13 S488. Ans.(e)
?≃ × 725 + × 643 Sol.
100 100
? ≃ 192.34 ? ≃ 48 × 35 ÷ 3
? ≃ 192 ? ≃ 560

S481. Ans.(b) S489. Ans.(d)


Sol. Sol.
? ≃ (36)2 − (21)2 ? = √9026 × (5.96)2
? ≃ 15 × 57 ≃ 95 × 62
? ≃ 855 ?≃ 3420

S482. Ans.(e) S490. Ans.(a)


Sol. Sol.
3
? ≃ √729 × √484 4734.96 – 3454.03 – 1612.86 =? – 1611.43
≃ 9 × 22 ⇒ ? ≃ 4735 – 3454 – 1613 + 1611
≃ 198 ? ≃ 1279
? ≃ 1280
S483. Ans.(a)
Sol.
S491. Ans.(e)
496 Sol.
?≃ × 0.5
0.6 323 971 56
? ≃ 413.33 ?= × ×
? ≃ 413 55 251 61
325 970 55
≃ × ×
S484. Ans.(c) 55 250 60
1261
Sol. ≃ ⇒ ? ≃ 21
150 200 91 60
?≃ × ×
17 13 16 S492. Ans.(c)
? ≃ 772.05 Sol.

For Free Study Material & Quizzes visit : www.mathsbyarunsir.com


For any queries or suggestions email us @ mathsbyarunsir003@gmail.com or whatsapp @ 8881331466
? ≃ 133 × 3 − 112 + 74 √899 × (12.005) ² + ? = 5000
≃ 399 − 38 ⇒ 30 ×12² + ? =5000
? ≃ 361
⇒ ? = 680
The nearest value to 361 in the options is 360
S499. Ans.(b)
S493. Ans.(b)
Sol. Sol. ? ≃ 8538 − 2416 − 222
≃ 5900
? ≃ 32 × 2800 ÷ 550 + 120
280
?≃32 × 55 + 120 S500. Ans.(c)
1020
?≃32 × 5 + 120 Sol. ? ≃ 60
?≃ 280 ≃ 17

S494. Ans.(a) S501. Ans.(d)


Sol. Sol. ? = 9.09 ≃ 9
160
2775 ×
√? = 5550 S502. Ans.(a)
√? = 80 Sol. ? ≃ 16 × 31 × 9
⇒? = 6400 ≃ 4464

S503. Ans.(c)
S495. Ans.(e) 530 88
Sol. Sol. ?× 100 ≃ 100 × 450
162 ? ≃ 75
252 × × 39 = 130 ×? ²
8 × 15
252 × 162 × 39 S504. Ans.(b)
⇒?2 =
8 × 15 × 130 Sol. ? ≃ 21 + 5 × 8 + 10
125 × 16
⇒ ?² = ≃ 71
5
⇒ ? ² = (5 × 4)²
S505. Ans.(b)
⇒ ? ² = 20²
75 40 48
⇒ ? = 20 Sol. ≈ 100 × 256 + = 100 × 650
√𝑥
40
S496. Ans.(e) ≈ 192 + = 312
√𝑥
Sol. 40
≈ = 120
√𝑥
72 35 ? 2
× 1200 + × 270 = × 600 40
100 100 100 ≈ 𝑥 = (120)
⇒ ? = 159.75 1
≈𝑥=9
⇒ ? ≃ 160
S506. Ans.(a)
S497. Ans.(c)
Sol. ≈ 294 × 𝑥8 − 15 × 120𝑥 = 255
Sol. 2
≈ 36.75𝑥 − 24𝑥 = 127.5
88 ? ≈ 12.75𝑥 = 127.5
× 450 ≃ × 530
100 100 ≈ 𝑥 = 10
⇒ ? ≃ 74.716
⇒ ? ≃ 75 S507. Ans.(e)
2009
Sol. 322 − 122 + ≃ 900
?
S498. Ans.(a) 2009
Sol. ⇒? ≃ 20

For Free Study Material & Quizzes visit : www.mathsbyarunsir.com


For any queries or suggestions email us @ mathsbyarunsir003@gmail.com or whatsapp @ 8881331466
⇒ ? ≃ 100 ? ~ 17,550
Nearest value 17557
S508. Ans.(b)
1575 S515. Ans.(a)
Sol. ? ≃ + 24 × 16
45 Sol.
≃ 419 4 5
≃ 420 × ? ≃ √3 × 51 + 30 × 3 + 367
5
S509. Ans.(d) ≃ 3 + 367
Sol. 370 × 5
⇒? ≃
1885 ÷ 145 + 7 + (? )2 ≃ 69 4
≈ 13 + 7 + (? )2 = 69 ? ≃ 462.5
≈?2 = 69 − 20 = 49
S516. Ans.(d)
∴ ? = √49 = ±7 Sol.
S510. Ans.(b) ? ≃ 14,869
Sol.
S517. Ans.(e)
(3.2)2 + (9.8)2 + (8.13)2 + (4.24)2
Sol.
≃ 32 + 102 + 82 + 42
3 238 46
≈ 9 + 100 + 64 + 16 ?≃ × × 388 + × 445
≈ 190 7 291 100
≃ 136 + 204.7
≃ 341

S511. Ans.(e) S518. Ans.(b)


Sol. Sol.
≃ 2418 + 88 ÷ 14 × 6 88 14
?≃ × 550 + × 850
≈ 2418 + 6 × 6 100 100
≃ 2454 ≃ 603 ≃ 600
≈ 2455 S519. Ans.(d)
Sol.448 ÷ 28 × 5
S512. Ans.(c) = 16 × 5 = 80
Sol.
3
√13824 + (47)2 S520. Ans.(d)
≈ 24 + 472 Sol.1680 ÷ 15 × 5
= 2209 + 24 = 112 × 5 = 560
≈ 2230
S521. Ans.(d)
S513. Ans.(b) Sol. 5238 − 6630 + 7154 − 2205
Sol.
= 12392 − 8835 = 3557 ≈ 3558
≈ 65% of 8920 + ? % of 5320 = 6830
≈ 5320 ×? = (6830 − 5798) × 100 S522. Ans.(e)
? ≈ 20 460×850
Sol. 100 + 2.665 × 6284 − 1486
62 4 = 3910 − 1486 + 2.66 × 6284
 840 ? ≈ 19140
S 514. (c) 550 + 100 100
Sol . S523. Ans.(d)
18.5+520.8~ 0.04×? Sol. (9321 + 5406 + 1001) ÷ (498 + 929 + 660)
702~0.04×? = 15728 ÷ 2087 ≈ 7.5

For Free Study Material & Quizzes visit : www.mathsbyarunsir.com


For any queries or suggestions email us @ mathsbyarunsir003@gmail.com or whatsapp @ 8881331466
Sol.
S524. Ans.(b) ?≈
67×800
− 231 +
23×790
100 100
Sol. ? ≃ 13 × 7 + 852 × 11 ≈ 536 − 231 + 181.7
≃ 79566 ≈ 490
Sol.534
45.96 × 29.98 + 167.98 × 1/3.98 × 1/1.92 – 120.99 = ?
S525. Ans.(c) ⇒ 46 × 30 + 168 × 1/4 × 1/2 – 121 = ?
12
Sol. ? 433 − 269 ÷ (100 × 75) ⇒ 1380 + 21 – 121 = ?
269
≃ 433 − 9 ⇒ ? = 1280
≃ 433 − 30 Sol.535
≃ 403 Given expression is
[(6.973 × 7.073) / (7.172 × 6.94)3] × 6.978 = 7.13?
S526. Ans.(d) After approximation
900
Sol. ? ≃ + 224 ⇒ [(73 × 73) / {72 × 7}3] × 78 = 7?
45
≃ 244 ⇒ [(73 × 73)/(7(2 × 3) × 73)] × 78 = 7?
[(73 × 73)/(76 × 73)] × 78 = 7?
S527. Ans.(a) ⇒ 1/73 × 78 = 7?
Sol. 2425
?
≃ 229 + 142 − 182 ⇒ 7? = 7(8 - 3)
≃ 101
⇒ ? ≃ 24
⇒?=5

S528. Ans.(c) Sol.536


40 650
Sol. ? ≃ 45 − 100 × 13 Given expression is
91.99 – [70.97 + {4.04 – (4.95 – 1.99))}] = ?
? ≃ 25
⇒ 92 – [71 + {4 – (5 – 2)}] = ?
S529. Ans.(c) ⇒ 92 – [71 + {4 – 3}] = ?
Sol. ⇒ 92 – [71 + 1] = ?
? ≃ 25 + 27 ⇒ 92 – 72 = ?
≃ 52 ⇒ 20 = ?
Solution 537 d
S530. Ans.(a) Given expression is
Sol. (19.99)2 × [35.98 × 24.04] ÷ (11.98 × 1.982) = ?
69 33
?≃ × 700 + × 400 ⇒ (20)2 × [36 × 24] ÷ (12 × 22) = ?
100 100
≃ 483 + 132 ⇒ 400 × [36 × 24] ÷ (12 × 4) = ?
≃ 615 ⇒ 400 × [864] ÷ (48) = ?
S531. Ans.(d) ⇒ 400 × 18 = ?
Sol. ∴ ? = 7200
(9320 + 5400 + 1000) Sol.538(e)
?≃ Given expression is
(500 + 930 + 660)
≃ 7.5 51.98 + 105.98 – ? × 2.93 = 130.91
⇒ 52 + 106 – ? × 3 = 131
S532. Ans.(d)
Sol. ⇒ 158 – ? × 3 = 131
64 ? ⇒ 158 – 131 = ? × 3
× 8924 + × 5324 ≃ 6828
100 100 ⇒ 27 = ? × 3
⇒ ? ≃ 21 ⇒9=?
Sol.539 (e)
S533. Ans.(a)

For Free Study Material & Quizzes visit : www.mathsbyarunsir.com


For any queries or suggestions email us @ mathsbyarunsir003@gmail.com or whatsapp @ 8881331466
Given expression is ⇒ ? = 10
(7.072 - 30.98) × 2.98 ÷ (7.982 - 45.96) = ? Sol.545 (e)
⇒ (72 - 31) × 3 ÷ (82 - 46) = ? Given expression is
⇒ (49 - 31) × 3 ÷ (64 - 46) = ? 24.95 - {16.98 - 11.98 ÷ (5.15 + 17.98 - 16.98)} = ?
⇒ (18 × 3)/18 = ? ⇒ 25 - {17 - 12 ÷ (5 + 18 - 17)} = ?
⇒?=3 ⇒ 25 - {17 - 12 ÷ (5 + 18 - 17)} = ?
Sol.540 (a) ⇒ 25 - {17 - 12 ÷ 6} = ?
Given expression is ⇒ 25 - {17 - 2} = ?
849.99 ÷ 33.94 + 4.953 = 5.96 × 4.85 × ? ⇒ 25 - 15 = ?
⇒ 850 ÷ 34 + 53 = 6 × 5 × ? ∴ ? = 10
⇒ 25 + 125 = 6 × 5 × ? Sol.546 (c)
⇒ 150 = 6 × 5 × ? 21.03 × 6.97 +12% of 401.09 - 2/7 of 699.81 = ? - 105.07
∴?=5 Taking approximate values, we get
21 × 7 +12% of 400 - 2/7 of 700 = ? - 105
Sol.541 (c) ⇒ 21 × 7 + 12/100 × 400 - 2/7 × 700 = ? - 105
Given expression is ⇒ 147 + 48 - 200 = ? - 105
{74.95 - (118.99 - 128.08 ÷ 8.08 × 6.98)} = ? ⇒ ? = 147 + 48 - 200 + 105 = 100
⇒ {75 - (119 - 128 ÷ 8 × 7)} = ?
⇒ {75 - (119 - 16 × 7)} = ? Sol .547 (c)
⇒ {75 - (119 - 112)} = ? 104.042 - 96.022 + 53.992 - 46.072 = 100 + ?
⇒ {75 - 7} = ? Taking approximate values, we get
1042 - 962 + 542 - 462 = 100 + ?
∴ ? = 68
Sol.542 (c) ⇒ (104 + 96)(104 - 96) + (54 + 46)(54 - 46) = 100 + ?
Given expression is ⇒ 200 × 8 + 100 × 8 = 100 + ?
11.98 × 84.98 ÷ 4.95 + 39.99% of 349.99 = ? ⇒ 1600 + 800 = 100 + ?
⇒ 12 × 85 ÷ 5 + 40% of 350 = ? ⇒ 2400 = 100 + ?
⇒ 12 × 85 ÷ 5 + 140 = ? ∴ ? = 2400 - 100 = 2300
⇒ 12 × 17 + 140 = ?
Sol 548. (e)
⇒ 204 + 140 = ?
2% of (298.99 + 15.982 - 255.11) = ?
Sol.543 (e) Taking approximate values, we get
Given expression is 2% of (300 + 162 - 255) = ?
16.992 - 88.99 - 13.942 + 3.94 = 1.922 × ? ÷ 10.98 ⇒ 2% of (300 + 256 - 255) = ?
⇒ 172 - 89 - 142 + 4 = 22 × ? ÷ 11 ⇒ 2/100 × (300 + 256 - 255) = ?
⇒ 289 - 89 - 196 + 4 = 4 × ? /11 ⇒ 2/100 × (300 + 256 - 255) = ?
⇒ 8 × 11 / 4 = ? ⇒ 2/100 × (300 + 1) = ?
∴ ? = 22 ⇒ ? = 2/100 × (301) ≈ 6

Sol.544 (c) Sol 549 (d)


Given expression is 13.97 × √9.08 + 300.04 - 99.97 = ?
11.922 + 15.982 – ?2 = 10.102 × 3.03 Taking approximate values, we get
⇒ 122 + 162 – ?2 = 102 × 3 ⇒ 14 × √9 + 300 - 100 = ?
⇒ 144 + 256 – ?2 = 100 × 3 ⇒ 14 × 3 + 200 = ?
⇒ ?2 = 400 – 300 = 100 ⇒ 42 + 200 = ?
⇒? = √100 = √(10 × 10) ∴ ? = 242

For Free Study Material & Quizzes visit : www.mathsbyarunsir.com


For any queries or suggestions email us @ mathsbyarunsir003@gmail.com or whatsapp @ 8881331466
2
Ans 550. (b) 15 170
( × 4800) ÷? ≃ ( × 7)
Solution 100 100
18.02 - (? - 19.01) + 10.982 = 120.04 72,000
⇒? ≃
Taking approximate values, we get 14,161
⇒? ≃ 5
18 - ? + 19 + 121 = 120
⇒ ? = 121 - 120 + 19 + 18 S558. Ans.(a)
∴ ? = 38 Sol.
3
× 240 ≃?÷ (1.6 × 0.5) ⇒? ≃ 29
20
Ans 551. (d)
Let √50 ≈ √49 S559. Ans.(e)
Now, the given expression: Sol.
8787 ÷ 343 × √50 =? ? ≃ 36 ÷ 9 ÷ 10 + 39.4
⇒ ? = 8787 ÷ 343 ×√50 ≃ 39.8 ? ≃ 40
⇒ ? = 25.61 ×√50
⇒ ? ≈ 25 × 7 Ans 560. (d)
Given expression is
⇒ ? ≈ 175
79.98% of 299.99 + 39.94% of 349.99 = ? + 59.99% of 399.99
Ans552. (a)
8399.99 ÷ 12.02 - 199.99 = ? ⇒ 80% of 300 + 40% of 350 = ? + 60% of 400
⇒ 8400 ÷ 12 - 200 = ? ⇒ 240 + 140 = ? + 240
⇒ 700 - 200 = 500 ⇒ 380 = ? + 240
Ans553. (a)
⇒ √2115.88 × (4.96)2 + 896.32 = ? ∴ ? = 380 - 240 = 140
Ans561. (e)
⇒ √2116 × (4.96)2 + 896.32 = ?
⇒ 46 × (5)2 + 896 = ? Given expression is
⇒ 1150 + 896 = ? 3.03 – (9.09 – 2.93 × 7.98 ÷ 1.92) = ?
⇒ ? = 2046 ⇒ 3 – (9 – 3 × 8 ÷ 2)
Ans.554 (a) ⇒ 3 – (9 – 3 × 4)
666.66 + 66.66 + 16 + 0.66 + 0.06 ÷ 0.001
⇒ 3 – (9 – 12)
= 666.66 + 66.66 + 16 + 0.66 + 60
= 809.98 ⇒3+3
≈ 810 ⇒6

S555. Ans.(d) Solution 562 d


Sol. Given expression is
√? ≃ (1347 + 47) ÷ 100 − 7
241.98 – [18.962 – (227.93 + 171.98) ÷ 1.92] = ?2
⇒ √? ≃ 13.94 − 7 ⇒ 242 – [192 – (228 + 172) ÷ 2] = ?2
⇒ √? ≃ 6.94 ⇒ 242 – [192 – 400 ÷ 2] = ?2
⇒ ? ≃ 49
⇒ 242 – [192 – 200] = ?2
S556. Ans.(e) ⇒ 242 – [361 – 200] = ?2
Sol. ⇒ 242 – 161 = ?2
2? ≃ 322 × 8 × 342 ÷ (29 × 172 ) ⇒ 81 = ?2
215 × 172
≃ 9 ∴?=9
2 × 172
2? ≃ 26 Solution 563 e
⇒? ≃ 6 Given expression is
S557. Ans.(e) (21.93)2 - (7.98)3 - √1295.96 + 63.98 = ?
Sol.

For Free Study Material & Quizzes visit : www.mathsbyarunsir.com


For any queries or suggestions email us @ mathsbyarunsir003@gmail.com or whatsapp @ 8881331466
⇒ (22)2 - (8)3 - √1296 + 64 = ? Sol.571 e
⇒ 484 - 512 - 36 + 64 = ?
⇒ 548 - 548 = ?
∴?=0
Solution 564 c
Given expression is
143.99 ÷ (5.93)2 × 7.92 – 23.97 = √?
⇒ 144 ÷ (6)2 × 8 – 24 = √?
⇒ 144 ÷ 36 × 8 – 24 = √?
⇒ 4 × 8 – 24 = √?
Ans.572 b
⇒ 32 – 24 = √?
⇒ 8 = √?
∴ ? = 64

S565. Ans.(d)
Sol.
? ≈ 150% of 30 + 25% of 140 = 45 + 35 = 80

S566. Ans.(b)
Sol.
4
? ≈ √0.0128 ÷ 6 × 972 = 4√(0.0064 × 2) × (324 × 3) ÷ 6 = Ans.573 b
√0.08 × 18 = √1.44 = 1.2 (18.64)2 + (8.32)2 ÷ (1.85)2 = ?2 + (1.23)2
S567. Ans.(a) ⇒ (19)2 + (8)2 ÷ (2)2 = ?2 + (1)2
Sol. ⇒ 361 + 64 ÷ 4 = ?2 + 1
?2 ≈ 1445 × 80 ÷ 100 = (172 × 5) × (42 × 5) ÷ (22 × 52 ) ⇒ 361 + 16 = ?2 + 1
Or ? ≈ 17 × 4 ÷ 2 = 34 ⇒ 377 - 1 = ?2
S568. Ans.(e) ⇒ ? = √376 = 19.39 ≈ 19
Sol.574 c
Sol.
Given expression is,
?3 ≈ (75)2 − (25)2 − (30)2 = 5625 − 625 − 900 = 4100
Or ? ≈ 16 ⇒ (453.81 + 29.89)1/2 ÷ 10.85 = (0.1 × ?) ÷ 4
We can write the given values as:
S569. Ans.(c) ⇒ (454 + 30)1/2 ÷ 10.85 = (0.1 × ?) ÷ 4
Sol. ⇒ (484)1/2 ÷ 11 = (0.1 × ?) ÷ 4
18 15 11 32 8 33 12
?= 5
× 4
× 9
− 5
÷ 3
= 2
− 5
= 16.5 − 2.4 = 14.1 ≈ 14 ⇒ 22 ÷ 11 = (0.1 × ?) ÷ 4
⇒ 2 × 4 = 0.1 × ?
Sol.570 a
⇒ ? = (2 × 4)/0.1 = 80
Given expression is,
⇒ ? = 80
⇒ 47.88 + 84.07 – 99.95 = 7.86 × ?
⇒ 48 + 84 – 100 = 7.86 × ? Ans.575 d
⇒ 132 – 100 = 7.86 × ? 49.99% of (152.01 + 63.99 ÷ 2) × 7.01 = 46 × ?
⇒ 32 = 7.86 × ? ⇒ 50% of (152 + 64/2) × 7 = 46 × ?
⇒ 32 = 8 × ? ⇒ 50/100 × (152 + 32) × 7 = 46 × ?
⇒ ? = 32/8 = 4 ⇒ 184/2 × 7 = 46 × ?
⇒?=4 ⇒ 46 × ? = 92 × 7⇒ ? = 14

For Free Study Material & Quizzes visit : www.mathsbyarunsir.com


For any queries or suggestions email us @ mathsbyarunsir003@gmail.com or whatsapp @ 8881331466
⇒ 26 + 12 = √x + 18
Ans.576 b
⇒ √x = 26 + 12 - 18
59.99% of 310 – 102.97 + 30.08% of 260 = ?
⇒ √x = 20
Approximating the values to the nearest integer:
(?) = 310 × 60/100 – 103 + 260 × 30/100 ⇒ x = 400
(?) = 186 – 103 + 78
(?) = 161 SOL.582 D
Given expression is
Ans.577 c (683.94 + 387.01) ÷ 2.93 + 162.93 - 163.99 = ?
Given Expression, ⇒ (684 + 387) ÷ 3 + 163 - 164 = ?
79.74 ÷ 4.25 + 772.80 = 1009.14 - (?)3 ⇒ (1071) ÷ 3 + 163 - 164 = ?
Given expression becomes, ⇒ 357 + 163 - 164 = ?
⇒ 80 ÷ 4 + 773 = 1009 - (?)3 ⇒ ? = 356
⇒ 20 + 773 = 1009 - (?)3 Sol.583
⇒ (?)3 = 1009 - 793 Given expression is
⇒ (?)3 = 216 4499.95 + 499.95 ÷ 24.95 – 919.92 = 17.91 × ?
Ans.578 e ⇒ 4500 + 500 ÷ 25 – 920 = 18 × ?
2998.88 ÷ 101.02 – 4.999 × 2.8998 + 6.978 = ? ⇒ 4500 + 20 – 920 = 18 × ?
Taking approximate values, ⇒ 4520 – 920 = 18 × ?
⇒ 3000 ÷ 100 – 5 × 3 + 7 = ? ⇒ 3600 = 18 × ?
⇒ 30 – 15 + 7 = ? ⇒ ? = 200
⇒ 37 – 15 = ? Sol.584 e
∴ ? = 22 Given expression is
(979.8 + 888.08) ÷ 4.04 + 632.93 = 683.84 + 25.96 × ?
Ans.579 a ⇒ (980 + 888) ÷ 4 + 633 = 684 + 26 × ?
Given expression is, ⇒ 1868 ÷ 4 + 633 = 684 + 26 × ?
(171.94 ÷ 7.99) × 12.01 = ? × 5.81 + 23.99
⇒ 467 + 633 = 684 + 26 × ?
⇒ (172 ÷ 8) × 12 = ? × 6 + 24
⇒ 1100 = 684 + 26 × ?
⇒ 21.5 × 12 = ? × 6 + 24
⇒ 1100 - 684 = 26 × ?
⇒ 258 = ? × 6 + 24
⇒ 416 = 26 × ?
⇒ 258 - 24 = ? × 6
⇒ ? = 416 /26
⇒ ? × 6 = 234
⇒ ? = 16
⇒ ? = 234/6 Ans585. C
⇒ ? ≈ 39 Given expression is
96.98 + 709.99 – 142.93 = 3.98 × ?
Sol.580 b ⇒ (97 +7 10) – 143 = (4 × ?)
Given expression is
⇒ 807 – 143 = (4 × ?)
255.96 ÷ 1.923 × ? = 15.66% of 2999.99
⇒ ? = (807 - 143)/4 = 664/4 = 166
⇒ 256 ÷ 23 × x = 16% of 3000 Sol.586 d
⇒ 256 ÷ 8 × x = 3000 × 16/100 Given expression is
⇒ 32 × x = 480 325.05 – [(11.92)2 – 74.95] = (?)2 – 67.8
⇒ x = 480/32 = 15 ⇒ 325 – [(12)2 – 75] = (?)2 – 68
Sol.581 e ⇒ 325 – [144 – 75] + 68 = (?)2
Given expression is
⇒ 325 – 69 + 68 = (?)2
598.01 ÷ 22.93 + √143.94 = √? + 287.88 ÷ 15.89
⇒ 324 = (?)2
598 ÷ 23 + √144 = √x + 288 ÷ 16

For Free Study Material & Quizzes visit : www.mathsbyarunsir.com


For any queries or suggestions email us @ mathsbyarunsir003@gmail.com or whatsapp @ 8881331466
⇒ 18 = ? ? = 214

Sol.587 594. Ans.(a)


Given expression is Sol.
50×25
620.99 ÷ 26.97 × 1.92 – 36.97 = √? 25x =
10
x=5
⇒ 621 ÷ 27 × 2 – 37 = √? 595. Ans.(b)
Sol.
⇒ 23 × 2 – 37 = √? x 1
⇒ √? = 46 – 37 = 9 × 5600 × = 8 × 41 + 500 – 128
100 2
700
⇒ ? = 92 = 81 x=
Sol.588 d 28
x = 25
Given expression is
673.8 – 457.2 + 518.9 = ? + (18.89)2 596. Ans.(d)
⇒ 674 – 457 + 519 = ? + (19)2 Sol.
⇒ ? = 674 – 457 + 519 - (19)2 26
56 ×? + 100 × 450 + √16 = (31)2
⇒ ? = 674 – 457 + 519 - 361 56 ×? = 961 – 4 – 117
⇒ ? = 375 ?=
840
56
589. Ans.(d) ? = 15
Sol.
44 7.5
597. Ans.(b)
× 2.5 + 3 = √? Sol.
4
√? = 11 × 3 + 3 546
+
56
× 550 =
28
× 1250
? 100 100
36 = √? 546
= 350– 308
?
? = 1296 546
?= , ? = 13
42

590. Ans.(c) 598. Ans.(c)


Sol. Sol.
2 1
(12)3 + (16)2 – √?+ (26)2 = (36)2 – 80
(? )3×2 + (2)10 + (5)3 = 1157
1 1728 + 256– √?+ 676 = 1296 − 80
(? )3 + 1024 + 125 = 1157 √? = 1444
1
? = 38
(? )3 = 1157 – 1149 = 8
? = 512 599. Ans.(a)
Sol.
591. Ans.(e) ? 12
Sol. 8% of 450 + 6% of 1450 = 25% of ? 100
× 340 + 100 × 2175 + √3721 = (21)2
?
25% of ? = 36 + 87 100
× 340 + 261 + 61 = 441
25 ?
×? = 123 × 340 = 119
100 100
123 × 100 ? = 35
?= = 492
25
600. Ans.(b)
592. Ans.(b) Sol.
Sol. ? = 84% of 1650 + 20% of 870 25
547 + 243 – ? = 100 × 2584
? = 1386 + 174 = 1560
790 – ? = 646
593. Ans.(a) ? = 790 – 646
Sol. ? = 64% of 250 + 36% of 150 ? = 144
? = 160 + 54

For Free Study Material & Quizzes visit : www.mathsbyarunsir.com


For any queries or suggestions email us @ mathsbyarunsir003@gmail.com or whatsapp @ 8881331466
For Free Study Material & Quizzes visit : www.mathsbyarunsir.com
For any queries or suggestions email us @ mathsbyarunsir003@gmail.com or whatsapp @ 8881331466

You might also like